X



トップページ物理
1002コメント356KB
■ちょっとした物理の質問はここに書いてね245■ ・
レス数が1000を超えています。これ以上書き込みはできません。
0001ご冗談でしょう?名無しさん
垢版 |
2019/11/10(日) 21:55:32.47ID:T5g+6Suw
★荒らし厳禁、煽りは黙殺
★書き込む前に   >>2   の注意事項を読んでね
★数式の書き方(参考)はこちら   >>3-5   (予備リンク:   >>2-10    )
===質問者へ===
重要 【 丸 投 げ 禁 止 】

・質問する前に
1. 教科書や参考書をよく読む
2. http://www.google.com/
  などの検索サイトを利用し、各自で調べる
3. 学生は自分の学年、物理科目の履修具合を書く
4. 宿題を聞くときは、どこまでやってみてどこが分からないのかを書く
5. 投稿する前に、ちゃんと質問が意味の通る日本語か推敲する、曖昧な質問文には曖昧な回答しか返せない
・「力」「エネルギー」「仕事」のような単語は物理では意味がはっきり定義された言葉です、むやみに使うと混乱の元
・質問に対する回答には返答してね、感謝だけでなく「分からん」とかダメOK
・質問するときはage&ID表示推奨
・高度すぎる質問には住人は回答できないかもしれないけれど、了承の上での質問なら大歓迎

===回答者へ===
・丸投げは専用スレに誘導
・不快な質問は無視、構った方が負け
・質問者の理解度に応じた適切な回答をよろしく
・単発質問スレを発見したらこのスレッドへの誘導をよろしくね
・逆に議論が深まりそうなら新スレ立てて移動するのもあり
・板違いの質問は適切な板に誘導を
・不適切な回答は適宜訂正、名回答は素直に賞賛
0002ご冗談でしょう?名無しさん
垢版 |
2019/11/10(日) 21:55:49.33ID:???
数式の書き方の例 ※適切にスペースを入れると読みやすくなります
●括弧: (), [], {}を適切に入れ子にして分かりやすく書く
●スカラー: a,b,...,z, A,...,Z, α,β,...,ω, Α,Β,...,Ω,...(「ぎりしゃ」「あるふぁ〜おめが」で変換)
●ベクトル: V=(v1,v2,...), |V>,V↑, (混乱しないならスカラーの記号でいい。通常は縦ベクトル)
●テンソル: T^[i,j,k...]_[p,q,r,...], T[i,j,k,...; p,q,r,...] (上下付き1成分表示)
●行列: M[i,j], I[i,j]=δ_[i,j] M = [[M[1,1],M[2,1],...], [M[1,2],M[2,2],...],...], I = [[1,0,0,...],[0,1,0,...],...]
(右は全成分表示。行または列ごとに表示する。例:M=[[1,-1],[3,2]])
●対角行列: diag(a,b) = [[a,0],[0,b]]
●転置行列・随伴行列:M^T, M†("†"は「だがー」で変換可) ●行列式・トレース:|A|=det(A), tr(A)
●複号: a±b("±"は「きごう」で変換可)
●内積・外積: a・b, a×b
●関数・汎関数・数列: f(x), F[x(t)] {a_n}
●平方根: √(a+b) = (a+b)^(1/2) = sqrt(a+b) ("√"は「るーと」で変換可)
●指数関数・対数関数: exp(x+y)=e^(x+y) ln(x)=log_e(x) (底を省略して単にlogと書いたとき多くは自然対数)
括弧を省略しても意味が容易に分かるときは省略可: sin(x) = sin x
●三角関数、逆三角関数、双曲線関数: sin(a), cos(x+y), tan(x/2), asin(x)=sin^[-1](x), cosh(x)=[e^x+e^(-x)]/2
●絶対値:|x| ●ノルム:||x|| ●共役複素数:z^* = conj(z)
●階乗:n!=n*(n-1)*(n-2)*...*2*1, n!!=n*(n-2)*(n-4)*...
0003ご冗談でしょう?名無しさん
垢版 |
2019/11/10(日) 21:56:02.53ID:???
質問・回答に標準的に用いられる変数の例

a:加速度、昇降演算子 A:振幅、ベクトルポテンシャル B:磁束密度 c:光速 C:定数、熱・電気容量
d:次元、深さ D:領域、電束密度 e:自然対数の底、素電荷 E:エネルギー、電場
f:周波数 f,F:力 F:Helmholtzエネルギー g:重力加速度、伝導度
G:万有引力定数、Gibbsエネルギー、重心 h:高さ、Planck定数 H:エンタルピー、Hamiltonian、磁場
i:虚数単位 i,j,k,l,m:整数のインデックス I:電流、慣性モーメント j:電流密度・流束密度
J:グランドポテンシャル、一般の角運動量 k:バネ定数、波数、Boltzmann定数 K:運動エネルギー
l,L:長さ L:Lagrangian、角運動量、インダクタンス m,M:質量 n:物質量 N:個数、トルク
M:磁化 O:原点 p:双極子モーメント p,P:運動量、圧力 P:分極、仕事率、確率 q:波数
q,Q:一般化座標、電荷 Q:熱 r:距離 R:抵抗、気体定数 s:スピン S:エントロピー、面積 t,T:時間 T:温度
U:ポテンシャル、内部エネルギー v:速度 V:体積、ポテンシャル、電位
W:仕事、状態数 x,y,z:変数、位置 z:複素変数 Z:分配関数

β:逆温度 γ:抵抗係数 Γ:ガンマ関数 δ:微小変化 Δ:変化 ε:微小量、誘電率 θ:角度 κ:熱伝導率
λ:波長、固有値 μ:換算質量、化学ポテンシャル、透磁率 ν:周波数 Ξ:大分配関数 π:円周率 ρ:(電荷)密度、抵抗率
σ:スピン τ:固有時 φ:角度、ポテンシャル、波動関数 ψ:波動関数 ω:角振動数 Ω:状態密度
0004ご冗談でしょう?名無しさん
垢版 |
2019/11/10(日) 21:56:35.92ID:???
ちょっとした質問を書くスレです

お願い事をするスレでも、質問に答えてもらえるスレでもありません
0007ご冗談でしょう?名無しさん
垢版 |
2019/11/11(月) 09:53:53.25ID:???
ちょっとした事で行き詰った物理問題に答えるという意味だろ
詳しくは専門教科書を読むかググレばよい
0014ご冗談でしょう?名無しさん
垢版 |
2019/11/11(月) 21:39:09.21ID:???
力を先に定義すれば質量の定義ですし、質量を先に定義すれば力の定義ですし
そもそも他のやり方で定義することもできるかもしれませんね
0022ご冗談でしょう?名無しさん
垢版 |
2019/11/12(火) 01:18:54.69ID:???
f=ma は「ニュートンの運動方程式」の定義
現実には高速運動の状態で成り立たないから妄想するだけ無駄。
0023ご冗談でしょう?名無しさん
垢版 |
2019/11/12(火) 01:35:53.49ID:???
>>15
Hamilton・Jacobiの方法は、相空間の半分だけの循環座標と対応する保存量を一挙に求めてしまおうという、解析力学の最終奥義
0027ご冗談でしょう?名無しさん
垢版 |
2019/11/12(火) 11:21:44.50ID:???
3つの物理変数の方程式が定義されているなら、2つの物理量が決まれば残りの1つの値が決まるだけ。
0028ご冗談でしょう?名無しさん
垢版 |
2019/11/12(火) 12:22:33.62ID:RjA4yf0v
でも溶けないでしょ
0030ご冗談でしょう?名無しさん
垢版 |
2019/11/12(火) 13:45:10.66ID:???
>>11
大元から説明すると、日の出〜日没や振り子などの周期現象から時間概念が抽出され、
物差しを移動して測っても不変な事から距離の概念が抽出される。
それから速度や加速度も時間・距離からの誘導概念として存在する。
以上を前提としてガリレオの斜面実験を見れば、斜面の傾きによって重りを押し上げる手応えが変わり、
手を離した時の加速度が変わる。
その手応え=力と加速度の関係式に重りに依存する係数=質量を加えたものが
ma = F である。
つまり、力と質量は同時に定義され、そのような同時定義が可能であることがガリレオの結論である。
0031ご冗談でしょう?名無しさん
垢版 |
2019/11/12(火) 13:52:26.64ID:???
おっと、他の実験で質量が物体の保存量とか、力が伝達できる独自の物理量とか
付属情報が色々あるよ
0032ご冗談でしょう?名無しさん
垢版 |
2019/11/12(火) 15:05:58.12ID:???
>>30
俺様説だな
ma = F という式は俺様説と同様に後世の学者が書き直したものだ。

ニュートンの著作プリンキピアによれば、元々の運動方程式は dp/dt = f の形であり
運動量p をニュートンは p=mv とした。
dp/dt = f は、運動量と力の方程式ということになる、特殊相対性理論でも同じ形式である。
天才ニュートンの先見性と偉大さが判る。(マックスウェルの方程式についても同様なことがいえる)
後世の学者が、一般人向けに書き直したせいで方程式の先見性が失われた。
0034ご冗談でしょう?名無しさん
垢版 |
2019/11/12(火) 18:20:45.79ID:pjt2lLbS
なんで角運動量lのベクトルの絵が、運動量pで運動している粒子から生えてるんじゃなくて、位置rの根元から生えてる絵ばっかりなの?
角運動量を持ってるのは粒子じゃなくて原点なの?
ベクトルだから平行移動できるという理由じゃなくて、そうする意味を教えてほしい
0035ご冗談でしょう?名無しさん
垢版 |
2019/11/12(火) 18:39:27.10ID:???
>>34
座標の取り方で角運動量変わるかやってみ
変わるんなら角運動量は物質がもってるものではないってことやん
0038ご冗談でしょう?名無しさん
垢版 |
2019/11/12(火) 21:33:44.73ID:???
>>32 >>37
ニュートン力学の角運動量と力のモーメントの式は、運動方程式 dp/dt = f から導出され
相対運動する慣性座標どうしの座標変換でも不変な式になる。
力のモーメントがゼロで回転運動してない物体は他の慣性系から観測してもゼロである。
特殊相対性理論でも運動方程式 dp/dt = f が成り立つなら同じことが言えないか?
答えは Yes
回転してない物体が回転するたぐいの特殊相対性理論パラドックスは始めから存在しない。
0041ご冗談でしょう?名無しさん
垢版 |
2019/11/12(火) 22:48:42.98ID:RjA4yf0v
パラドクスって、何時代の恐竜? ジュラ紀かな?白亜紀かな? 
0042ご冗談でしょう?名無しさん
垢版 |
2019/11/12(火) 22:48:51.41ID:FACXvC6y
超対称性理論、超弦理論、インフレーション理論って何か実証的な証拠はあるの?
0044ご冗談でしょう?名無しさん
垢版 |
2019/11/12(火) 23:01:14.73ID:???
インフレーションは信じるけどクオリアは信じない

これがここの人たちですね

どちらも根拠なんてないのに
0045ご冗談でしょう?名無しさん
垢版 |
2019/11/12(火) 23:06:00.35ID:RjA4yf0v
今はどっちかというと、デフレーションでしょ。
インフレーションは、昭和時代の遺物。
0046ご冗談でしょう?名無しさん
垢版 |
2019/11/12(火) 23:12:18.56ID:Gnao9/3X
>>11
お前何回聞いてんだよアホ。

・f=ma(第2法則)
・作用反作用の法則(第3法則。変形すると運動量保存則)

この2つで力と質量の2つを定義するんだってーの。
そして慣性の法則(第1法則)が成り立つ座標を基準座標として
これらを定義しろってーのが「力学」なんだよアホザルが。

作用反作用より、m1a1+m2a2=0
m1が基準質量であると定義すれば、a1とa2の測定によりm2が決まる。
m2が決まればf2も決まる。

作用反作用がなければ基準質量m1を決めたって
そこから一歩も進まんことをとっとと理解しろや。

m1a1+m2a2=0がバネなど重力以外ならば得られるのは慣性質量であり、
重力ならば重力質量である。

こういう根本をまったく理解せず
素粒子論やら量子論やら知ったかのアホザルどもには
腹の底から笑わせられるわ。

くっくっく
0047ご冗談でしょう?名無しさん
垢版 |
2019/11/12(火) 23:22:43.19ID:Gnao9/3X
で、お前らが持ち上げてるファインマンは
こんな明瞭簡潔に説明してくれてんのか、ああ?

んなわけないわな。
どの教科書もワシレベルに達してるものは
見たことないわ。

どんどんアホしかおらんようになって
いくのうー

くっくっく
0048ご冗談でしょう?名無しさん
垢版 |
2019/11/12(火) 23:26:28.69ID:fFM13MRQ
http://www.qualia-manifesto.com/manifesto.j.html
茂木健一郎はクオリアが解明されたらアインシュタイン以来の最大の科学革命になると言ってるよ
0049ご冗談でしょう?名無しさん
垢版 |
2019/11/12(火) 23:26:56.62ID:RjA4yf0v
アホいとり
0050ご冗談でしょう?名無しさん
垢版 |
2019/11/12(火) 23:29:43.03ID:Gnao9/3X
エネルギーを理解してないバカほど
運動量運動量運動量
ハミルトニアンハミルトニアンハミルトニアン

もうな、
「運動量」と「ハミルトニアン」の
2つだけでコイツはバカザルだとすぐ分かるんだよなあ。
根本をまったく理解していない知ったかバカご用達キーワードが
「運動量」と「ハミルトニアン」だ。

くっくっく
0051ご冗談でしょう?名無しさん
垢版 |
2019/11/12(火) 23:30:46.97ID:???
>>42
超対称性理論や超弦理論は只の仮説だが
インフレーションは宇宙背景放射の精密観測と超新星観測で証明された
というのが現在の定説
0055ご冗談でしょう?名無しさん
垢版 |
2019/11/12(火) 23:38:42.29ID:qwIAn29A
クオリアというのは人間の脳が行う感覚信号の処理を特定の視点から見たときに現れる見かけ上の概念でしかない
0057ご冗談でしょう?名無しさん
垢版 |
2019/11/12(火) 23:41:11.61ID:Gnao9/3X
>>48
コイツも
f=maが力なのか質量なのか
どっちを定義してるのかうんぬんの
バカザルなのは99.99%間違いないだろ。

ざっと見たら
ただの厨二病患者ねえーかよ。

くだらんもん貼るなボケザルが。
目が腐ったわアホかボケ

くっくっく
0058ご冗談でしょう?名無しさん
垢版 |
2019/11/12(火) 23:47:34.92ID:???
>>54
宇宙の誕生はまだエネルギーが足りんから無理としか
実証できるエネルギーを作れるかいなかもわからんけど

でもクオリアは実証無理なのをクオリアといってんじゃないの?
しらんけど
0059ご冗談でしょう?名無しさん
垢版 |
2019/11/12(火) 23:47:41.03ID:fFM13MRQ
>>57
お前はこのレベルのものは書けないだろwww
0060ご冗談でしょう?名無しさん
垢版 |
2019/11/12(火) 23:49:02.48ID:fFM13MRQ
知的に誠実であり勇気を持つ者達よ、「クオリア」の解明のために団結せよ!
0061ご冗談でしょう?名無しさん
垢版 |
2019/11/12(火) 23:49:46.04ID:RjA4yf0v
宇宙は生まれてさえいないことに誰も気づいていない
0062ご冗談でしょう?名無しさん
垢版 |
2019/11/12(火) 23:49:46.38ID:???
>48
>クオリアが解明されたら

ないないw
二十年研究してまだこんな能書き以上のことが書けないレベルなのだから推して知るべし。
0064ご冗談でしょう?名無しさん
垢版 |
2019/11/12(火) 23:53:38.95ID:fFM13MRQ
茂木さんは東大の物理学科卒だから物理の深い知識もある
0065ご冗談でしょう?名無しさん
垢版 |
2019/11/12(火) 23:55:13.35ID:qwIAn29A
脳から見た「外」の入力を区別するためのラベルがクオリアのようにふるまうだけ
コンピュータで言ったらポート番号みたいなもんでしかない
0068ご冗談でしょう?名無しさん
垢版 |
2019/11/13(水) 07:47:58.81ID:TwM5b3tv
格子振動の音響モードと光学モードについての質問なんですが、
https://ja.m.wikipedia.org/wiki/格子振動
このウィキペディアの音響モードと光学モードの節で、

N個の原子からなる結晶では、振動モードは3N個だけある。そのうち3個は音響モードであり、残りの3(N-1)個は光学モードである。波数ベクトルkが0の極限で固有振動数ωが0になるようなモードを音響モードという。一方0にならないモードを光学モードという。

とあるのですが、自由度が3Nとなるのは3次元に振動できる場合であって、グラフのような1次元の振動の時はNになりますか?
さらに、同じ箇所で、ウィキペディアのグラフでは赤い線でプロットされた分散曲線にアコースティックと書いてありますが、音響モードとはk=0の部分だけではないのですか?
0069ご冗談でしょう?名無しさん
垢版 |
2019/11/13(水) 07:52:45.94ID:WE7sabi4
>>38
>特殊相対性理論でも運動方程式 dp/dt = f が成り立つなら同じことが言えないか?
> 答えは Yes
>回転してない物体が回転するたぐいの特殊相対性理論パラドックスは始めから存在しない。

特殊相対論では成り立たない。
よって、ある慣性系で回転してない物体が、別の慣性系では、回転して観測されうる。
とーます歳差運動でググれ。
0072ご冗談でしょう?名無しさん
垢版 |
2019/11/13(水) 10:08:14.05ID:???
>>69
アホ、日本語文が読解できないらしい
>>38 >回転してない物体が回転する の意味は
物体の角運動量と力モーメントがゼロのとき、並進運動する他の慣性座標系から観測しても不変。
加速度座標系うんぬんとは関係ない。
0073ご冗談でしょう?名無しさん
垢版 |
2019/11/13(水) 10:10:50.82ID:UAYV/zpH
つまり、一般相対性理論では成り立たないんだね。勉強になるなー
0075ご冗談でしょう?名無しさん
垢版 |
2019/11/13(水) 10:29:11.67ID:???
ニュートン力学では慣性力と呼ぶ見かけの力を導入して加速度座標系を記述できるから
特殊相対性理論でも同様に見かけの力を導入すれば可能だが暫定的になる。
0078ご冗談でしょう?名無しさん
垢版 |
2019/11/13(水) 10:54:18.32ID:???
>>71
k→0でω(k)→0になる関数ω全てが音響モードという事ですか?
k=0だと原子がみんな同じ方向に同じ距離だけ変位するので光学的に不活性というのはまだわかるんですけど
kがそれ以外の時は変位がまちまちで電気双極子ができそうな気がする
0079ご冗談でしょう?名無しさん
垢版 |
2019/11/13(水) 11:07:33.99ID:UAYV/zpH
上っ面だけを知ったかぶり。
0085ご冗談でしょう?名無しさん
垢版 |
2019/11/13(水) 13:39:56.83ID:q0L7ODvV
ダイナモ理論で天体の磁場は、天体内部の流体が磁力線を引きずって発生させると言う話だった。
じゃあ、太陽の黒点が表面に対して垂直な磁場を持っているとすれば、
過去に何か黒点から外部に流体が飛び出して行ったと言う事なのか?プロミネンスを噴出した跡とか
表面から何も飛び出さないなら、表面と平行に運動するプラズマによって表面と平行な磁場ができるだけなのではないか?
0086ご冗談でしょう?名無しさん
垢版 |
2019/11/13(水) 17:13:23.27ID:???
対流で捻れて強化された磁力線は自身で動く
太陽から飛び出した磁力線の根本が黒点
プラズマに動かされたんじゃない
強化前はプラズマに引きずられてたがな
0087ご冗談でしょう?名無しさん
垢版 |
2019/11/13(水) 17:21:10.75ID:eGw3A6Yd
直管型の蛍光灯のちらつきについて質問です
東日本(50Hz)では100Hzで明滅しているところまでは理解できたのですが、時間的にミクロの単位で考えると完全に消えている瞬間があるとの理解で良いのでしょうか?
0088ご冗談でしょう?名無しさん
垢版 |
2019/11/13(水) 20:13:47.82ID:q0L7ODvV
太陽の黒点って地球で言う「高気圧」だったりする?(高気圧:上層の空気を集めて下層で噴出させる現象)
下層のプラズマより上層のプラズマの方が単位質量当たりの体積が大きい分、磁場を引きずる力が強くなっていて、
上層で周囲から集まってくるプラズマが渦電流を生み出して磁場を発生させている。
0089ご冗談でしょう?名無しさん
垢版 |
2019/11/13(水) 20:38:04.87ID:q0L7ODvV
>>88
>下層のプラズマより上層のプラズマの方が単位質量当たりの体積が大きい分、磁場を引きずる力が強くなっていて、

地球で言うと、地上よりも上空の方が風速が速いから、太陽表面も似たような事になっていて、
「下層よりも速度の速い上層の流体の方が強く磁場を引きずるのではないか?」
と言う考えを言っている。
0090ご冗談でしょう?名無しさん
垢版 |
2019/11/13(水) 22:37:40.02ID:???
太陽の磁場はもっと深い太陽内部で生成されたもので、
そんな表面近いところのプラズマの運動が起源ではなかろう
0091ご冗談でしょう?名無しさん
垢版 |
2019/11/13(水) 22:43:56.49ID:UAYV/zpH
ダイナモは太陽の対流圏だよ。だから、意外と浅い。
0093ご冗談でしょう?名無しさん
垢版 |
2019/11/13(水) 23:14:49.50ID:???
>>91
ID:q0L7ODvVが勘違いしかねないが、意外と浅いと言っても、
太陽の対流圏は太陽半径の70%あたりから光球に至る、
れっきとした太陽内部の層。
0094ご冗談でしょう?名無しさん
垢版 |
2019/11/13(水) 23:24:04.55ID:???
>>87
肉眼の観測ではちらつき程度しか判らない。web検索で満足するか、やる気があるなら
60HzのWebカメラなどで撮影してコマ送りで画像を調べるとか、1/100秒以下で光量が
記録できる装置を自作して観測する。白熱電球、LED電球なども調べればキミは
理工系の分野で成功する可能性がある。
0095ご冗談でしょう?名無しさん
垢版 |
2019/11/13(水) 23:27:14.78ID:???
>>92
例えばラグランジアンLが
L=∂φ†∂φ+φ†φなら
φ→exp(-a)φ (aは定数)
としても
φ†→exp(a)φ
なので不変である。(大域的U(1)対称性)

だけどもしaを局所的に変化するx依存性を持たせたとき
計算したらわかるが微分∂がは言っている項が不変でない
しかし量子力学とかでわかるけどそういう位相変換で物理は変わらないはずなので
そういう局所的変換で対称にしたい。それがゲージ対称性

理論にゲージ対称性を課したいなら微分を変えてやればよくて
そうすることでいわゆる電磁気学が系に入る
ってイメージ
間違ってたらすまん

まぁそういう局所的な変換で物理は変わらないって課してラグランジアンが決定される
0096ご冗談でしょう?名無しさん
垢版 |
2019/11/13(水) 23:29:17.82ID:???
ゲームとかの30fpsと60fpsは違うーとか言う人はいても100fpsの蛍光灯の違いはわからないんですね
不思議ですね
0097ご冗談でしょう?名無しさん
垢版 |
2019/11/13(水) 23:39:08.06ID:q0L7ODvV
>>93
>ID:q0L7ODvVが勘違いしかねないが、

勘違いって何の事だ?
黒点を高気圧と言った理由は、もし黒点が低気圧だったら黒点内部からプラズマが湧き上がってきて、
そのプラズマに引きずられて、高密度にまとまっていた磁束が周囲に散り散りになって、
強磁場領域が維持できないと思ったからだけど?
0098ご冗談でしょう?名無しさん
垢版 |
2019/11/14(木) 00:00:14.93ID:uIElVX2x
>>94
お答えありがとう。
白熱電球なら電子振動だしエネルギーの急激な変化もないから完全なオフの状態がないのはわかるので実験しなくても想像つくんだわ。
あとLEDは原理から明らかに完全オフの時間があるし、実際に10msまで計測できる実験系でオフが確認できる。

ただ、励起されて電磁波が生じる蛍光灯の場合、特に直管型の場合に、インバーターがなければ交流50x2Hzで振動するのまではわかるんだけど、
明滅のオンオフだけじゃなくて、弱い電流の流れている時間もあるから、サンプリングレート100Hz程度の実験系だと観測できないようで。
励起された電子が元の軌道に戻るまでにある程度の時間がかかるなら完全にオフしないんじゃないかなと。

そこんとこわかる人がいたら教えてください。
もしくはしかるべきスレへ誘導願えるとありがたいです。
0099ご冗談でしょう?名無しさん
垢版 |
2019/11/14(木) 00:21:11.83ID:QW1G86CG
>>93
>太陽の対流圏は太陽半径の70%あたりから光球に至る、
光球の意味を勘違いしてはりますな
0100ご冗談でしょう?名無しさん
垢版 |
2019/11/14(木) 04:21:39.68ID:???
LEDの点滅と言えば、夜など交差点の信号機のライトに対して素早く視線を横切らせたときの網膜に残る残像から、目だけで実際に点滅していることが確認できる。
0101ご冗談でしょう?名無しさん
垢版 |
2019/11/14(木) 09:20:21.26ID:???
>>97
黒点のある光球は太陽表面の事象。
太陽磁場の起源とされているダイナモ効果が起きている対流層は光球の内側の対流層。
黒点付近でのプラズマのダイナミクスの影響が0とは言わないが、磁場を少し乱す程度で
太陽磁場の大部分は太陽内部の話でしょう
0102ご冗談でしょう?名無しさん
垢版 |
2019/11/14(木) 10:41:50.94ID:???
>>100
人間の感覚測定器は当てにならないので物理学と機械・電気的な装置が発明されたのだよ
交流100V用のLED電球の光は
1.直流変換した定常光
2.周期点滅している光
頭が良い人なら定量的に判定できる装置を考案するのは簡単だ、大学入試の筆記試験に採用すべき。
0104ご冗談でしょう?名無しさん
垢版 |
2019/11/14(木) 11:24:53.37ID:???
なんか実体験を否定されたような気がするな。
嘘を付いてるわけじゃないぞ。LEDが点滅するのを、俺は他人の解説からでなく経験で知った。
0105ご冗談でしょう?名無しさん
垢版 |
2019/11/14(木) 11:28:03.78ID:???
最近流行ってるみたいなのでちょっと考えてみた。

「身体の物理状態が同一ならば意識状態も同一である」
なぜなら自分の意識経験の内容を他人に語る過程は客観的には物理過程としてみることが出来るから
もし異なる意識状態があっても物理的には同じ内容を経験として語ってしまうことになる。
これは自分からみると異なる意識状態がある時は自分の意識経験を正直に語ることが不可能になるということになる。
0106ご冗談でしょう?名無しさん
垢版 |
2019/11/14(木) 11:36:07.86ID:???
哲学的ゾンビくらいは知っておきましょうよ

外から見て同一であることは、中身も同じということと同じではないですし、外から見る限り区別なんてできないですよね
0108ご冗談でしょう?名無しさん
垢版 |
2019/11/14(木) 11:52:21.54ID:QW1G86CG
iPhoneとの区別くらい簡単でしょ
0110ご冗談でしょう?名無しさん
垢版 |
2019/11/14(木) 13:01:24.54ID:???
>>92
物理には対称性という概念が重要で
例として相対論で出てくるローレンツ変換で物理法則が変わらないとか
量子論で出てくる波動関数の位相を一斉に変えても結果は同じとかがある
以上の対称性は全時空間で同じ変換をした場合の対称性で大域対称性と言うが
もっと一般化した対称性で局所対称性と言うものがある
これは時空の各点で違う変換をした場合の対称性。
もちろん大域対称性をそのまま局所化して成り立つはずもないが
付加的な場を考えると成り立つようにできる
例えば慣性系を回転系に座標変換すると、これは各点で運動方向が違う変換だが
遠心力とコリオリ力を追加してセットで考えれば物理法則は変わらない
このような局所化した変換をゲージ変換と呼び
この追加した力の場をゲージ場と言う(場の量子論でしか言わないけどね)
そうして出来た局所対称性をゲージ対称性と言う
最初の例で言えば、ローレンツ変換を局所化して出るゲージ場が重力場で
波動関数の位相変換を局所化して出るゲージ場が電磁場
その他には、カラーの大域対称性(クォークの3色が入替え自由)を
局所化して出るゲージ場がグルーオン場
こんなふうに大域対称性があったら局所化してゲージ場が導けるってのが
ゲージ原理
0111ご冗談でしょう?名無しさん
垢版 |
2019/11/14(木) 13:03:59.11ID:???
>>104
基本的に個人の体験談は物理学とはいえない
物理学は再現性がある物理量間の数量的関係でなければならないから、数量的に示せなければ参考程度だ。
人間の意識やクオリアが必要となるのは、多数の観測による数量的関係を認めるかの論理判断といえる。
0112ご冗談でしょう?名無しさん
垢版 |
2019/11/14(木) 13:29:17.49ID:???
>>111

それが物理学かどうかなんて話をしたつもりはないがな。
ただしかし、数量的なことを言えれば物理学だというのなら、信号機のLEDの点滅周期は0.5秒以下、くらいのことは目視確認でも言えるぞw
0113ご冗談でしょう?名無しさん
垢版 |
2019/11/14(木) 13:42:09.12ID:???
>>112
>数量的なことを言えれば物理学だ・・・
日本文が読めないのか、「再現性がある物理量間の数量的関係」
キミ個人の意識時計?で0.5秒以下と主張したところで、肝心の物理量間の数量的関係が無いだろ
0114ご冗談でしょう?名無しさん
垢版 |
2019/11/14(木) 15:48:31.47ID:???
数値的に示せなければ物理学ではない、と
数量的なことを言えれば物理学だ、は裏の関係だから、
論理学的には同等ではないよ。
対偶なら同等だけどね
0115ご冗談でしょう?名無しさん
垢版 |
2019/11/14(木) 16:17:13.81ID:QW1G86CG
算術と物理の違いについて、三行以内で述べよ。
0117ご冗談でしょう?名無しさん
垢版 |
2019/11/14(木) 17:34:46.54ID:???
>>113
「再現性がある物理量間の数量的関係」を具体的にいえば
アンドロイド(スマホ)が撮影する動画設定が60fpsなら1/60秒ごとの撮影でその精度は
アンドロイド内部の水晶振動子の物理振動数の精度で、スマホ機種に依存しない。
アンドロイドがLED信号機を撮影した画像が、t秒間にn回点滅してれば
LED信号機の点滅周期Tは T = t/n [sec] の数量的関係となる。
0118ご冗談でしょう?名無しさん
垢版 |
2019/11/14(木) 17:51:14.33ID:???
今計算してみたんだが

陽子半径は
8.751E-16 m
8.751E-14 cm
2.8057E-39 cc

陽子質量は
1.6726E-27 kg
1.6726E-24 g

比重は(g/cc)は

5.96143E+14 g/cc

球で近似してますが、実際の形状と、質量分布と、電荷の分布を教えてください
0119ご冗談でしょう?名無しさん
垢版 |
2019/11/14(木) 18:13:07.79ID:???
>>118
陽子半径とは核力の到達距離
陽子質量とは陽子の静止エネルギーの換算値
それ以外と質問内容はあまり意味をもたない。
0120ご冗談でしょう?名無しさん
垢版 |
2019/11/14(木) 20:02:16.00ID:???
>>117
撮影の時間間隔と点滅の時間間隔が近いとうなり現象が起こって、
点滅周期よりずっと長い時間間隔で点滅するように撮影されるよ。
極端な話、時間間隔が完全に一致すると、撮影された画像は全く点滅しない(点滅周期=無限大)。
(ずっと点灯してるか、運が悪いとずっと消えてるか、どっちかになる)

逆にこの現象を使うと蛍光灯でも点滅しているのが観測しやすい。
蛍光灯下で止まりかけの扇風機を見ていると、まだ回っているのに
目には羽根が止まって見える瞬間が何度かある。蛍光灯の点滅周期と
扇風機の回転周期÷羽根の数が一致したときに起こる。
0123ご冗談でしょう?名無しさん
垢版 |
2019/11/14(木) 21:37:33.72ID:???
>113

あー。それなら俺はなおさら最初から物理の話なんかしてないわ。
>>111は「一見関係ありそうで実は関係のない話」ということでいいかな?
0125ご冗談でしょう?名無しさん
垢版 |
2019/11/14(木) 22:49:05.73ID:???
茂木某は物理学に意識、感覚のたぐいを持ち込んで素人を煙に巻いてるだけ
まともな実験物理学の研究者にとって迷惑以外にない。
0126ご冗談でしょう?名無しさん
垢版 |
2019/11/14(木) 23:18:40.23ID:???
>>104
誰でも確認できる事は充分に再現性のある実験と言えるんだから
馬鹿なイチャモンなんぞ放っとけよ
相手にして喜ばせる必要はない
0127ご冗談でしょう?名無しさん
垢版 |
2019/11/14(木) 23:20:25.30ID:???
空が青いのは、波長が短く散乱してるからと、物理雑学系本に書かれてるけど、もっと波長の短い紫はどうなってるの?
0131ご冗談でしょう?名無しさん
垢版 |
2019/11/14(木) 23:39:25.25ID:???
大昔ならともかく、自分の感覚を使って何かの再現実験してもそんだけでは物理学にならない
物理量の基本的理論から演繹的に説明出来なければ、物理学ではゴミ箱行き。
0132ご冗談でしょう?名無しさん
垢版 |
2019/11/14(木) 23:51:02.38ID:???
インフレーションだとか多次元宇宙とかはとても現実的とは思えませんし再現性もなさそうですけど物理なんですね
0138ご冗談でしょう?名無しさん
垢版 |
2019/11/15(金) 00:47:43.35ID:???
>>127
>空が青いのは、波長が短く散乱してるからと、物理雑学系本に書かれてる
これこそ物理学ではなく、「物理雑学」の説明の見本、素人はそれで満足なのが可笑しい。

青い色と意識するのと波長が短い可視光の相関関係は知っているとしても
大気からの光が目に入るのは大気に当たって曲がった(散乱した)からであって言い換えにすぎない。
物理学の説明ならば、波長が短い可視光が大気で散乱する理由を理論的に説明する必要があるだろ。
0140ご冗談でしょう?名無しさん
垢版 |
2019/11/15(金) 01:09:45.43ID:???
ソニー社内にエスパー研究室なるものがあることはソニー社内では知られていたから
エスパー研究室を作った創業者が亡くなって1998年に閉鎖されたのも知っている。
茂木某は1997-1998年にソニーに在籍し研究所が閉鎖とともに他に移っている
この事実は、茂木某がエスパー研究室(超能力研究)に関わっていたということの証拠である。
0141ご冗談でしょう?名無しさん
垢版 |
2019/11/15(金) 02:58:58.41ID:+QaYwH68
クオリアが解明されたらアインシュタイン以来の最大の科学革命になるのだが何か?
0144ご冗談でしょう?名無しさん
垢版 |
2019/11/15(金) 09:07:22.19ID:???
茂木某は某科学と称する宗教教祖と同様な素人騙しの宣伝本などで得た
数億円の所得隠しが発覚して追徴課税された。
0145ご冗談でしょう?名無しさん
垢版 |
2019/11/15(金) 11:28:35.65ID:sshby21t
質問します。
重力って恒星の時点に関係なく3次元的に等しく影響するでしょ。
なんで太陽系や銀河系の天体は同一平面上に分布してんの?
なんで平たく集まっているように見えるの?
0146ご冗談でしょう?名無しさん
垢版 |
2019/11/15(金) 11:29:50.18ID:sshby21t
時点×
自転◯
0148ご冗談でしょう?名無しさん
垢版 |
2019/11/15(金) 11:55:37.22ID:mCbiNXyc
>>145
銀河系の重力源の主役は暗黒物質。恒星はそこを漂うあぶくのようなもの
0149ご冗談でしょう?名無しさん
垢版 |
2019/11/15(金) 12:13:42.86ID:sshby21t
>>147
不規則である物が、なんで一様な回転方向が出来るの?
0151ご冗談でしょう?名無しさん
垢版 |
2019/11/15(金) 12:56:04.13ID:/4dSSPP9
銀河の渦面から垂直に飛び出して周期運動する恒星もある。
もし太陽系がそれらの一つだったなら、夜には大パノラマやね。
0152ご冗談でしょう?名無しさん
垢版 |
2019/11/15(金) 14:28:54.85ID:???
銀河も惑星系もガスの抵抗が有効な初期に平面回転に落ち込んだ
星ができてからの衝突ではない
(宇宙スレでも回答があったな)
0153ご冗談でしょう?名無しさん
垢版 |
2019/11/15(金) 16:02:05.84ID:sshby21t
>>150
砕けたら、一様な方向にそろうの?
0154ご冗談でしょう?名無しさん
垢版 |
2019/11/15(金) 16:09:15.52ID:sshby21t
土星の輪だって、当初は無秩序 てんでバラバラに様々な方向と速度を持ってたはずじゃん。

なんで平面上に集まるんだ?
訳わからん。
0155ご冗談でしょう?名無しさん
垢版 |
2019/11/15(金) 16:55:30.25ID:P3agIM+f
質問5-4)なぜ、太陽系の惑星は、同じ平面を、同じ方向に公転しているの?
https://www.nao.ac.jp/faq/a0504.html
0156ご冗談でしょう?名無しさん
垢版 |
2019/11/15(金) 17:49:49.82ID:sshby21t
>>155
ありがとう。
色々な速度と方向を持った質量が合成された結果だと理解すれば良いのか。

へー。
0157ご冗談でしょう?名無しさん
垢版 |
2019/11/15(金) 19:06:18.45ID:???
>>145
ピザ職人が片手でピザの生地をグルグル回してると小麦粉の塊を円盤形に均されていくでしょ?
あれと同じ。

生まれたての銀河はガスの塊だけどガス全体の合成角運動量は一意に定まる。
その方向に偏っていくんだよ。
0160ご冗談でしょう?名無しさん
垢版 |
2019/11/15(金) 23:35:53.92ID:mCbiNXyc
神様はピザ食人
0161ご冗談でしょう?名無しさん
垢版 |
2019/11/16(土) 01:25:52.61ID:???
電磁気学によれば、荷電粒子が加速されると電磁波を放出するとのことですが、
加速を受けることと重力を受けることは区別できないという等価原理を用いると、
重力場の中で静止している荷電粒子もまた電磁波を放出することになります。

地球上の(地球の重力場の中にある)荷電粒子は静止していても電磁波を放出しているのでしょうか?
それは観測されているのでしょうか?
0162ご冗談でしょう?名無しさん
垢版 |
2019/11/16(土) 02:13:21.60ID:???
座標変換がよくわかってないようですね

どんなに粒子が加速していても、適当に自分も加速すれば追いつける
これが等価原理です

止まってる粒子は止まってるんですから加速しないですよね
0163ご冗談でしょう?名無しさん
垢版 |
2019/11/16(土) 03:27:40.58ID:???
加速してる粒子は慣性力を受けるので静止した粒子とは全く状態が違います。
観測者が粒子と一緒に加速していようが慣性系に留まっていようが粒子の状態には全く関係ありません。

観測者が粒子と一緒に加速すれば粒子にかかる慣性力も消えるのですか?

消えないですよね。
0166ご冗談でしょう?名無しさん
垢版 |
2019/11/16(土) 08:16:27.53ID:xH5lPoFn
>>157
それは違うと思う。
回したら平たくなるのは分かる。

あっちこっち無秩序な運動だったものが、何で一様の方向性に収束してるんだ?って質問だよ。
0168ご冗談でしょう?名無しさん
垢版 |
2019/11/16(土) 09:51:44.30ID:???
>>161
正解は、等価原理とは完全に等しいことではない。
質量物体による一定重力と、一定加速度運動による慣性作用が同じ重力という意味で
粒子が質量物体による釣り合いで静止状態と、外力で加速する状態は物理的に異なる。
また、荷電粒子はクーロン力の電荷を持つ点ではない。電場が空間中に無限に広
がっている。
電磁波は荷電粒子の加速度運動により電場が電磁場となって外部に放射される現象である。
0169ご冗談でしょう?名無しさん
垢版 |
2019/11/16(土) 11:17:13.24ID:khr28oGD
慣性運動する荷電粒子は、その廻りに電磁場を形成し、無限遠で0になる。
加速度運動する荷電粒子は、その廻りに電磁場を形成し、その一部の「エネルギー流」は、
無限遠まで拡げた閉曲面をも貫いて発散し、孤立系外へと漏れ出る。
それが電磁波。
ちなみに、重力場に留まる家電粒子が電磁波を放出しないのは、「等加速度運動」だから。
等加速度運動する荷電粒子の「自己力」は0。
これを書き込むと、

「それは、速度が0の瞬間で、観測者との相対速度が無視できないほど大きくなれば、そうとは限らん!」

とかいうアホも出てくる。
荷電粒子の運動方程式をローレンツ変換すれば、観測者との相対速度によらず、「自己力」は0だとわかる。
0172ご冗談でしょう?名無しさん
垢版 |
2019/11/16(土) 12:25:15.50ID:???
>>168
続き 一様重力(場)の空間で、自由落下する荷電粒子には重力以外の外力の作用がないから
自由落下の座標系では、等価原理により重力(場)がゼロで荷電粒子は静止、電場は変化しない。
0175ご冗談でしょう?名無しさん
垢版 |
2019/11/16(土) 13:04:54.43ID:UtDxZrOk
>>170
>等加速度運動でも放射しますよね?
ウンルー効果をgugu
0177ご冗談でしょう?名無しさん
垢版 |
2019/11/16(土) 13:38:29.77ID:UtDxZrOk
ハミルトニアンの固有状態だから
0179ご冗談でしょう?名無しさん
垢版 |
2019/11/16(土) 13:52:13.01ID:???
>>174
>全空間が等加速度運動なら放射しない
意味不明
>等加速度運動する荷電粒子の「自己力」は0。
よりは少しましな程度
0180ご冗談でしょう?名無しさん
垢版 |
2019/11/16(土) 14:37:48.64ID:???
一様重力中で静止している(基底状態の)荷電粒子が電磁波エネルギーを放出しない理由は
外部からのエネルギー流入がなく、電荷の保存法則で電場のエネルギーに変化がないからとなる。

また、電磁力の電荷と荷電粒子の質量の関係には等価原理のようなものは無い。
0181ご冗談でしょう?名無しさん
垢版 |
2019/11/16(土) 15:00:32.27ID:???
力=質量×加速度
運動量=質量×速度
の違いが分かりません。
凄く馬鹿なので、例えを入れて分かりやすくお教え下さい。
0184ご冗談でしょう?名無しさん
垢版 |
2019/11/16(土) 18:21:09.85ID:WvY4j6bZ
等加速度運動する荷電粒子を慣性系から観測すると電磁波を放出するので、
一様重力場中に静止している荷電粒子を自由落下している観測者が観測すると同じ電磁波が観測されることは
等価原理が保証している。

ただ、1クーロン、地球上の重力加速度、荷電粒子と観測者の距離が数メートルのオーダーで計算してみたが、
ほとんど微量な電磁波しか観測されない。
これを観測できる性能の装置はないんじゃないかってくらい。
0185ご冗談でしょう?名無しさん
垢版 |
2019/11/16(土) 18:29:47.44ID:???
この問題がうまくとけません。私の解答のどこが違うのでしょうか?
 熱の仕事当量をJとして、水当量がωである熱量計の中に質量mの水を入れ、熱量計の抵抗線の両端に電圧Vを加え、電流Iを時間tだけ流したとき、水温がθ1からθ2まで上昇したとすると、
0186ご冗談でしょう?名無しさん
垢版 |
2019/11/16(土) 18:30:33.58ID:???
J= Vlt/(c(m+ω)(θ2−θ1))が成り立つことを用いて、ωの誤差(例えば、温度計の熱容量を無視したことによる誤差)が10%あったとき、Jの誤差はいくらになるか、計算によって求めよ
0187ご冗談でしょう?名無しさん
垢版 |
2019/11/16(土) 18:31:19.72ID:???
私の解答
ωの誤差をΔω、Jの誤差をΔJとすると、
J+ΔJ = Vlt/(c(m+ω+Δω)(θ2−θ1))が成り立つので、
ΔJ=Vlt/(c(m+ω+Δω)(θ2−θ1))ΔJをJを用いて表すと、
ΔJ=-J/(((m+ω)/Δω)+1)となりました。
Jのズレには、mがどうしても残ってしまうようなのですが、
水当量だけで議論出来るのでしょうか?
0188184
垢版 |
2019/11/16(土) 18:53:28.17ID:???
その重力場中の電磁波は、1平米を貫くエネルギー量が10のマイナス15乗ワットの水準。
とても小さくて観測にかからんだろう
0189ご冗談でしょう?名無しさん
垢版 |
2019/11/16(土) 19:58:41.27ID:???
光は昼は波長の短い青が見えて、夕方は波長の長い赤が見えますが、昼と夜の間の時間に、波長の長さが青と赤の間の緑や黄色にならないのはなぜでしょうか?
0192ご冗談でしょう?名無しさん
垢版 |
2019/11/16(土) 20:06:39.32ID:???
>>178
物理的な機序を知りたい、という質問だったのですが・・・。
そう名付けたから輻射は出ない、という説明は説明になってないと思います。
0193ご冗談でしょう?名無しさん
垢版 |
2019/11/16(土) 20:13:02.72ID:???
平均とったら電子の速度0で一定な気がしますから、加速運動しているわけではないから矛盾ではない、みたいに古典的に考えるならなるんじゃないですかね
0199ご冗談でしょう?名無しさん
垢版 |
2019/11/16(土) 21:47:17.42ID:UtDxZrOk
かくて振り出しに戻る。頑張ってね
0201ご冗談でしょう?名無しさん
垢版 |
2019/11/16(土) 21:52:15.22ID:???
あぁ、固有値だからエネルギー保存されるのは当たり前というくだらない理由ですか

わからないんですね
0204ご冗談でしょう?名無しさん
垢版 |
2019/11/16(土) 22:24:55.80ID:???
古典的にはありえないから量子論なんだろ

量子論でいうと本当は加速=輻射じゃないってことや
それ以上でもそれ以下でもない
0205ご冗談でしょう?名無しさん
垢版 |
2019/11/16(土) 22:31:19.84ID:???
>>145
NHKで紹介していた 仮説は、 最初三次元的に自由に回っていると互いの重力で 軌道が乱され 太陽に落下するか 系外に飛ばされる。
たまたま同一平面上に残った塵だけが安定軌道を保てるので、それが惑星になったと言う説でした。
根拠は物理シミュレーションの結果との事。
同じ根拠でガス惑星が2個以下なのも必然と説明していましたね。
ご参考まで
0207ご冗談でしょう?名無しさん
垢版 |
2019/11/16(土) 22:39:19.28ID:???
>>184
>一様重力場中に静止している荷電粒子を自由落下している観測者が観測すると同じ電磁波が観測されることは
>等価原理が保証している。

その俺様説を裏付けるweb記事を載せてくれ
0208ご冗談でしょう?名無しさん
垢版 |
2019/11/16(土) 22:41:48.42ID:O7q59Izt
>電磁気学によれば、荷電粒子が加速されると電磁波を放出するとのことですが、
>加速を受けることと重力を受けることは区別できないという等価原理を用いると、
>重力場の中で静止している荷電粒子もまた電磁波を放出することになります。

そうだな。
等価原理が正しければそういうことになる。

等価原理は簡単だ。完全なデタラメだがな。
光をAからBへ照射したとき、慣性系ならBに到達するが
加速系なら光は曲がってBに到達しない「はず」であるという。

加速系ABが箱の中なら、それが宇宙の無重力空間で加速するエレベーター内なのか
地上で重力を受けて静止する箱の中なのか区別ができないという。

よって、光は重力で曲がり、それならば
荷電粒子も重力場で静止しながら電磁波を放射しなければならなくなるわな。
しかし、こんなものはすべてオオウソだぞ。

くっくっく
0212ご冗談でしょう?名無しさん
垢版 |
2019/11/16(土) 22:57:20.21ID:O7q59Izt
これが真相である。

・荷電粒子同士ではローレンツ力は働かず、必ず物質の存在が必要である。
 つまり自身が物質か、相手が物質でなければならない。

・電磁波の放射もまったく同じであり、荷電粒子単体では
 いかなる運動をしても電磁波は放射しない。それを観測しようとする物質が
 遠隔作用により電磁現象を受けたとき、荷電粒子から電磁波を放射されたように
 見えるだけである。
 つまり、電磁波など初めから存在せず、実体は遠隔作用である。

本当にアホだよな、お前らサルどもは。
こう考えれば何の問題もなくなり、もちろん相対論がデタラメであることも
すぐ気づけるんだよ。

くっくっく
0213ご冗談でしょう?名無しさん
垢版 |
2019/11/16(土) 23:13:43.72ID:???
>>207
運動してる電荷を近くで観測すれば電場の他に磁場が観測されるのは電磁気学の常識。
それと電磁波の放射は異なり、電荷の運動とは独立に光速度で伝搬し吸収されるまで消えない。
磁場と違って座標変換(観測者の運動状態)で現れたり消えたりしない情報エネルギー伝搬といえる。
0214ご冗談でしょう?名無しさん
垢版 |
2019/11/17(日) 01:19:41.26ID:???
>>213
思考実験で、電荷の球を積んだ飛行機が一様な重力中を水平方向に一定速で運動している
この状態では電荷による電磁波放射は無いと仮定する。
1. 飛行機を水平方向に1Gで加速した場合、マックスウェル方程式によれは電磁波を
 放射する(加速度に垂直方向が強い)。
2.飛行機を失速させて放物運動で1Gの自由落下させた場合、1と同じく電磁波を放射する。
3.等価原理によれば、自由落下の座標系では無重力の慣性系で電荷(電場も)静止している
 ので電磁波は放射しない。 2と矛盾する。
4.等価原理によれば、1では1Gで水平加速させた座標系の重力と垂直方向の1Gの重力は同じものである
  1,2が正しいならば3は矛盾する。
5.web 記事等によれば、3が正しいらしい。 となって矛盾の堂々巡りになる。

一般相対性理論の基本原理が矛盾だらけになるはずがない、キミに解決できるかな。
0215ご冗談でしょう?名無しさん
垢版 |
2019/11/17(日) 01:37:12.87ID:???
>この状態では電荷による電磁波放射は無いと仮定する。

この時点で仮定が間違ってる。

一様重力場中に静止してる(水平方向なら等速運動していても)電荷は先の議論のように
それだけで電磁波を放出している。
0218ご冗談でしょう?名無しさん
垢版 |
2019/11/17(日) 01:41:51.68ID:Yp4Cb3HO
>>215
>それだけで電磁波を放出している。
その電磁波の定義から始めよう
0219ご冗談でしょう?名無しさん
垢版 |
2019/11/17(日) 01:45:57.05ID:???
加速する荷電粒子が放出する電磁波は制動放射という。
具体的にはリエナール・ヴィーヘルトポテンシャルからポインティングベクトルが導出される。
0220ご冗談でしょう?名無しさん
垢版 |
2019/11/17(日) 03:40:30.33ID:???
>>214
>4.等価原理によれば、1では1Gで水平加速させた座標系の重力と垂直方向の1Gの重力は同じものである

飛行機の中の人は後方に1Gの慣性力、下方に1Gの重力が働いているからベクトルの合成則により
後方斜め下に√2Gの力が働いているように感じる。
等価原理によれば慣性力と重力は区別できないから、飛行機の中の人は後方斜め下に√2Gの重力がかかっているのと
同じである。
でしょ。

これから2や3が矛盾か無矛盾かなんて推論できないよ。
0221ご冗談でしょう?名無しさん
垢版 |
2019/11/17(日) 03:57:31.12ID:zF0Cnqwu
>>197
194の説明じゃダメなの?
物理的な機構の説明になっていると思うけど。
輻射は状態が時間的に変化することで起こる。
時間的に変化しない状態では輻射は起きない。
定常状態は時間的に変化しない状態である。
したがって定常状態では輻射は起きない。
0222ご冗談でしょう?名無しさん
垢版 |
2019/11/17(日) 03:58:08.19ID:E//erb+1
本当にアホザルしかおらんわ。

荷電粒子単独ではどんな運動しようが
クーロン力しか存在しない。

そこに観測しようとする物質があると
それに静電誘導が起こり電流が流れて
電磁誘導と変位電流が発生するために
それらが荷電粒子からの電磁波の放射に見えるだけだ。

物性なんだよ物性。
荷電粒子は物質ではない。よって最低限のクーロン力しか持たない。
電子と陽子がくっついて物質を構成して初めて電磁現象が発生するのだ。

電流もローレンツ力も電磁誘導も変位電流も
荷電粒子単独では発生しえず、物質がなければクーロン力しか存在せん。

物質が基準だから
観測者によって矛盾が起こるというバカなこともない。

アホノシュタインのバカに騙されて
「物質の物性」に気づかないお前らサルは
ホントに滑稽だわボケザルどもが。

くっくっく
0224ご冗談でしょう?名無しさん
垢版 |
2019/11/17(日) 04:12:58.15ID:E//erb+1
電流とは、物質内における荷電粒子(電子、陽子、イオン)の相対速度差によるものである。
よって物質外の以下の流れは電流ではない。

・荷電粒子の流れ
・帯電した分子や帯電した物体の流れ

これにいつまで経っても気づかんのだから笑えるわ。
思考力がなさすぎだろうがアホザルどもが。

くっくっく
0225ご冗談でしょう?名無しさん
垢版 |
2019/11/17(日) 04:27:26.76ID:E//erb+1
空間に単独で存在する電子や陽子やイオンや帯電物体の流れが
物質内を流れる電流とまった同じだと思い込んでるから
お前らはドツボにはまってんだよ。

違うに決まってんだろ。
物質によって導電率も透磁率も誘電率もすべて違うのに
なんで素っ裸の単独電子や単独陽子が
オールマイティにどんな電磁現象も引き起こせると思えるのか
ホントに底抜けのアホザルばっかだよなお前らサルどもは。

くっくっく
0226ご冗談でしょう?名無しさん
垢版 |
2019/11/17(日) 04:35:23.03ID:E//erb+1
20世紀の物理学者どもは
荷電粒子の電磁的特性が「物質内」と「物質外」でまったく違いがないと
思い込んでいたマヌケぞろいであった、だからあるはずのない観測者によるパラドックスを
解消しようとしたためアホノシュタインの相対論にすがりついたというのが真相だ。
コイツらぜんぶ雑魚なんだよ雑魚。

くっくっく
0227ご冗談でしょう?名無しさん
垢版 |
2019/11/17(日) 04:43:02.57ID:???
ベクトル空間からベクトル空間への写像が全単射の時、基底をこの写像で写して出来るベクトルの組も基底になる理由教えてください、、、
0230ご冗談でしょう?名無しさん
垢版 |
2019/11/17(日) 05:54:06.12ID:???
>>227
証明の概略は次の通り。

φ:K→K' を全単射とし、B⊂KをKの基底とする。
φが単射だからφ(B)⊂K'は線形独立性を持ち、
φが全射だからφ(B)は完全性を持つ。
故にφ(B)はK'の基底である。
0231ご冗談でしょう?名無しさん
垢版 |
2019/11/17(日) 05:54:41.65ID:zF0Cnqwu
>>229
様式美は笑った。
0232ご冗談でしょう?名無しさん
垢版 |
2019/11/17(日) 07:26:25.78ID:???
くっくっくも自分でおかしいってわかってるとは思うけど、
今までずっと主張してきたものを間違いだって認められないんだと思う
お爺ちゃんになると頑固になるからね
0233ご冗談でしょう?名無しさん
垢版 |
2019/11/17(日) 08:36:34.32ID:???
>>214
の様な矛盾が起こらない為には、一様な重力中に静止または等速運動してる荷電粒子から
電磁波が常時放射されているとするしかない。
ところが、この仮説ではエネルギー保存の保存の法則に反する。
一様な重力中に静止している荷電粒子を包む球面を考えると、この球面からエネルギーが
常時流出している。(ポインティングベクトル)
重力は仕事をしてないから、球面内の内部エネルギーが減少し続けなければならないが
電荷の保存法則により電荷と電場(エネルギー)の状態の時間変化はない。
これはエネルギー保存の法則に反する。

外力による荷電粒子の加速運動の電磁波放射は、外部の電場などからエネルギーが流入しており
エネルギー保存の法則と矛盾しない。(減速時の放射では粒子の運動エネルギーが減少する)
0234ご冗談でしょう?名無しさん
垢版 |
2019/11/17(日) 08:43:21.97ID:???
一様な重力中に静止または等速運動させようとすると、
重力以外の何らかの外力で支え続けないといけないから、
その外力の発生機構込みで考えないとね
0235ご冗談でしょう?名無しさん
垢版 |
2019/11/17(日) 08:56:12.46ID:???
>>234
>重力以外の何らかの外力で支え続けないといけないから
荷電粒子が静止している前提条件だから、「何らかの外力」も仕事もゼロ。

よく言われる「筋力で錘を支えていても物理では仕事をしてない」と同じ。
0236ご冗談でしょう?名無しさん
垢版 |
2019/11/17(日) 09:16:51.38ID:???
>>233
web検索でも、理論的に一様な重力中で静止している荷電粒子が電磁波を放射し続けるという
マトモな物理記事はない。
0237ご冗談でしょう?名無しさん
垢版 |
2019/11/17(日) 09:43:10.42ID:???
基本的な質問で申し訳ないんだけど
ある物体の質量をA g、それと同じ体積の水の質量をB gとして
その時の水の温度における密度がC g/cm^3の場合
この物体の比重はA/B×Cとなる

ってとある参考書に書いてあったんだけど比重って同じ体積で比べたときの質量比だよね
なんで密度をかけるの?「〜℃における水とこの物質の比重は…」って感じでA/Bでも良さそうだけど…
0239ご冗談でしょう?名無しさん
垢版 |
2019/11/17(日) 10:31:17.43ID:qphaRHrt
物理の仮説(理論)に、背理法なんか使えるか?
そもそも「排中律」が適用できない分野でそ。
0241ご冗談でしょう?名無しさん
垢版 |
2019/11/17(日) 10:49:54.12ID:???
>>238
>背理法から放射はあるわけですか
>>214
の論理記述が基本的に矛盾しないようにするには重力静止で電磁波放射しなければならないが
>>233
の論理によれば、重力静止での電磁波放射はエネルギー保存法則に反する。
つまり、論理矛盾によって振り出しに戻ってしまう。

Web 資料などによれば、一般相対性理論の数学計算では基本的に重力静止、自由落下
の両方とも電磁波の放射が起こらない。矛盾も無い。
このスレの物理用語による論理推論だと矛盾が起きている、これをどう解決するか?
0242ご冗談でしょう?名無しさん
垢版 |
2019/11/17(日) 11:10:41.30ID:???
>>237
wiki に在るように、比重とは「4度Cの水」と同じ体積の物体の質量比(密度比)になる。
或る温度での水での実験結果を「4度Cの水」に換算しなければならないということ
だから、「4度Cの水」の(見做し)密度が1g/cm^3 が概知でないと問題が解けない。
0243ご冗談でしょう?名無しさん
垢版 |
2019/11/17(日) 11:34:07.19ID:???
>>235
重力場が電磁波のエネルギーをまかなっている

等価原理からは一様重力場中に静止した荷電粒子が電磁波を放出することは明らか。
0247ご冗談でしょう?名無しさん
垢版 |
2019/11/17(日) 11:46:29.68ID:???
ただ、Webを探してもないから事実ではないとい発想はあまりにも科学的ではない。
逆に言うとWeb記事なら何でもすんなり信じちゃうんだろうなそういう人は
Webに得体の知れない情報が混じってるからこそ自分の頭できちんと考えるクセつけないといけないのに。
0250ご冗談でしょう?名無しさん
垢版 |
2019/11/17(日) 11:58:17.78ID:???
位置エネルギーは実在に感じられるのに、ポテンシャルが計算上の存在に思えるのって感覚の問題?
0251ご冗談でしょう?名無しさん
垢版 |
2019/11/17(日) 12:00:42.41ID:Yp4Cb3HO
見えないものを信じれるか、信心の問題でしょ。
0254ご冗談でしょう?名無しさん
垢版 |
2019/11/17(日) 12:44:33.60ID:Yp4Cb3HO
xxxかな
0255ご冗談でしょう?名無しさん
垢版 |
2019/11/17(日) 12:46:02.15ID:???
このスレの住人のレベルならGoogleかYahoo!でいんでね?

Web記事に物理学の根拠を求める奴ばっかだから。
0256ご冗談でしょう?名無しさん
垢版 |
2019/11/17(日) 12:49:21.83ID:???
>>235
それはちょっと正確ではないな

>荷電粒子が静止している前提条件
ではなく、荷電粒子が「一様重力場の中で」静止している前提条件、ね
0258ご冗談でしょう?名無しさん
垢版 |
2019/11/17(日) 17:13:14.70ID:???
>>241
電荷を重力で落下させるから「電磁波を放射しない」が正しい?のであれば
思考実験で、帯電球と錘を滑車で吊るし重力を釣り合わせたとする。
外部の電場の力によって真下方向に1Gで加速する。(自由落下と同じ運動)
1.加速された帯電球から電磁波が(水平方向に)放射される
2.加速された帯電球から電磁波の放射は無い
結果はどちらか?
0259ご冗談でしょう?名無しさん
垢版 |
2019/11/17(日) 17:42:24.17ID:Yp4Cb3HO
いちよう重力じゃないからダメだよ
0261ご冗談でしょう?名無しさん
垢版 |
2019/11/17(日) 19:06:58.19ID:???
>>258
横レスだが、その系には一様重力と電荷につなげた糸の張力と電場があって複雑化してるから思考実験としてはスマートではないと思う。

ただ、等価原理によれば重力と慣性力は区別がつかないからどんな種類の力が電荷に加わってたとしても、電荷にかかる合力がゼロになった瞬間は電荷は慣性系にいると考えてよく、従って電磁波は放射しないでしょう。

ちなみに、制動放射を水平方向と書いているのは恐らくラーモアの公式を想定してると推察されるが、あれは荷電粒子が光速より十分小さいときに成り立つ式だからね。
光速に近づくと進行方向に指向性の強い放射となる。
相対論の思考実験では対象は光速に近い場合を含めるだろうからここで水平方向と断定するのは良くない。
0262ご冗談でしょう?名無しさん
垢版 |
2019/11/17(日) 19:12:19.59ID:???
>電荷にかかる合力がゼロになった瞬間は
間違った。

重力以外の全ての力の合力がゼロ。
すなわち外力が重力しか働いておらず、従って重力のままに自由落下しているときね。
0263ご冗談でしょう?名無しさん
垢版 |
2019/11/17(日) 19:36:24.98ID:???
>>261
>等価原理によれば重力と慣性力は区別がつかないからどんな種類の力が電荷に加わってたとしても
>>258 の(複雑な)思考実験の意味は重力と慣性力は区別がつかないという概念だけでは
電磁波の放射が有るか無いか説明できないと言いたい為のもの。
>・・・従って電磁波は放射しないでしょう。
では説得力に欠けるということ、協力には感謝する。
0264ご冗談でしょう?名無しさん
垢版 |
2019/11/17(日) 19:47:54.24ID:???
糸の張力と電場が荷電粒子に及ぼす力が釣り合っていれば重力しか働いていない。
この時、荷電粒子は自由落下しているから、荷電粒子を基準とした系は慣性系と区別がつかない。

慣性系で静止している荷電粒子は電磁波を放射しないから、一様重力場中で自由落下している荷電粒子も放射をしない。
0265ご冗談でしょう?名無しさん
垢版 |
2019/11/17(日) 19:58:29.04ID:???
自由落下する電子が電磁波を放出するわけないじゃん。

電子を自由落下させたら電磁波が出るとかどんな書物にも書いてない。
0266ご冗談でしょう?名無しさん
垢版 |
2019/11/17(日) 20:06:14.55ID:???
>>264
意味不明
>>258  の思考実験で重力と釣り合ってるのは糸の張力
帯電球の加速度運動の力は外部の電場であり、電場が仕事をしているのが誰にでも判る。
0267ご冗談でしょう?名無しさん
垢版 |
2019/11/17(日) 20:10:49.23ID:???
>>266
仕事の概念をニュートン力学からそのまま持ってこようとするのは危険だよ。
加速系と慣性系では距離の概念も異なってくるし。

いまのところ>>264の説明が一番合理的
0270ご冗談でしょう?名無しさん
垢版 |
2019/11/17(日) 20:12:56.47ID:???
「意味不明」って言われてもな。

「自分が理解できません」って無知を暴露してるのと同じじゃん。
0272ご冗談でしょう?名無しさん
垢版 |
2019/11/17(日) 20:22:38.38ID:???
一様重力場中を自由落下している荷電粒子は電磁波を放出しない。
一様重力場中に静止している荷電粒子は電磁波を放出する。

ということ。
0273ご冗談でしょう?名無しさん
垢版 |
2019/11/17(日) 20:28:30.27ID:???
>>258
1.一様重力場中に静止している人と慣性系を等加速度運動しているエレベーターの中で慣性力を受けている人はどのような実験によっても区別できない、というのは正しいか?
2.エレベーターの中の観測者はエレベーターもろとも仕事をされている、というのは正しいか?
3.1.2.とも正しいなら、一様重力場中に静止している人は仕事をされているということになるが、そもそも静止しているので矛盾
0276ご冗談でしょう?名無しさん
垢版 |
2019/11/17(日) 21:06:48.34ID:???
>>274
思考実験をすると>>272が正しいとなるのは分かるのですが、そうなると一様重力場中に静止した観測者は、
その観測者からみて荷電粒子が静止して見えるにも関わらず電磁波を放出していることになると思います。
しかし、それはエネルギー保存則的にどういう説明になるんでしょうか?

等価原理的に正しく>>274を表現すると
「一様重力場中に静止した荷電粒子を自由落下している観測者が観測すると電磁波を放出する」
といのが正確なのでしょうが、電磁波の放出はエネルギーの移動を伴うので、いかに加速度系⇔慣性系の
特殊な座標変換を施したといえども、エネルギーの移動について意見が異なるというのは考えにくいと思います。
(例えば制動放射が殺人的なエネルギーであり、重力場中に静止している第三者がそれを浴びたら死ぬ場合、
自由落下している人と重力場中に静止している人で生死の結論が異なるということはありえない)
つまり自由落下している人が電磁波を観測したなら一様重力場中に静止している観測者もまた、(形状は異なるにせよ)
電磁波を検出するはずです。

自由落下している人から見たら荷電粒子は加速しているので重力が仕事をして荷電粒子を加速させ、それにより
制動放射が発生したと説明するのでしょうが、重力場中に静止している人が観測するその電磁波のエネルギーは
どこから来ると見えるのでしょう?
0277ご冗談でしょう?名無しさん
垢版 |
2019/11/17(日) 21:09:48.15ID:Yp4Cb3HO
エネルギー源は妄想からでしょ
0278ご冗談でしょう?名無しさん
垢版 |
2019/11/17(日) 21:10:27.11ID:???
あっ。違うか。

自由落下している人から見たら重力場はないのと同じなので
>自由落下している人から見たら荷電粒子は加速しているので重力が仕事をして荷電粒子を加速させ
というのは間違いですね。
0281ご冗談でしょう?名無しさん
垢版 |
2019/11/17(日) 21:25:57.75ID:???
簡単に言えば、一般相対性理論の電磁方程式を解いた結果によれば
一様重力中の荷電粒子が重力静止、自由落下のどちらも電磁波の放射は無い。

5ちゃんの物理板では等価原理のディレンマに陥って、誰も両方の結果を言葉による
物理説明できない。ということ。
0287286
垢版 |
2019/11/17(日) 21:39:18.39ID:???
>>284
ごめん。レス間違えた。

>>286は無視してください。
0292ご冗談でしょう?名無しさん
垢版 |
2019/11/17(日) 21:43:36.59ID:???
片やスレ内で文章を書いているのに片や検索したら分かる、ではまったくお話にならない。

質問スレの意味もない。
0293ご冗談でしょう?名無しさん
垢版 |
2019/11/17(日) 21:46:05.45ID:???
そう。
そもそも「ググれ」と言っている人間自体に信用がないのだから、その通りにググった内容はなおさら信用できない。
0295ご冗談でしょう?名無しさん
垢版 |
2019/11/17(日) 22:07:35.29ID:???
>>281
によれば、
静止状態で電磁波放射が無いことは、電荷とエネルギー保存の法則から正しい。
自由落下で電磁波放射が無いことは、自由落下の座標系が慣性系ならば正しい。
両方とも物理的に矛盾がない。
矛盾が有るのは、5ちゃんスレの「等価原理」の俺様解釈だということが誰でも判る。
0297ご冗談でしょう?名無しさん
垢版 |
2019/11/17(日) 22:14:21.67ID:???
>>295
1文目はエネルギー保存則。2文目は等価原理。
違う原理を持ち出して互いに矛盾がないから正しいなんてそれこそ俺様理論。

だれも説得できない。
0299ご冗談でしょう?名無しさん
垢版 |
2019/11/17(日) 22:28:08.56ID:???
一般相対論でも電磁気学でもいいけど
粒子の重力や電荷の自己結合をどう扱ってるのか全く知らねえ
0300ご冗談でしょう?名無しさん
垢版 |
2019/11/17(日) 22:40:15.34ID:???
>>298
点電荷が扱えなかったら質点も扱えないだろ。

テスト電荷という概念があるように、自分自身の作り出す場は無視するのがスタート。
自分自身の場の影響も考慮するのは応用だね
0302ご冗談でしょう?名無しさん
垢版 |
2019/11/17(日) 22:57:22.92ID:???
厳密に言えば思考実験に登場する全ての物体が作り出す場を考慮しないといけないんだろうけど、
アインシュタインの重力方程式にあるとおり、エネルギーや運動量があると重力場の源になっちゃうからな。

落下するエレベーターから水平に光を出すというお馴染みの思考実験も、その電磁波のエネルギーが生み出す重力場が云々なんて話になってまう。
0305ご冗談でしょう?名無しさん
垢版 |
2019/11/18(月) 00:03:58.36ID:APc4PmVn
殿下は重力源じゃないんだけどね。
0306ご冗談でしょう?名無しさん
垢版 |
2019/11/18(月) 00:16:42.50ID:S2eb2P0/
>>295
自由落下が慣性系とかガイジか?
0307ご冗談でしょう?名無しさん
垢版 |
2019/11/18(月) 00:27:30.87ID:???
質問スレで暴言とばすのやめてほしいんだけど。

定義と理論に基づいて地に足のついた議論をしよう。
0308ご冗談でしょう?名無しさん
垢版 |
2019/11/18(月) 00:34:26.77ID:???
仮に地球が電荷を帯びていたとしても地球の公転は自由落下だから地球は電磁波を発しない
ってことでおk?
0309ご冗談でしょう?名無しさん
垢版 |
2019/11/18(月) 00:57:37.18ID:???
太陽がつくる重力場は一様重力場じゃないから公転による自由落下を慣性系とみなせるのは局所的にだけだねえ。
有限の時間を考えるなら局所慣性系を次から次へと乗り換えることになるから加速系になってしまうねえ。
0310ご冗談でしょう?名無しさん
垢版 |
2019/11/18(月) 01:00:05.01ID:TR/2nouH
自由落下は慣性系ではないよね。
オレなんか勘違いしてる?
0311ご冗談でしょう?名無しさん
垢版 |
2019/11/18(月) 01:02:13.97ID:TR/2nouH
>>309
そうだよね。
よかった。勘違いではなかったようだ。
0313ご冗談でしょう?名無しさん
垢版 |
2019/11/18(月) 01:08:55.15ID:TR/2nouH
>>312
一様重量場ならば、だね。
0314ご冗談でしょう?名無しさん
垢版 |
2019/11/18(月) 01:32:58.24ID:???
言葉の定義は人によって異なるが「一様重量場」で検索すると「一様重力場」に訂正されるから
検索を好むここの住人にはウケが悪いかもねえw
0315ご冗談でしょう?名無しさん
垢版 |
2019/11/18(月) 01:35:28.48ID:???
話あんまり追ってないんですけど、地上で静止している人から見たら自由落下する電荷は電磁波を発生しないということですか?
0320ご冗談でしょう?名無しさん
垢版 |
2019/11/18(月) 02:01:53.79ID:???
英語が分からん。

そのサイトによると重力場中に静止している荷電粒子は制動放射を行うかどうか
なんて書いてるの?
0321ご冗談でしょう?名無しさん
垢版 |
2019/11/18(月) 02:10:28.92ID:???
Likewise, a charged particle at rest in a gravitational field does not radiate in its rest frame, but it does so in the frame of a free falling observer.

なんか行わないって書いてるように見えますね
0322ご冗談でしょう?名無しさん
垢版 |
2019/11/18(月) 02:19:26.72ID:???
What about a supported charge, then? Does it not radiate due to the equivalence principle? To answer this question, start again in the falling frame.

As observed from the freefalling frame, the supported charge appears to be accelerated uniformly upward.
The case of constant acceleration of a charge is treated by Rohrlich.

The corresponding electric and magnetic fields of an accelerated charge are also given in Rohrlich.[10]
To find the fields of the charge in the supporting frame, the fields of the uniformly accelerated charge are transformed according to the coordinate transformation previously given.

When that is done, one finds no radiation in the supporting frame from a supported charge, because the magnetic field is zero in this frame.

重力場中を制止している電荷の場合、自由落下している観測者から見ると確かに放射はあるのですが、それをまた地上に制止している観測者に合わせて座標変換するときに放射が消えると言っているようですね
0323ご冗談でしょう?名無しさん
垢版 |
2019/11/18(月) 02:31:23.23ID:???
上空から自由落下している人と地上に静止している人がいたとして、前者が後者とすれ違った瞬間、
両者は同じ場所にいるにも関わらず前者は放射を検出するけど後者は検出しないと。

仮に放射が殺人的な威力をもっているほど強いとしたら、2人は死ぬんですかね?生きてるんですかね?

いまいちしっくりこない。
0324ご冗談でしょう?名無しさん
垢版 |
2019/11/18(月) 02:37:55.60ID:???
ああ… そういうことか

David G. Boulware (1980)[11] finds that the radiation goes into a region of spacetime inaccessible to the co-accelerating,
supported observer. In effect, a uniformly accelerated observer has an event horizon, and there are regions of spacetime inaccessible to this observer.
0326ご冗談でしょう?名無しさん
垢版 |
2019/11/18(月) 04:20:08.30ID:???
でもどうなんだろ。

一様重力gの中で静止してる観測者には鉛直下方向c^2/gの距離に事象の地平面が広がってるけど、
それだけの距離離れてるからなあ
自由落下している人には検出される制動放射が重力場中で静止してる観測者の観測にかからないように事象の地平面に
消え去っていくってこと?
重力場のどの場所で静止してても検出を逃れる放射の仕方ってあるんだろうか…。
0327ご冗談でしょう?名無しさん
垢版 |
2019/11/18(月) 04:34:14.83ID:???
>>325
制動放射の強さは加速度と電荷の自乗に比例するから、加速度は死なない程度の強さにして電荷の方を大きくしてやれば良いよ。
0328ご冗談でしょう?名無しさん
垢版 |
2019/11/18(月) 09:49:57.78ID:???
>>322
>重力場中を制止している電荷の場合、自由落下している観測者から見ると確かに放射はあるのですが、
>それをまた地上に制止している観測者に合わせて座標変換するときに放射が消えると言っているようですね

その物理解釈は結果的に正しいとしても、特殊相対性理論による双子のパラドックスの解釈と同じで
慣性系からの一方的な解釈にみえる。
一般相対性理論の立場ならば、ロケットの加速度座標系からみて慣性系の時間が進むのか
説明できなければならない。
同様に、一様重力で静止している座標系からみて自由落下で加速度運動する荷電粒子が
電磁波を放射しない理由も、一般相対性理論の電磁気理論から説明できなければならない。

普通の人の数学レベルでは、一般相対性理論の電磁気理論で理解するのは困難だといえる。
0329ご冗談でしょう?名無しさん
垢版 |
2019/11/18(月) 10:11:00.56ID:???
>>325
これは打てないなという魔球

1) あまりに速くてドップラーで強い青方シフトして、真空紫外領域に達する為
  迫って来る時は見えず、バッターの横を通りすぎる瞬間だけ見える消える魔球

2) あまりに速くてドップラーでγ線領域までシフトして、打者を被曝させる原爆魔球

3) あまりに速くて超光速になり重力は無限大となってブラックホールになり
  乙女座銀河団ごと打者を飲み込むブラックホール魔球
0330ご冗談でしょう?名無しさん
垢版 |
2019/11/18(月) 10:15:42.43ID:???
重力って常に万物を引っ張ってるわけじゃん
これって無限のエネルギーということにはならんの?
0332ご冗談でしょう?名無しさん
垢版 |
2019/11/18(月) 10:40:47.87ID:???
>>328
そんな感じで考えるなら、マックスウェル方程式に追加される追加の項が、本当なら発生するはずの電磁波とは逆の負の電磁波を発生してトータルで0になるみたいな感じなんですかね
0334ご冗談でしょう?名無しさん
垢版 |
2019/11/18(月) 11:09:11.55ID:???
>>332
>>322 の物理解釈を >>328 の立場で再解釈すれば
制動放射の様な電磁波放射の現象は、観測する加速度座標系によって強さも方向も変わる、
特別な座標系から観測した場合のみゼロになる。
つまり、一様な重力中で静止している座標系からみて自由落下の荷電粒子の電磁波放射がゼロでも
静止している座標系からほんの僅かでも加速度運動してる観測者からみれば電磁波放射がゼロでない。

ただし、原子からの電磁波放射や送信機からの電磁波放射などは消せない。
0335ご冗談でしょう?名無しさん
垢版 |
2019/11/18(月) 13:23:20.04ID:???
重力場のない慣性空間で静止している観測者が定加速運動している荷電粒子を観測すると電磁波放射がある。
それを加速系(リンドラー座標)で静止している観測者が見ると時間変化しない重力場中で静止している電荷なので放射はない(時間変化しない重力場中なら遅延ポテンシャルが定義できる)。
ただしリンドラー座標には事象の地平線があるので、そこで静止している観測者は地平線の先は観測できず、外で放射があっても矛盾はない。
慣性系で静止している観測者はリンドラー座標で加速運動し、電荷による電場は空間で一様じゃないので電磁場の変化を観測する。
そして地平線を越えると波動帯に入り放射を観測する。
(電波源の近くでは電磁場の変化が波動なのか判らない。離れると波動放射が判るので、放射が判る領域を波動帯と言う)
次に、シュバルツシルト重力場で円運動(自由落下)してる荷電粒子を考えると、静止観測者から見れば電磁波放射がある。
それを回転系で見れば時間変化しない重力場中で静止している電荷なので放射はない。
ただし回転系には回転速度が光速に達する距離に事象の地平線があり、その外に放射があっても矛盾はない。
地平線には電磁場が誘導されるので電荷が減速し、放射エネルギーとなる。
0336ご冗談でしょう?名無しさん
垢版 |
2019/11/18(月) 13:25:50.54ID:APc4PmVn
漁師効果入れなきゃ、
0337ご冗談でしょう?名無しさん
垢版 |
2019/11/18(月) 14:46:17.36ID:???
>>335
荷電粒子からの制動放射が事象の地平面の向こうに吸い込まれるから
加速系の人には観測できないってのはもっともらしいけど、
初期状態として荷電粒子自体が事象の地平面の向こうにあるわけじゃないから
荷電粒子と事象の地平面の間にいる加速系の観測者はこの制動放射を検出しちゃう気がするんだけど。
どういう仕組みでこれを回避しているんだろ。
0338ご冗談でしょう?名無しさん
垢版 |
2019/11/18(月) 16:08:08.28ID:???
>>337
>荷電粒子からの制動放射が事象の地平面の向こうに吸い込まれる
いやそれは何かおかしい。

等加速度系(リンドラー座標系)の原点に荷電粒子が静止しており、そこから加速方向に対して
水平に離れたところに観測者がいるとする。
(一様重力場に例えれば地表上の2点に荷電粒子と観測者が静止しているとする)

ある時観測者が自由落下を始めると、その瞬間観測者は荷電粒子から制動放射が発射されるのを観測する。
その瞬間から観測者は慣性系にいるのだから、制動放射は初速ゼロ、加速度gの荷電粒子から放たれるものと同じであり、
荷電粒子から観測者に向けたまっすぐに飛んできて、しかもラーモアの公式に従って観測者方向に一番強く放射される。

荷電粒子と観測者の距離がそんなに離れてなければその放射は等加速度系にとっての事象の地平面に到達する前に観測者に届く。
その放射の軌跡上に等加速度系の観測者がもう一人いるとすると、制動放射はその観測者を貫いて自由落下の観測者に届く。
(少なくとも自由落下の観測者は貫くのを見る)

だから、英語版Wikipediaの記述が正しければ(等加速度系にいる観測者にはいかなる放射も検出しない)、
>荷電粒子からの制動放射が事象の地平面の向こうに吸い込まれる
のではなく、放射自体がもとから発生しなかったと言わざるを得ない。

つまり、慣性系で記述した制動放射の方程式をリンドラー座標系に変換してやると、きれいにゼロになるとしか説明できない。
0340ご冗談でしょう?名無しさん
垢版 |
2019/11/18(月) 17:17:45.52ID:???
ウンルー効果みたいに加速系に独特の効果があるならば
仮に静止荷電粒子からの制動放射があったとしても違和感はないね。
0341ご冗談でしょう?名無しさん
垢版 |
2019/11/18(月) 17:21:01.65ID:APc4PmVn
ないよ
せいぜい、ウンコー放射の産卵くらい
0344ご冗談でしょう?名無しさん
垢版 |
2019/11/18(月) 17:40:23.69ID:???
上に記載の英語によると、
同じ地点にいる観測者でも慣性系にいる観測者には制動放射によるエネルギーの流出が認められ、
加速度系にいる観測者には制動放射は認められないという。

同じ地点にいてもエネルギーの検出について観測者によって見解が異なるというのは不可解だな。
0346ご冗談でしょう?名無しさん
垢版 |
2019/11/18(月) 18:32:00.54ID:pA8oYISv
ホーキンス輻射は無視して、パルサーのような恒星は、電磁波を出してるよな。
ということは、慣性運動してても荷電体の電磁波が観測されるわけか。
なら、自由落下の荷電粒子だろうが、公転運動の荷電粒子だろうが、電磁波出てることにならね?
観測者が自由落下しているか、重力場に留まっているかで、電磁波の有無が逆になるのであれば、
なんとなく理解できるな。
ニュートン力学での「慣性加速度」が、荷電粒子の電磁波の論拠になるって考えれば、
自由落下の荷電粒子は電磁波を出すし、重力場に留まる荷電粒子は、「重力加速度」と相殺されて、
電磁波が出ない。
0347ご冗談でしょう?名無しさん
垢版 |
2019/11/18(月) 18:46:54.38ID:APc4PmVn
>>346
>自由落下の荷電粒子
球対称重力場は平行場ではないので、ごくごくごくごくごく弱い制動放射は起きる。
磁場があれば荷電粒子は螺旋運動して、シンクロトロン放射する。それだけ
0348ご冗談でしょう?名無しさん
垢版 |
2019/11/18(月) 18:50:19.60ID:???
パルサーが出している電磁波は単なる回転磁気双極子による電磁放射であって、
重力場内を運動ないし停止している荷電粒子が電磁波を出すか否かの判断材料にはなるまい
0350ご冗談でしょう?名無しさん
垢版 |
2019/11/18(月) 21:06:25.52ID:???
>>346
>ニュートン力学での「慣性加速度」が、荷電粒子の電磁波の論拠になるって考えれば、
自由落下の荷電粒子は電磁波を出す・・・

それが、ニュートン力学の慣性力と一般相対性理論の重力場の違いによる、効果の1つになるが
他の一般相対性理論の効果の観測と同じく、重力が弱い地球上の実験で検証することが非常に難しい。
0351ご冗談でしょう?名無しさん
垢版 |
2019/11/18(月) 21:45:20.38ID:4DoU2Xfq
慶應の白鳥先生
0352ご冗談でしょう?名無しさん
垢版 |
2019/11/18(月) 22:05:28.30ID:???
パンツを盗んで逃げる速度をv_pとする
追跡者の速度をv_cとすると
パンツを盗んだ教授と追跡者を含めた全系のハミルトニアンはどうなるか
0353ご冗談でしょう?名無しさん
垢版 |
2019/11/19(火) 02:35:24.38ID:P4nGu16t
示談にしてよ
飯塚よりマシだろ
優秀な研究成果とパンツを天秤にかけるまでもない
パンツなんて単なるセルロース

未来のノーベル賞候補
0354ご冗談でしょう?名無しさん
垢版 |
2019/11/19(火) 12:19:03.16ID:QK8K10ho
物理学を勉強するために数学を勉強してるんだけど、これ数学だけで4年終わらないか?
ルベーグ積分とかヒルベルト空間とかワケワカメ
0356ご冗談でしょう?名無しさん
垢版 |
2019/11/19(火) 12:20:55.01ID:hSap70mi
無能を自覚して諦めればすぐ終わる。
0357ご冗談でしょう?名無しさん
垢版 |
2019/11/19(火) 12:38:29.07ID:QK8K10ho
お前らって本当にデルタ関数を定義するために超関数論を勉強したり相対性理論をやるためにリーマン幾何学を勉強したりしてんの?
数学科の平均的な学生より数学に詳しそう
0360ご冗談でしょう?名無しさん
垢版 |
2019/11/19(火) 12:56:53.32ID:???
物理学の能力適正がある人は、現実の複雑な物理現象から共通な概念を洞察する能力
が優れている。
無いと自覚した人は、物理系と関係が薄い職業選択を早くしたほうが幸せになれる。
物理学に道具として必要な数学理論は厳密に理解する必要はない、数学者の仕事だ。
0361ご冗談でしょう?名無しさん
垢版 |
2019/11/19(火) 12:57:08.89ID:hSap70mi
道を間違えたんだな。土壌は救わないけど
0362ご冗談でしょう?名無しさん
垢版 |
2019/11/19(火) 13:26:20.72ID:???
>>360
ガリレオ・ガリレイは静止している物体と等速運動の物体が同じであり、
静止している物体と自由落下の物体は同じであると洞察した。
当時の数学は代数学、微積分学が未完成であり、物理理論として構成することができなかった。
現代の一般人にも天才ガリレイの発想そのものが理解できない。
0363ご冗談でしょう?名無しさん
垢版 |
2019/11/19(火) 13:31:34.67ID:???
俺は物理に必要だから数学科に入った
在学中は数学の方が面白くなってしまったが
仕事では物理に戻って「場の古典論」で自習した
数学やってたおかげで楽勝だった
0364ご冗談でしょう?名無しさん
垢版 |
2019/11/19(火) 13:36:56.60ID:9o9pa7b5
なんか茂木さんが劣等生のように書かれてるが
茂木さんはここにいる誰よりも物理ができるだろ
0365ご冗談でしょう?名無しさん
垢版 |
2019/11/19(火) 13:52:09.06ID:hSap70mi
へぇー
0367ご冗談でしょう?名無しさん
垢版 |
2019/11/19(火) 14:29:00.19ID:???
オカルト茂木は、韓国で反日活動を続ける同類のオカルト鳩山夫婦と親しく団体に所属している。
0368ご冗談でしょう?名無しさん
垢版 |
2019/11/19(火) 15:01:09.68ID:???
茂木は科学研究の適性が無いことを自覚し、オカルト・エンターテーメント分野で
大金を稼いだ大成功者。 科学の適性が無い人は模範にすべき人物。
0369ご冗談でしょう?名無しさん
垢版 |
2019/11/19(火) 15:14:28.52ID:???
ガリレイは一般人に理解不能な物理概念を公言して宗教裁判にかけられ軟禁状態にされたが
茂木は一般人どころか馬鹿でも感じる感覚のオカルト・エンターテーメントで大成功。
0372ご冗談でしょう?名無しさん
垢版 |
2019/11/19(火) 18:33:17.27ID:W/u21z1p
↓のような説明が書いてある本がないのはなぜでしょうか?
↓のような説明だと回転行列を使って説明できて便利だと思います。
普通は、ベクトル r と、ある座標系と、それを回転させた座標系を描いて、幾何学的に説明します。



2次元のある座標系で (x, y) と表される点は、座標系を θ だけ回転した座標系では、

X = cos(θ) * x + sin(θ) * y
Y = -sin(θ) * x + cos(θ) * y

である X, Y を用いて、 (X, Y) と表されます。

このことを説明するのに、新しい座標系から見たベクトル r の見え方は、
古い座標系から見た r を -θ だけ回転させた r' を古い座標系から見たときの見え方に等しい。

よって、ベクトル (x, y) を -θ だけ回転させたベクトルの成分と (X, Y) は等しい。
0373ご冗談でしょう?名無しさん
垢版 |
2019/11/19(火) 18:56:33.26ID:W/u21z1p
数学では、物理のように基底が時間とともに変化するような状況は扱わないのはなぜですか?
0374ご冗談でしょう?名無しさん
垢版 |
2019/11/19(火) 19:19:11.22ID:QK8K10ho
数学ではcoordinate-freeといって座標系を使わないで議論するのが好まれる
0375ご冗談でしょう?名無しさん
垢版 |
2019/11/19(火) 19:23:35.34ID:LDxsOD6g
宇宙空間にある天体を動かすにはどのような力が必要になってくるのでしょうか?
0376ご冗談でしょう?名無しさん
垢版 |
2019/11/19(火) 19:29:39.71ID:LDxsOD6g
>>375
単純に動かしたい方向に外力を加えるだけでいいんでしょうか、どなたか教えてください
0377ご冗談でしょう?名無しさん
垢版 |
2019/11/19(火) 20:13:11.80ID:???
大学院修了した人に聞きたいです。

東大の物理研究科に行きたいんだけど、希望する研究室が柏キャンパスで、授業は本郷らしいのだが、もしかして柏から本郷へ通いで行くんですかね?
それとも授業は柏でやってくれるんですか?訳が分からない。何で研究室と授業のキャンパスがバラバラなのか。一般的な東大大学院生は柏か本郷かどちらに住むのですか?(研究室が柏の場合)
0380ご冗談でしょう?名無しさん
垢版 |
2019/11/19(火) 23:41:44.67ID:???
宇宙は膨張しているそうですが
膨張しているという事は
時間を巻き戻すと、膨張前になると思うのですが
何がきっかけで膨張したのでしょうか?
0384ご冗談でしょう?名無しさん
垢版 |
2019/11/20(水) 00:04:12.90ID:xYFdeysH
>>380
宇宙が膨張って、してないでしょ
太陽系の惑星間の距離が膨張しているか?
アンドロメダが遠ざかっているか
膨張と定常の境界はどこだよ、境界面が凄いことになるわ
0386ご冗談でしょう?名無しさん
垢版 |
2019/11/20(水) 01:57:02.97ID:pexJysPh
>>366-399
お前は茂木さんを貶めるほどの知能を持ってないだろwww
0392建築木造
垢版 |
2019/11/20(水) 12:53:58.64ID:???
小保方さん、いまアメリカで研究つづけてるんでしょ?
0393ご冗談でしょう?名無しさん
垢版 |
2019/11/20(水) 13:00:51.02ID:???
β壊変では中性子が崩壊して電子と陽子になりますが
このまますべての原子の中性子がβ壊変すると宇宙の構造はどうなるのでしょうか
1H→1H
2D→2He
4He→4Be
6Li→6C
8Be→8O
10B→10Ne
12C→12Mg

全体に軽くなりますか
0399ご冗談でしょう?名無しさん
垢版 |
2019/11/20(水) 17:19:29.89ID:WRaig9A+
超新星直前の恒星の内部構造についてだけど、
以下のPDFの16〜17ページでは、
太陽の20倍の恒星内部に出来る鉄の核よりも太陽の15倍の質量の恒星の内部に出来る鉄の核の方が大きくなっている。
なぜ質量が逆転してしまっているんだ?

超新星前兆ニュートリノと重力崩壊直前における大質量星の内部構造進化
http://www.lowbg.org/ugnd/workshop/groupC/sn20160106/files/sn2016_08.pdf
0400ご冗談でしょう?名無しさん
垢版 |
2019/11/20(水) 17:38:04.80ID:jOCs4A+K
>>387
むしろお前がバカ
茂木さんはアインシュタインの相対性理論の本を書いてる
お前は本を書くどころか相対性理論を理解してないだろ
0403ご冗談でしょう?名無しさん
垢版 |
2019/11/20(水) 22:51:27.76ID:xYFdeysH
茂木さんは合理的な考え方で人気あるよ
脳何でも相談室は自分もよく読みに行く

人間(脳)には自由意志はないとしている
何故なら物理主義的世界観と整合性がとれないから
0405ご冗談でしょう?名無しさん
垢版 |
2019/11/21(木) 00:19:55.90ID:am2A7k53
>>399
>なぜ質量が逆転してしまっているんだ?
模式図から定量的なこと読み取ることは幼稚だよ
0406ご冗談でしょう?名無しさん
垢版 |
2019/11/21(木) 00:28:58.82ID:Omm+C4nE
>>405
>模式図から定量的なこと読み取ることは幼稚だよ

何が言いたいんだ?
17ページの表は、初期質量が15Mの恒星には1.5Mの鉄の芯ができて
初期質量が20Mの恒星には1.44Mの鉄の芯ができると言う意味で間違いないだろ?
0407ご冗談でしょう?名無しさん
垢版 |
2019/11/21(木) 00:37:31.12ID:Npxubwlh
茂木って受動意識仮説(笑)とか信じてるオカルト人だろ
0410ご冗談でしょう?名無しさん
垢版 |
2019/11/21(木) 01:45:17.42ID:kgVAVfB3
茂木健一郎は脳科学者だぞ
何だお前ら

脳科学の本をちょっと読めば意識なんて幻想で、何なる物理現象とわかるはず

物理の事を少し勉強すれば、この世界は物理法則が全てとわかるのと同じ
0414ご冗談でしょう?名無しさん
垢版 |
2019/11/21(木) 11:46:14.66ID:???
太陽光のように、宇宙線を集めるレンズのようなものはできないのでしょうか
それができれば加速装置がなくてもガンの治療に有効活用できます
0416ご冗談でしょう?名無しさん
垢版 |
2019/11/21(木) 12:45:23.03ID:???
物理現象かどうか分類することと、物理学の理論で解析可能かは全く異なる。
例えば、現代の量子論の方程式で高分子の問題すら解析的に解けないし、スパコンで
も困難だ。
つまり階層別に解析する科学、化学、分子生物学、生物学、行動科学、心理学などが
物理学の理論に置き換わることはない。
「物理現象」と称して、科学的な方法で実際に研究解析していなければ、疑似科学説
になる。
0417ご冗談でしょう?名無しさん
垢版 |
2019/11/21(木) 12:51:11.60ID:KhQbpkF+
>>411
相対性理論の本を書いてる
脳科学者にして物理学者
0420ご冗談でしょう?名無しさん
垢版 |
2019/11/21(木) 13:15:06.84ID:am2A7k53
>>417
相対性理論を本当に理解してる学者は世界で三人程度らしいじゃん。
茂木さんや竹内さんは、やっぱ頭いいんだよ。
0421ご冗談でしょう?名無しさん
垢版 |
2019/11/21(木) 13:44:51.02ID:???
>>420
>相対性理論を本当に理解してる学者は世界で三人程度らしいじゃん。
(相対論が出た直後の)1905年ごろならそんなこともあったかもな。
100年以上たった現代でそれはないわ。
0422ご冗談でしょう?名無しさん
垢版 |
2019/11/21(木) 15:17:15.54ID:???
>>420
低レベルの5ちゃん物理でさえスレチっぽいだけで
物理学や科学研究者として無名でも、疑似科学ジャーナリストで成功し金稼げたからそれでいいでない。
0423ご冗談でしょう?名無しさん
垢版 |
2019/11/21(木) 16:25:00.74ID:am2A7k53
>>421
>1905年ごろ
それ相対性理論の生まれる前じゃないか。
0424ご冗談でしょう?名無しさん
垢版 |
2019/11/21(木) 16:45:43.13ID:???
韓国の原子物理学者によれば実験用プルトニウム爆弾1個、実戦用核爆弾2個の開発に6か月
開発費920億円(イージス艦一隻分)で作れる。韓国内の原発廃棄物のプルトニウムで
核爆弾4300個分。
0425ご冗談でしょう?名無しさん
垢版 |
2019/11/21(木) 17:16:02.62ID:am2A7k53
日本も早く核開発しないと、金輪際、竹島の実力奪還は不可能になるね。安倍さん頑張れ!
0426ご冗談でしょう?名無しさん
垢版 |
2019/11/21(木) 17:49:37.17ID:Pkmlq7UV
>>422
茂木さんはお前より物理の実績あるだろ
0431ご冗談でしょう?名無しさん
垢版 |
2019/11/21(木) 21:19:35.50ID:Omm+C4nE
>>399 の質問に対する答えだけど?
超新星直前にできる鉄の核について太陽の20倍の恒星と15倍の恒星で質量が逆転する理由は、対流核の有無だったりする?
ケイ素核の大きさでは、20倍だった方の恒星の物が大きいけど、15倍のケイ素核内部には対流核が有って、
ケイ素核の広い範囲が核燃料として消費される事で最終的に対流核を持たない20倍の恒星よりも大きな鉄の核ができる。

当てずっぽうだけど↑の考えで正しいでしょうか?
0432ご冗談でしょう?名無しさん
垢版 |
2019/11/21(木) 21:22:30.57ID:Omm+C4nE
>>431
対流核が無い場合は、中心部の一部しか燃料として消費されないけど、
対流核が有れば、核融合が起こる中心以外の範囲も燃焼されるという意味で言っている。
0436ご冗談でしょう?名無しさん
垢版 |
2019/11/22(金) 11:38:44.88ID:???
Q1だけで与えられたQ全体に一致する理由の説明に全くなっていません
読んでから解答していただくようお願いします
0439ご冗談でしょう?名無しさん
垢版 |
2019/11/22(金) 12:03:40.26ID:???
サイクルなのでQ+ーQ-=Wなのは理解しています
そこについてのみ解答して下さったのは分かりました
質問はQ+=Q1の理由を教えてくださいということです
0440ご冗談でしょう?名無しさん
垢版 |
2019/11/22(金) 12:12:15.89ID:???
Q+てどういう意味なんですか?
Q+が与えた熱、Q-が放出した熱かと思ってたんですけど違うんですか?
0443ご冗談でしょう?名無しさん
垢版 |
2019/11/22(金) 15:29:48.52ID:???
>>362
相対論で反変ベクトルと共変ベクトルの2つを考える理由がいまいちしっくりしません。
使い方は分かるので使うことはできるのですが、分かった気になってる状態です。

e'_μ=Λ_μ^ν*e_νで変換される共変基底e_μを選んだとき、なぜ転置行列による変換e'^μ=Λ_ν^μ*e^νを受ける反変基底もセットで考えないといけないのでしょうか?
0444ご冗談でしょう?名無しさん
垢版 |
2019/11/22(金) 15:50:02.71ID:NMolx/yz
別に普通の平面でもいいんだけど、直交座標以外を用いで、三平方定理を座標表現することを考えてみれば良い。
0446ご冗談でしょう?名無しさん
垢版 |
2019/11/22(金) 16:07:26.88ID:???
>>443
反変共変を使う理由なんて便利だからに決まってるやろ
自分は反変共変基底の変換など使わんかったがな
教科書の説明じゃ不満だったんで微分演算子の基底から
テンソルを自分で構成して反変共変成分だけで充分
自分で構成すれば理解は完璧で後はスイスイだね
0450ご冗談でしょう?名無しさん
垢版 |
2019/11/22(金) 18:01:17.97ID:???
それって任意の正方行列A(行列式は非ゼロ)について、
A^-1*A=I
という当たり前のことを言い換えてるに過ぎませんか?

なんでそんなことに仰々しく共変とか反変とか名が与えられて明確な区別がなされているのかわかりません。
0453ご冗談でしょう?名無しさん
垢版 |
2019/11/22(金) 18:21:51.39ID:NMolx/yz
虹形式だからだよ。
0454ご冗談でしょう?名無しさん
垢版 |
2019/11/22(金) 18:32:57.89ID:109a/c2J
>>445
>どっちにしろヘリウム燃焼が始まったら対流核などないだろ

ニュートリノが熱を持ち去るから対流する必要ないと言いたいのか?
対流核が無ければ、一番温度が高い部分しか燃焼しないじゃないか?
これは、ヘリウム核の中に大きな炭素酸素核ができる事と矛盾するよ?
0455ご冗談でしょう?名無しさん
垢版 |
2019/11/22(金) 18:43:02.01ID:???
>>450
基底ベクトルが共変的に振る舞うなら、成分は反変的に振舞います

それがわかってれば十分ですよ

極座標とか複雑な座標変換考え始めたとしても、これが基本になるわけですね
0456ご冗談でしょう?名無しさん
垢版 |
2019/11/22(金) 19:10:24.16ID:109a/c2J
>>445
>どっちにしろヘリウム燃焼が始まったら対流核などないだろ

確認の為書き込んでおくけど、
>>431 は、ヘリウム燃焼の話をしているのではなく、ケイ素燃焼の話をしている。
0457ご冗談でしょう?名無しさん
垢版 |
2019/11/22(金) 19:48:09.87ID:???
共変反変の意義に拘っていては先に進めないのですね。
共変反変の作法は物理学者の美学というか様式美としてとらえておきます。
0458ご冗談でしょう?名無しさん
垢版 |
2019/11/22(金) 20:01:35.42ID:dMHUv7zd
共変、反変はなぜ難しいと思う人が多いんですが?
0459ご冗談でしょう?名無しさん
垢版 |
2019/11/22(金) 20:10:23.00ID:NMolx/yz
直交座標に慣れすぎてんだな
0460ご冗談でしょう?名無しさん
垢版 |
2019/11/22(金) 22:33:17.71ID:dMHUv7zd
A Student's Guide to Vectors and Tensors (Student's Guides) ペーパーバック ? 2011/9/22
Daniel Fleisch (著)

↑ってどうですか?
0462ご冗談でしょう?名無しさん
垢版 |
2019/11/22(金) 23:21:09.39ID:dMHUv7zd
線形代数の教科書で「テンソル」が扱われていることがあります。

例えば、佐武一郎さんの本などでです。

それと物理でいうテンソルは同じものなんですか?
0463ご冗談でしょう?名無しさん
垢版 |
2019/11/22(金) 23:26:57.62ID:dMHUv7zd
で、もし同じものだとしての話ですが、物理の本でテンソルを勉強したほうがいいのでしょうか?

それとも佐武さんの本のような数学者の書いた本で勉強した方がいいのでしょうか?
0464ご冗談でしょう?名無しさん
垢版 |
2019/11/22(金) 23:42:42.71ID:???
ある座標系xをとったとき、xの反変ベクトルをyとするとxとyは共変・反変の変換則gで移りあう:
y=g・x また x=g^-1・y

同じ空間で別の座標ξをとったとき、ξの反変ベクトルをηとするとξとηは共変・反変の変換則hで移りあう:
η=h・ξ また ξ=h^-1・η

また座標系xと座標系ξは座標変換Λによって移りあう:
ξ=Λ・x また x=Λ^-1・ξ

相対論は似たような変換が沢山あって難しい。。。
0465ご冗談でしょう?名無しさん
垢版 |
2019/11/23(土) 00:17:30.43ID:???
>>462
同じですけど物理の文脈で出てくるテンソルはテンソルの成分しか見ないのでちゃんとわかってる人はあまりいないかもしれませんね
0469ご冗談でしょう?名無しさん
垢版 |
2019/11/23(土) 07:05:51.30ID:???
すみません、
n次元のベクトル空間で、線型独立なベクトルがn個見つかったら、それが基底になるって友達言ってるんですが、なんでですか?
0470ご冗談でしょう?名無しさん
垢版 |
2019/11/23(土) 07:14:02.37ID:???
それが基底にならなかったら、線形独立なベクトルがn+1個以上あることになって、"n次元のベクトル空間"という前提条件に反するからだよ。
0472ご冗談でしょう?名無しさん
垢版 |
2019/11/23(土) 10:32:02.77ID:7WT3mNoe
次元と等しいです。
0476ご冗談でしょう?名無しさん
垢版 |
2019/11/23(土) 12:24:38.71ID:???
アインシュタインやハイゼンベルクが線形代数学を直に知らなくても
特殊相対性理論と量子力学を建設したのだから、天才の創造力は凡人には理解できない。
それで
一般人には線形代数の基礎を学校で刷り込ませてから、後世の物理学者が教科書用に書き直した
(演算規則どおりやれれば馬鹿でもわかる)特殊相対性理論や量子力学を学習させる。
0478ご冗談でしょう?名無しさん
垢版 |
2019/11/23(土) 12:51:37.98ID:???
>>450
線形変換ごときで共変反変なんぞ出すなよ
一般座標変換だからこそ便利で有り難みがある
とはいえ共変反変は線形空間の双対関係ではあるがな
元は共変と反変の関係は双対空間だから別々のベクトルだが
相対論じゃ同じベクトルの別表現に過ぎないから混乱するんだろう
0479ご冗談でしょう?名無しさん
垢版 |
2019/11/23(土) 17:09:46.46ID:ykTuJMUf
>>427
相対性理論の本を書いてるだろ
お前には書けないし、理解もできないだろうがな
0480ご冗談でしょう?名無しさん
垢版 |
2019/11/23(土) 17:17:23.80ID:7WT3mNoe
よく、お茶は最後の一滴まで入れるみたい説明を見るのですが、これは物理学的に合理的なことなのでしょうか?

まず「最後」というのがよく分かりません。待っていれば次から次へお茶の滴が落ちてきます。
0481ご冗談でしょう?名無しさん
垢版 |
2019/11/23(土) 17:18:19.39ID:7WT3mNoe
最後の一滴が一番おいしいみたいな主張だと思います。
0482ご冗談でしょう?名無しさん
垢版 |
2019/11/23(土) 18:22:29.15ID:aEJvrBdI
熱力学の第二法則って統計力学的に導出できるんでしたっけ?基礎方程式が時間可逆だから不可能な気がしてます。
統計力学が熱力学を基礎づけるとよく言われるのですが実は限定的にしか基礎づけてないのかな?とも思います。どう考えます?
0485ご冗談でしょう?名無しさん
垢版 |
2019/11/23(土) 19:37:35.90ID:???
>>482
熱力学の第二法則は、熱力学の第一法則(エネルギー保存)と同じ原理である
(巨視的な)熱力学の理論全体は矛盾が無く現実の実験と矛盾しない。
統計力学は、(巨視的な)熱力学の法則を分子運動論で統計的に説明する理論

ただし、莫大な質点の多体系の運動方程式を解析的に解くことは不可能で予測不可
統計力学の熱力学の第一法則二法則に相当する原理は、エネルギーの当配分や
ボルツマンの原理などが必要になるということ。
例えば、数個の質点運動の運動方程式を解いた物理解釈からは熱力学の法則が出てこない。
0487ご冗談でしょう?名無しさん
垢版 |
2019/11/23(土) 19:56:15.70ID:RMMLhEnR
カタコト物理学でした
0489ご冗談でしょう?名無しさん
垢版 |
2019/11/23(土) 21:03:11.32ID:aEJvrBdI
>>485
つまりどういうこと?
よくわからなかった。
0490ご冗談でしょう?名無しさん
垢版 |
2019/11/23(土) 21:05:43.13ID:aEJvrBdI
的に解くことは不可能で予測不可
統計力学の熱力学の第一法則二法則に相当する原理は、エネルギーの当配分や
ボルツマンの原理などが必要になるということ。

第二法則でてきます?統計力学から。
0491ご冗談でしょう?名無しさん
垢版 |
2019/11/23(土) 21:08:40.24ID:7WT3mNoe
よく、お茶は最後の一滴まで入れるみたい説明を見るのですが、これは物理学的に合理的なことなのでしょうか?

まず「最後」というのがよく分かりません。待っていれば次から次へお茶の滴が落ちてきます。

最後の一滴が一番おいしいみたいな主張だと思います。
0502ご冗談でしょう?名無しさん
垢版 |
2019/11/24(日) 00:57:10.05ID:???
オームの法則みたいに低エネルギー領域では近似的に線形ですが、高エネルギーになると高次の項が効いて非線形になります
はストーリーとして単純だし理解できる

ただ、量子論や相対論触ってるとなにか線形であることが本質的な気がしてくる
アインシュタイン方程式自体は非線形でも一般座標変換は線形変換だし、ユニタリ性なんか線形であることより強力な要請だし
0503ご冗談でしょう?名無しさん
垢版 |
2019/11/24(日) 01:09:12.06ID:9IAq1Q5m
その調子。どんどん混乱してね
0504ご冗談でしょう?名無しさん
垢版 |
2019/11/24(日) 02:53:36.98ID:???
>>272
だったら重力場中にある(止まった)電子はみんな電磁波出してんの?
あ、止まれないのか、不確定性があるから。

電子を自由落下させるってどーやんの?
0505ご冗談でしょう?名無しさん
垢版 |
2019/11/24(日) 05:26:24.58ID:SGzrWSUn
>>494
第二法則どうやって出してます?
つまり力学以外のどの仮定から?
0506ご冗談でしょう?名無しさん
垢版 |
2019/11/24(日) 05:34:09.27ID:SGzrWSUn
等分配の仮定だけからは出ないよね?
例えば、2つの温度の異なる理想気体の入った箱をくっつけてしきりをとってひとつの大きな箱にする。最初は右側が高温、左が低温(平衡状態にない)、時間がたつと気体が混ざり、温度も均一になっていく(平衡状態になる)。
この過程でエントロピーは増える。

これって統計力学的にどうやったら記述できるんだっけ?
0508ご冗談でしょう?名無しさん
垢版 |
2019/11/24(日) 11:36:41.20ID:???
>>506
簡単に解釈すれば、熱平衡状態の粒子は唯一の運動量(エネルギー)分布になる。

閉じた系の熱平衡状態では、エネルギー保存(第1法則)、ボルツマン分布(第2法則)が原理として成り立つ。
2つの温度の異なる閉じた系を合わせた直後は熱平衡状態でないから分子運動量の分配
が起こる(熱エネルギーの移動)
合わせた系が唯一のボルツマン分布(熱平衡状態)になるまで続くこれがエントロピー増大。
0509ご冗談でしょう?名無しさん
垢版 |
2019/11/24(日) 12:04:13.59ID:sFa29Znu
>>508
そうなんだけど、平衡分布にない状態からボルツマン分布に至る過程を統計力学的な方程式で書けるかな?というのが疑問です。
書けないと思っているんだけど。
0511ご冗談でしょう?名無しさん
垢版 |
2019/11/24(日) 12:45:39.63ID:9IAq1Q5m
非平衡統計力学なんてのは超難問で、特別な場合を除いて、まだ駅上がっちゃいないよ
0512ご冗談でしょう?名無しさん
垢版 |
2019/11/24(日) 12:51:12.68ID:SGzrWSUn
そう。拡散方程式のような時間非可逆な方程式が統計力学の前提からどうしたら出てくるかという疑問です。

平衡状態にない分布から出発してリウビル方程式で時間発展を追っても平衡状態の分布には到達しないよね。そこが不思議なんだけど。
先の箱をくっつけて非平衡から平衡へ緩和する過程では位相空間内での分布が拡がらないといけないんだけど、密度分布関数ρの体積も密度も時間発展で変わらないからエントロピーは増えない。
0513ご冗談でしょう?名無しさん
垢版 |
2019/11/24(日) 12:53:16.41ID:SGzrWSUn
>>511
それは分かっている。
平衡への緩和過程に関して何か気の利いた説明は知らないですか?
0514ご冗談でしょう?名無しさん
垢版 |
2019/11/24(日) 12:53:54.26ID:???
>>504
>だったら重力場中にある(止まった)電子はみんな電磁波出してんの?

天才ガリレオは静止してる物体、等速直線運動の物体、自由落下の物体は同じ物理状態と洞察した。
ニュートンは静止してる物体、等速直線運動の物体は同じ物理状態としたニュートン力学を創造した。
アインシュタインはさらにガリレオの洞察を再発見して一般相対性理論を創造した。
つまり
荷電粒子と静止している(加速も含めた)座標系での観測では荷電粒子は電磁波を放射しない。
加速運動するロケット内部の座標系で静止した荷電粒子の観測でも電磁波を放射しない。
0516ご冗談でしょう?名無しさん
垢版 |
2019/11/24(日) 14:00:44.80ID:SGzrWSUn
ほぼ講義で。

読んだのは、古いよ。
ランダウ、高橋こう、久保亮五、あべりゅうぞう、トルマン、並木さんの解析力学あたり。
新しいのだとうまい説明ありますか?
0517ご冗談でしょう?名無しさん
垢版 |
2019/11/24(日) 14:01:59.87ID:???
電子の移動方向と電流の向きは逆になるっていうけど
それってつまり正電荷が高い電位から低い電位に移動するのと負電荷が低い電位から高い電位に移動するのは電磁気学的に全く等価ってことなんですか?
0518ご冗談でしょう?名無しさん
垢版 |
2019/11/24(日) 14:15:18.72ID:???
>>517
そうだね
電流の向きは正電荷の運動の方向と定義している。
歴史的に負電荷の電子が後から発見されただけ、マックスウェルも導線電流の物性を知らなかった。
0519ご冗談でしょう?名無しさん
垢版 |
2019/11/24(日) 14:19:57.86ID:9IAq1Q5m
フランクリが悪い
0523ご冗談でしょう?名無しさん
垢版 |
2019/11/24(日) 14:51:39.10ID:???
>>521
安定に存在しない×
電子と陽電子で対消滅が起こる◯
宇宙ができたときに電子だけ残った
これは時間反転が破れているからcpも破れていて
結果電子だけ残ってしまった
0526ご冗談でしょう?名無しさん
垢版 |
2019/11/24(日) 16:26:31.42ID:SGzrWSUn
質問に答えてほしい。

新しいのだとうまい説明ありますか?

あといくつか最近のをあげてくれたら有り難い。田崎さんのとか?
平衡状態を「典型性」で議論するの、話は聞いたけど、教科書でも出てくるようなら、読もうかな。
0527ご冗談でしょう?名無しさん
垢版 |
2019/11/24(日) 17:26:03.02ID:9IAq1Q5m
エルゴートからの華麗なる頭皮
0529ご冗談でしょう?名無しさん
垢版 |
2019/11/24(日) 17:42:05.94ID:SGzrWSUn
>>527
これね。
これはかなり関心あります。
直観的にはうまく行く?という期待感あります。
0530ご冗談でしょう?名無しさん
垢版 |
2019/11/24(日) 17:43:42.76ID:SGzrWSUn
>>528
一旦古典力学系でいい。
この講義ノートにはうまい説明があるってことですね。まあ見てみます。ありがとう。
0532ご冗談でしょう?名無しさん
垢版 |
2019/11/24(日) 18:21:46.10ID:???
この道を進んでも山頂には行けないので
一旦引き返して
別の道を登り直した方がいいですよ
という感じかな
0533ご冗談でしょう?名無しさん
垢版 |
2019/11/24(日) 18:26:54.26ID:SGzrWSUn
せっかくいろいろ紹介してくれて、申し訳ないですが勿体ぶった言い方でなく、端的に言ってくれたらありがたい。
でもまあ、読んで見ますよ。
0534ご冗談でしょう?名無しさん
垢版 |
2019/11/24(日) 19:30:34.69ID:SGzrWSUn
関係するところ読んだ。
4.9.2で自分の疑問にあたるところが書かれてる。式(4.64)みたいなことを自分は考えてたが、それではうまくいかないということで、やっぱそうだよね。というところまで確認。

まあエントロピーの時間発展とか言ってはいかんとして、エントロピーを見るのはやめるか。
エネルギーの分布で考えるかな。
0536ご冗談でしょう?名無しさん
垢版 |
2019/11/24(日) 20:30:28.19ID:???
ベルギー出身オランダに住む9歳のローラン・シモンズ君はオランダのアイントホーフェン工科大の
電気工学を専攻し来月には学士課程を修了する予定。
0538ご冗談でしょう?名無しさん
垢版 |
2019/11/24(日) 21:10:02.18ID:dEz6jh3+
>>534
熱力学、統計力学のスレッドへどうぞ。
0540ご冗談でしょう?名無しさん
垢版 |
2019/11/24(日) 21:51:18.38ID:Kz9vxdtd
田崎さんが公開している数学のファイルですが、

「市販されている(優れた)教科書に匹敵する品質になっていると考えている」

「市販の教科書のなかにはおそろしく質の低い物が少なからずあることを知っておくべきだろう。大学レベルの
専門書がきわめて安直に出版されていることを考えれば、この質の低さはうなずけるのだが。」

「初学者用の本ならそんなことは当たり前だと思うだろうが、そうでもない。内容を理解しているわれわれが読
んでも何が書いてあるのか分からないようないい加減な本や、解き方だけの上っ面だけを書いた人を馬鹿にし
たような本が、それなりの値段で売られているのを見ると暗澹たる気持ちになる。」

などと書いていますね。
0541ご冗談でしょう?名無しさん
垢版 |
2019/11/24(日) 23:33:53.74ID:dEz6jh3+
>>506
ちょっと待ってよ、よっぽど難しい難問で悩んでると思ったら、こんなハナシか…
たぶん数十年前から、ちゃんと書いてある教科書あるよ。
>>516 に並んでるテキストには記述ないの?
0542ご冗談でしょう?名無しさん
垢版 |
2019/11/25(月) 00:12:02.74ID:Vlg9tFU3
>>506
なぜ「最新」の本見ないの?
「現代熱力学」
ISBN:4254130856
0544ご冗談でしょう?名無しさん
垢版 |
2019/11/25(月) 00:58:38.87ID:A8PtFmZL
(特殊)相対論で物質が「光速を超えられない」理由として、
座標変換に伴う速度の合成則に修正が加えられ、つねに(光速以下の速度)+(光速以下の速度)→(光速以下の速度)だから
というのを見ますが、これは
「座標変換によっては光速を超えられない」理由=「光速以下のものはどう視点を変えても(どのように座標を取り替えても)光速以下のままである」理由
の説明に過ぎないように感じてしまいます。

これは、たとえば電子を電場で加速させ続けたとして、(それを同じ座標系で見続けても)電子はやはり光速を超えることはない、という説明になっているのでしょうか?
0545ご冗談でしょう?名無しさん
垢版 |
2019/11/25(月) 01:14:22.57ID:???
>>544
そんなことはあり得ないけど
電子が光速ならその粒子としては時間が流れなくなるので
そもそもの加速するための力を受けてないことになる
0546ご冗談でしょう?名無しさん
垢版 |
2019/11/25(月) 01:33:56.52ID:Vlg9tFU3
>>543
いや、ひょっとしたら真ん中に「の」が入る最新のと
間違えたら面白いのに…てw 5ch

ねえねえ、今から凄いこと言っていい?
0547ご冗談でしょう?名無しさん
垢版 |
2019/11/25(月) 01:49:05.46ID:Vlg9tFU3
>>534
「エントロピーの時間発展を考えてはいかん」って、どういう意味?それと、
今参照なさってるテキストのアドレス教えてくださいませんか?
0548ご冗談でしょう?名無しさん
垢版 |
2019/11/25(月) 02:01:47.87ID:Vlg9tFU3
このスレッドの5チャンネル版と2ちゃんねる版は、
全く別物になっちゃってる。
0550ご冗談でしょう?名無しさん
垢版 |
2019/11/25(月) 15:36:21.94ID:z58ojc/r
講義で先生は「しゅれでぃんがー」と発音しながら「シュレーディンガー」と板書する
なんで長音「ー」を入れるのか?
昔から伝わる教条主義か
0552名無しさん
垢版 |
2019/11/25(月) 16:12:41.15ID:a0jOZ8vq
>>547
528が言ってる清水の講義ノート
0553ご冗談でしょう?名無しさん
垢版 |
2019/11/25(月) 16:27:52.55ID:a0jOZ8vq
>>541
516の教科書にはないし他のも出てるの見たことない。リウビルの定理がある限り相空間上で時間発展を追っても平衡への緩和でエントロピーは増えない。簡単な問題ではないと思うけど。簡単?
528の紹介ので多少整理出来てきたけど。
0556ご冗談でしょう?名無しさん
垢版 |
2019/11/25(月) 19:21:18.98ID:???
>>553
ニュートン力学の解析学で、熱力学・統計力学の原理を否定したいのかな
そもそも、分子同士の衝突運動すらも運動方程式で記述不可能。
0557ご冗談でしょう?名無しさん
垢版 |
2019/11/25(月) 19:24:43.64ID:wd4Q36By
マックスウェルとかラプラス並みの頭脳があれば可能だよ。
0558ご冗談でしょう?名無しさん
垢版 |
2019/11/25(月) 19:45:51.39ID:???
>>556
日本は歴史的にギャンブルを管理・規制し続けてるせいで、一般人が確率・統計学に疎く
儲け話に騙されたり、高額保険に加入させられたりしている。
サイコロ賭博では大数の法則が成り立たなければイカサマ賭博ということになる。

熱力学・統計力学によれば、同量の20度と30度の水を合わせれば確率1(100%確実)
で25度に近づく
莫大な個々の水分子の初期状態に無関係である。サンプル実験は一回で十分ともいえる。
マトモな学生は妄想に陥って統計力学の学習で挫折しないように。
0559ご冗談でしょう?名無しさん
垢版 |
2019/11/25(月) 19:57:18.10ID:POqN1gvR
てす
0560ご冗談でしょう?名無しさん
垢版 |
2019/11/25(月) 20:02:58.14ID:a0jOZ8vq
困ったな。どうも、話が噛み合わんな。
問題意識に共感できる人のレスがほしい。
0561ご冗談でしょう?名無しさん
垢版 |
2019/11/25(月) 20:04:27.42ID:POqN1gvR
あれ?書き込めましたね
なんかBB2Cからだと書き込めなくなっちゃいました

>>553
エントロピーはあくまで平衡系に対して定義されますよね
熱平衡とは十分時間が経った状態のことですね

どんな相空間における初期状態でも、いずれは平衡状態に落ち着く
それは、相空間の特定のパターンが、マクロに見れば区別することができず、典型的であるからです
その時間発展した最終系が平衡状態であり、そこでのみエントロピーは定義されます

つまり時間発展考えてもエントロピー増大則は見えないんですよ
エントロピー増大則は熱力学の系に対して何かしらの操作を加えた時です
ピストンガチャガチャやるとかですよね
それは、力学的に見ればハミルトニアンそのものを変化させることに対応します

しかし、ハミルトニアンの変化をゆっくりにすれば、断熱定理により相空間の体積に変化はありませんね
熱力学の準静的過程ではエントロピーは変化しないということです

もしハミルトニアンの変化がゆっくりではないなら、つまり準静的過程でないのなら、断熱定理は適用できずに相空間の体積に変化があるかもしれませんね
この変化が必ず正だというのがエントロピー増大則ですね

このような文脈で式でちゃんと示すのはできてるんですかね?
よく知りませんけど
0562ご冗談でしょう?名無しさん
垢版 |
2019/11/25(月) 20:16:17.30ID:POqN1gvR
>>560
話が噛み合わないのは、わかってない人が回答しているからです
わからないのを悟られたくないから必死に関係ないことを書き込むわけですね
0564ご冗談でしょう?名無しさん
垢版 |
2019/11/25(月) 20:19:01.72ID:POqN1gvR
わからないんですね
0565ご冗談でしょう?名無しさん
垢版 |
2019/11/25(月) 20:21:32.32ID:a0jOZ8vq
>>561
どうもありがとうございます。
こういうの待ってました。

時間発展考えてもエントロピー増大則は見えないんですよ

↑が少し前にエントロピーの時間発展を追ってはいけないと書いた意味です。
で温度の違う理想気体をくっつけて平衡に緩和する過程を統計力学的に記述したい、どうすれば良いのか、というのが問題意識です。
そもそもできるのか?という意味です。

最初、うかつにもエントロピーの時間発展を追ってと考えたのでうまくいかなかった。
じゃあ、エネルギーの変化を追って、エントロピーはその都度、熱力学関数に立ち返って熱力学的な関係から計算すれば間接的にエントロピーの増えるさまが見えるかな?
と考えてますが、まだ、計算してない。
0566ご冗談でしょう?名無しさん
垢版 |
2019/11/25(月) 20:28:26.79ID:???
>>561
定常状態で定義されてると言っておきながら、時間発展うんぬんが矛盾だろ
大数の法則の極限値に近づく無数の数列の物理的意味を求めるような妄想という。

馬鹿は出来もしないことを妄想し、賢者は出来ることに全力を尽くす。
0567ご冗談でしょう?名無しさん
垢版 |
2019/11/25(月) 20:29:45.66ID:a0jOZ8vq
>>561
私はピストンガチャガチャでのエントロピー増大までは、まだ考えてなかったけど、ここで、言ってるハミルトニアンの変化って、外力に、対応するパラメータを入れておいてそれを変化させるってことですよね。
あとで計算してみるかな。
0568ご冗談でしょう?名無しさん
垢版 |
2019/11/25(月) 20:35:38.33ID:POqN1gvR
系1 粒子n個、ハミルトニアンH1、相空間3n次元

系2 粒子m個、ハミルトニアンH2、相空間3m次元


これを

系1+2 粒子 n+m個、ハミルトニアンH=H1+H2、相空間3(n+m)

こう変えた時どうなりますかってことですよね

そもそも相空間の次元から変わるので色々変わると思いますよ

系1+2での典型的な状態は、系1と系2のそれぞれの典型的な状態と合わせたものではないというだけではないんですかね

系1と系2の平衡状態を初期状態として持ってきて、合わせてHのハミルトニアンで時間発展させる
このときの初期状態は、Hにとっては典型的ではないと
0569ご冗談でしょう?名無しさん
垢版 |
2019/11/25(月) 20:38:14.42ID:POqN1gvR
>>566
わからないなら黙ってたらどうなんですかw?
典型的な状態がわからないんですね

あと、熱力学だろうが統計力学だろうが、平衡状態に緩和するという現象自体はありますし、実際に観測もできる現象ですよね

そんな子供でもわかる現象説明できないと思ってるんですかねあなたは
0570ご冗談でしょう?名無しさん
垢版 |
2019/11/25(月) 20:40:24.31ID:+rHIMVAA
>>568
あー壁のポテンシャルとかあるから、H=H1+H2ではないですね

もう壁を取っ払っちゃうってことですから、全く違うハミルトニアンになるわけです
0571ご冗談でしょう?名無しさん
垢版 |
2019/11/25(月) 20:43:35.05ID:a0jOZ8vq
>>566
当然、非平衡状態に概念を拡張して考える必要があります。
561は平衡状態で定義されてるから時間発展を追えないと説明しました。それは正しいのですが
非平衡状態に拡張して、ρ logρの積分で定義してもリウビルの定理からエントロピーは変化しないのです。結局。相空間の体積というか確率分布だけで決まってるからこうなる。
でも初期状態と平衡状態になったあとで改めてこの量を計算すると増加している。計算してるものが違うんでしょうが、
この辺りをちゃんと理解したい。
0572ご冗談でしょう?名無しさん
垢版 |
2019/11/25(月) 20:47:28.00ID:a0jOZ8vq
>>568
そうです。そうです。
モデルとしては扱いやすいので計算してみたら
いろいろわかるかなと思って。
合成系のHは系1と系2の相互作用も必要ですよね。ないと孤立系が2つあるだけなんで。、
0573ご冗談でしょう?名無しさん
垢版 |
2019/11/25(月) 20:49:54.51ID:+rHIMVAA
非平衡状態でのエントロピーてか熱力学とかってちゃんとわかってるんですかね?
0574ご冗談でしょう?名無しさん
垢版 |
2019/11/25(月) 20:50:03.85ID:a0jOZ8vq
H=H1+H2+V12
V12はHよりうんと小さい、ですね。
あとは、やってみます。
0575ご冗談でしょう?名無しさん
垢版 |
2019/11/25(月) 20:57:56.83ID:+rHIMVAA
理想気体じゃなかったんですか?
0576ご冗談でしょう?名無しさん
垢版 |
2019/11/25(月) 21:00:54.97ID:a0jOZ8vq
>>575
あっそうだ。ありがとう。
でも箱を接触前後で何か相互作用ないと、
熱の移動起きないですよね。

どうするかな。設定が難しいな。
0577ご冗談でしょう?名無しさん
垢版 |
2019/11/25(月) 21:02:23.84ID:POqN1gvR
今まで壁があったけどなくなったから行き来できるようになったんですよね

てかこんくらい別に教科書とかに乗ってるんじゃないですか?
0578ご冗談でしょう?名無しさん
垢版 |
2019/11/25(月) 21:05:45.19ID:POqN1gvR
1の粒子は1の壁と相互作用していた
2の粒子は2の壁と相互作用していた

合成すると、1と2の粒子が新しい壁と相互作用するようになった

こういう設定ですよ
粒子同士は相互作用しません
0579ご冗談でしょう?名無しさん
垢版 |
2019/11/25(月) 21:10:16.19ID:a0jOZ8vq
そう?
教科書にある?
なんか壁がいきなりとれて体積が増えるのは
そのときの仕事がどうなるんだろうとか
ちょっとややこしそうなんで壁をとるのではなく熱接触させることにします。
つまり系1 2の体積は変わらない。
で理想気体という前提もなしにします。
V12は壁を介した熱のやりとり。
ある瞬間に接触させるので階段関数とする。
0580ご冗談でしょう?名無しさん
垢版 |
2019/11/25(月) 21:12:47.89ID:a0jOZ8vq
>>578
そうか。
これだと体積が一気に増えるのでその瞬間に
相空間が増える。
考えるまでもないか。
0581ご冗談でしょう?名無しさん
垢版 |
2019/11/25(月) 21:13:51.32ID:a0jOZ8vq
579にします。
壁を取るのは当たり前過ぎました。
0582ご冗談でしょう?名無しさん
垢版 |
2019/11/25(月) 21:15:07.80ID:a0jOZ8vq
あとは自分でなんとかします。
ありがとうございました。
0583ご冗談でしょう?名無しさん
垢版 |
2019/11/25(月) 23:18:52.25ID:???
   

電子レンジでブドウを加熱するとこんなに美しく発光する
http://hayabusa9.5ch.net/test/read.cgi/news/1574690729/


11 スターダストプレス(ジパング) [JP] sage ▼ New! 2019/11/25(月) 23:08:30.04 ID:NmvrZaCn0 [1回目]
ヤバすぎワロタ

プラズマ発生って何やねん
理系説明しろや!
https://i0.wp.com/nazology.net/wp-content/uploads/Feb-19-2019-17-15-37.gif
    
0584ご冗談でしょう?名無しさん
垢版 |
2019/11/26(火) 01:34:09.21ID:R8T4EH82
>>571
もしかして、ピストンガチャガチャするといったような熱力学的操作を、ハミルトニアンが時間に依存して変化したと考えたとき、リウビルの定理より相体積変わらないからエントロピー変わるはずないんじゃないかってことだったんですかね

それに対する答えなんとなくわかった気がします

ハミルトニアンが時間に依存して変わるということはもはやエネルギー保存則が成り立たなくなりますよね
ですから、エントロピー考える時に必要なエネルギー一定の体積はエネルギーが変化するから変わると考えることができます

でも、必ずしもエネルギーが変わるとも限りませんね
体積だけ変わったけどエネルギーは変わりませんでしたー、とか
その場合は、おそらくエネルギー一定の意味する体積の範囲がハミルトニアンが変わる前と後で変わるんだと思います

リウビルの定理でわかるのは、最初に与えられた体積がどう変化するかです

ハミルトニアンH1=Eの領域は、ハミルトニアンH2=Eの領域でなくて、リウビルの定理で追える体積の時間変化というのはH1=Eでしかないんだと思います

エントロピー考えたいならH2=Eを満たすような領域の体積を求め直さなければなりません
0585ご冗談でしょう?名無しさん
垢版 |
2019/11/26(火) 01:38:29.89ID:SPy9TJF8
開放系ではエントロピーは保存しないよ
0586ご冗談でしょう?名無しさん
垢版 |
2019/11/26(火) 01:39:57.04ID:R8T4EH82
エントロピーの保存って何ですか?
0589ご冗談でしょう?名無しさん
垢版 |
2019/11/26(火) 09:51:58.14ID:p2Sxq+cG
>>584
ハミルトニアンが時間に陽に依存するケースまでは考えてなかったです。
0590ご冗談でしょう?名無しさん
垢版 |
2019/11/26(火) 09:52:53.40ID:p2Sxq+cG
昨日のやりとりでそっちもあるなとは気づいたのですが。
0591ご冗談でしょう?名無しさん
垢版 |
2019/11/26(火) 12:29:28.50ID:p2Sxq+cG
>>584
補足
ハミルトニアンが時間に陽に依存する場合はリウビル方程式に新たな項が追加になるので
ρの時間変化はありますね。
しかし外部から仕事をすることにもなるのでエントロピーが増えることが、おっしゃる通りエネルギーの変化による効果もあるのでそれだけで第二法則には繋がらないと思います。フリーエネルギーの増えかたとか、準静的過程かとかも考えないといけなくてややこしそうです。
0593ご冗談でしょう?名無しさん
垢版 |
2019/11/26(火) 13:00:45.06ID:Gmo5GtPK
話題の高純度フッ化水素に関連しての質問

核爆弾を小型化して核ミサイルや
核砲弾にするには、ウランを効率良く濃縮にする
高純度フッ化水素が必要だから?
プルトニウムは使用済核燃料から抽出するのにも
フッ化水素が必要?

試験管で出来る程度の高濃度ウランでは
微小量過ぎて核爆弾に至らない?
0596ご冗談でしょう?名無しさん
垢版 |
2019/11/26(火) 16:29:10.82ID:SPy9TJF8
六フッ化ウランだあれは、フッ化水素を生産できるよ。
0597ご冗談でしょう?名無しさん
垢版 |
2019/11/26(火) 18:09:34.94ID:???
日本の高純度フッ化水素は、先端半導体の製造に必要な材料でミサイルの誘導センサー
制御半導体などの軍事兵器の製造に転用できる。
韓国は日本の同盟国ではない、火器レーダー照射事件などで仮想敵国状態で弾道・巡行
ミサイルを開発しているため高純度フッ化水素などの先端材料の輸出を厳重に管理している。
0598ご冗談でしょう?名無しさん
垢版 |
2019/11/26(火) 21:38:47.36ID:???
>>593
>プルトニウムは使用済核燃料から抽出する
硝酸などの化学薬品で抽出する、日本にはIAEAに認可された再処理工場が稼動中
仮に北朝鮮などが賠償金10兆円払わなければ日本全土を核ミサイル攻撃すると脅した
国家非常事態になれば、半年以内に航空機、艦艇用のプルトニウム爆弾100発以上
製造可能な状態に維持されている。国家秘密でもなんでもない。
0599ご冗談でしょう?名無しさん
垢版 |
2019/11/26(火) 22:09:46.38ID:x8yJv7Kd
>>544
ローレンツ変換を使うのだから
当たり前のトリックだぞ。

光速不変の原理=光速を超えるものはない
というのがローレンツ変換だから
光速不変の説明にはまったくなっておらん。
アホはそこで騙されるんだよ。

くっくっく
0600ご冗談でしょう?名無しさん
垢版 |
2019/11/26(火) 22:13:47.00ID:???
光速を超える物理現象がない、という経験則をもとに特殊相対性理論が作られたのであって
相対論があるから光速を超える現象はない、などという主張は誤りです。
0601ご冗談でしょう?名無しさん
垢版 |
2019/11/26(火) 22:41:22.00ID:SPy9TJF8
マックスウェル方程式から導かれただけだろ
0605ご冗談でしょう?名無しさん
垢版 |
2019/11/27(水) 00:19:05.58ID:4PwIyUpP
茂木さんは相対性理論はアインシュタインの光を追いかける思考実験から作られたと言ってたけど
0608ご冗談でしょう?名無しさん
垢版 |
2019/11/27(水) 00:23:28.27ID:4PwIyUpP
>>607
あります
0610ご冗談でしょう?名無しさん
垢版 |
2019/11/27(水) 00:47:47.38ID:4PwIyUpP
>>609
あなたの知能が低いから理解できないのです。
0612ご冗談でしょう?名無しさん
垢版 |
2019/11/27(水) 00:58:28.32ID:4PwIyUpP
>>611
わかればいいです。
しっかりと勉強してください。
0614ご冗談でしょう?名無しさん
垢版 |
2019/11/27(水) 01:13:04.32ID:4PwIyUpP
>>613
私の言ったことがわかったのではないのですか?
あなたは勉強が足りません。
勉強が足りず知能が低いがために、私のいうことが理解できないのです。
0617ご冗談でしょう?名無しさん
垢版 |
2019/11/27(水) 11:28:38.74ID:???
ニュートン力学と特殊相対性理論(の力学)は同じ慣性の法則を原理にしている
慣性系を絶対的な基準とした力学である。
ところが5ちゃんスレで何度か(マトモな)疑問が出てくるように
特殊相対性理論(光速不変の原理)には、理論的な弱点がある。

ニュートン力学の教科書を詳しく調べれば解かるように、基本原理の3法則から
演繹されて等加速度運動や円運動の加速度座標系への座標変換が(みかけの)慣性力
を導入して運動方程式が(運動量やエネルギーの法則も)成り立ち”論理矛盾が無い”。

高速運動の実験で成り立つかどうかではなく、その物理理論の論理矛盾についての話。
おもしろい事に、同じ慣性系の理論なのに特殊相対性理論(光速不変の原理)に
(みかけの)慣性力を導入した加速度座標系への座標変換には”論理矛盾が有る”。
ゴミ箱行き。
0619ご冗談でしょう?名無しさん
垢版 |
2019/11/27(水) 12:00:52.81ID:???
>>617
さらに、ニュートン力学と電磁気学は論理矛盾してると誤解してる人が多いが
ちゃんと調べればニュートン力学と電磁気学には矛盾が無いことが解かる。

マックウェル方程式の電磁波の真空の伝播速度は、誘電率と透磁率から 1/√εμ
であって、”光速不変の原理”から与えられたものではない。
同様に電場や磁場の値もその座標系の(観測された)値で定義されている。
ニュートン力学の等加速度運動や円運動の加速度座標系への座標変換でも矛盾がない。
高速運動の実験で成り立つかどうかではなく、その物理理論の論理矛盾についての話。

誤解の原因は、物質の様な真空媒体(エーテル)を仮定して運動を考えたことにある。
0620ご冗談でしょう?名無しさん
垢版 |
2019/11/27(水) 12:03:42.46ID:4PwIyUpP
>>617
茂木さんはアインシュタインの理論は正しくて、ミンコフスキーの数学に間違えがあると言ってた。
0621ご冗談でしょう?名無しさん
垢版 |
2019/11/27(水) 12:11:26.33ID:ngId1zwP
一般相対性理論と矛盾するからね、ミンコフスキーはありえない
0622ご冗談でしょう?名無しさん
垢版 |
2019/11/27(水) 12:21:46.27ID:???
>>618
特殊相対性理論をかじった人は誰でも知ってるように
双子のパラドックスや回転円盤の座標変換を考えると矛盾が起きる
慣性力の拡張では無理筋ということ。
0624ご冗談でしょう?名無しさん
垢版 |
2019/11/27(水) 12:31:26.27ID:4PwIyUpP
>>616
茂木コンプ乙
0625ご冗談でしょう?名無しさん
垢版 |
2019/11/27(水) 12:41:20.51ID:???
>>619
一般相対性理論は、加速度系の座標変換でも論理矛盾が出ないように構成されている。
興味深いのは、アインシュタインが一般相対性理論の初期段階ではニュートン力学の
座標変換を基にして重力ポテンシャルの効果を考察していることだ。
0626ご冗談でしょう?名無しさん
垢版 |
2019/11/27(水) 13:02:36.84ID:n3s4kCS9
相間さんの特徴は特殊相対性理論だけを批判するということです
一般相対性理論は難しすぎてわからないのですね
0628ご冗談でしょう?名無しさん
垢版 |
2019/11/27(水) 13:36:03.17ID:ngId1zwP
相間は生涯のライバルだからね、
0630ご冗談でしょう?名無しさん
垢版 |
2019/11/27(水) 16:06:14.44ID:???
>>626
>一般相対性理論は難しすぎてわからない
キミに解けるのかい?
 加速度系の変換で矛盾のない一般相対論を使いたくでも、普通の数学能力の人には
非線形の連立方程式など(特別な場合以外では)手が出ない。専門でも難しい。
論理矛盾の無いニュートン力学(万有引力)と、慣性系だけの特殊相対論(光速不変)
を使い分けるしかない。
0631ご冗談でしょう?名無しさん
垢版 |
2019/11/27(水) 16:15:55.33ID:ngId1zwP
君にとってむつかしいのはよくわかった
0633ご冗談でしょう?名無しさん
垢版 |
2019/11/27(水) 16:25:05.45ID:???
洗濯した衣類の乾燥に電子レンジは使えませんが
使えるようにする方法はないのでしょうか
たとえば波長を変えるなどで
0634ご冗談でしょう?名無しさん
垢版 |
2019/11/27(水) 16:28:09.83ID:bHqyYFzk
>>630
少なくとも人工衛星飛ばす時とかは一般相対性理論の効果とか入れてるみたいですよ

簡単な等加速度系のリンドラー座標とかならとっつきやすいんじゃないですかね
リーマン幾何学的な考え方が必要になりますから、そこが乗り越えられるかどうかですね

それがわかれば、加速系というのは単に慣性系から一般座標変換により移っただけだというのがわかるかと思います

リッチテンソルは曲がってないわけですね
0636ご冗談でしょう?名無しさん
垢版 |
2019/11/27(水) 17:01:25.42ID:bHqyYFzk
>>635
あなたは加速度系一般が矛盾するみたいな書き方してるように見えたので

じゃ簡単な場合はすぐ解けると言うのはわかったんですね

よかったですね矛盾ないですよ
0637ご冗談でしょう?名無しさん
垢版 |
2019/11/27(水) 18:09:33.10ID:W/EjI09Z
>>636
お前バカだろ
解けることと無矛盾なことは同値じゃないだろwww
0638ご冗談でしょう?名無しさん
垢版 |
2019/11/27(水) 18:14:03.55ID:???
>>636
一般相対性理論(等価原理)では、特殊相対性理論やニュートン力学の座標変換とは異なるから
たとえば、一様重力のモデルでは基準位置からの重力ポテンシャルで光速度が変わる。
または時間が遅れる、この効果として自由落下の加速度運動では

2台のロケットのパラドッスの自由落下版では、糸の長さは変化せず、糸も切れない。
荷電粒子の自由落下のパラドックスでは、加速運動で電磁波を放射しない。

どちらの思考実験も実際に検証が困難だから、光格子時計で精度の高い時間遅れの検証実験が有望。
0640ご冗談でしょう?名無しさん
垢版 |
2019/11/28(木) 02:08:56.65ID:oqaaQkD2
くっくっく先生へのお詫び

私は今の今まで「遠隔作用」なる用語を、砂川等の教科書における定義(現在ではこれが最もコンセンサスを得られているものだろうと認識しており)にしたがってきたのですが、
どうやら色々な流儀があるようですね。というのも、こんなのが…

 遠隔作用という用語が人によって異なる意味で使われているようであるが、
ここでは、「物体同士が直接接していなくても力を及ぼし合う」という広い意味で使う。

                      (佐々木 節「一般相対性理論」p.76)

5ちゃんねる
0641ご冗談でしょう?名無しさん
垢版 |
2019/11/28(木) 02:17:18.84ID:oqaaQkD2
ですから、くっくっく先生のかなり独特な定義でも、さほど面妖なモノというわけでもなかったのです。
くっくっく先生、本当にどうもすみませんでした。




…でもや近接作用との堆肥で語るのであれば、やっぱり無茶苦茶だわなあ…w
0642ご冗談でしょう?名無しさん
垢版 |
2019/11/28(木) 02:25:35.58ID:oqaaQkD2
ちなみに、佐々木先生の教科書では、ニュートン重力理論の特徴の一つとして、

 遠隔作用1:物質のない空間にも重力は及ぶ

ニュートン重力を越える理論のそれとして、

 遠隔作用2:物質によって生じた重力場(時空の曲率)は真空中でも消えずに伝播する

…としてあります。
0643ご冗談でしょう?名無しさん
垢版 |
2019/11/28(木) 02:36:25.26ID:oqaaQkD2
わかんねなあ…、2ちゃんに書いたら2ちゃんのみ、
5ちゃんに書いたら5ちゃんにしか書かれてねえw
5ちゃんはえっちな広告が出るのがヤなんだよなは。

5ちゃん
0644ご冗談でしょう?名無しさん
垢版 |
2019/11/28(木) 02:57:05.90ID:oqaaQkD2
あらあ?5→2ちゃんねるには転送されてるわ。タイムラグがあるってことか。
でも2に立てた熱力学スレッドは5には見当たらないな。2→5へはコピー
してくれへんのか。

続き:つかK定義(理論)では、明らかに矛盾だらけになるんだよなは。つか、そもそも
説明にすらなってないし。つか、支離滅裂つか。遅延ポテンシャルとか持ち出したら、
即「学習」して、またねじ曲がった「定義」に変換して進化した「理論」誕生させるんだろなあ。
0645ご冗談でしょう?名無しさん
垢版 |
2019/11/28(木) 03:54:49.47ID:4PICCa47
遠隔作用=情報伝播速度が∞
近接作用=情報伝播速度が有限
0648ご冗談でしょう?名無しさん
垢版 |
2019/11/28(木) 10:53:43.65ID:???
一般相対性理論によれば、重力場の中で静止していても位置が違えば時刻も違う
スカイツリーの展望台の時計と比べて真下の地上の時計は遅れ続けている。

遠隔作用のクーロン力(qQ/r^2)なら、下に運動してる粒子間の力の距離rは
どこの時刻の距離か? 遠隔作用では距離が不明、式から力も不明になる。
0649ご冗談でしょう?名無しさん
垢版 |
2019/11/28(木) 12:13:05.81ID:???
>>648
遠隔作用のクーロン力は電磁場、重力場などの変化が伝わる速さ(光速)より非常に
遅い運動物体の近似式だということになる。
太陽系で最も速く運動する惑星は水星の楕円軌道で太陽との距離変化も大きいから
アインシュタインは遠隔作用の万有引力(クーロン力)の運動からのズレが観測できる
と確信し、一般相対性理論で計算した。
0650ご冗談でしょう?名無しさん
垢版 |
2019/11/28(木) 13:05:37.66ID:???
>>630
現象の理解に、わざわざ解く必要は無い
大抵は近似計算で充分
問題は数式サイズがデカくなることだから根気が必要
俺は数式処理に頼ってるがね
0651ご冗談でしょう?名無しさん
垢版 |
2019/11/28(木) 16:33:11.40ID:MpUrMhl5
ネオンが核融合を始める温度よりも酸素が核融合を始める温度の方が高いらしいけど何故でしょうか?
↓光崩壊とか書いてあるけど、核子の結合エネルギーがより低い酸素の方が優先的に光崩壊しないのですか?


ネオン燃焼過程
https://ja.wikipedia.org/wiki/%E3%83%8D%E3%82%AA%E3%83%B3%E7%87%83%E7%84%BC%E9%81%8E%E7%A8%8B
>このような高温では原子核の光崩壊 (photodisintegration) が無視できなくなり、ネオン原子核の一部はアルファ粒子を放出して崩壊する。
0652ご冗談でしょう?名無しさん
垢版 |
2019/11/28(木) 16:33:26.55ID:ETuv9wOv
前にも質問して未解決ですが
相対論のLovelock's theorem の証明がわからないですが初心者向けはありますか
うえのは証明つきですがあってるのかわかりません

Theorem 7.1 、7.2
On Geometric Objects, the Non-Existence of a Gravitational Stress-Energy Tensor, and the Uniqueness of the Einstein Field Equation
https://arxiv.org/abs/1808.08998

p16
ラブロックの定理 (1972)
証明は原論文 [10, 11] か教科書 [3] の解説を参照のこと。
http://www2.math.kyushu-u.ac.jp/~tanahashi/pdf/note_relativity.pdf
0653ご冗談でしょう?名無しさん
垢版 |
2019/11/28(木) 16:53:26.53ID:MpUrMhl5
>>651 質問したけど、酸素炭素の結合エネルギーまで考えたら自己解決した・・・
結合エネルギーの合計に関して・・・

ネオン-酸素-ヘリウム < 酸素-炭素-ヘリウム
0654ご冗談でしょう?名無しさん
垢版 |
2019/11/28(木) 18:19:33.29ID:???
>一般相対性理論によれば、重力場の中で静止していても位置が違えば時刻も違う
時間が違うなら「宇宙の年齢」というのはどこで測ったものか?
0655ご冗談でしょう?名無しさん
垢版 |
2019/11/28(木) 21:29:14.19ID:rYUwID4z
>>647
…っかしーなー。昔はあっちに書いても、ちゃんと5にも反映されてたのになあ。
誰か2に立てた熱力学スレッド5にコピーしてくりひん?

 熱力学かんけいの質問:現在の状況よく知らないんですが、プリゴジンの仕事って、
今では否定されたりしてるんですか?少なくともout of dateってことか。

あああああ、5チャンの仕様嫌い!ストレス貯まるわ。
0657ご冗談でしょう?名無しさん
垢版 |
2019/11/28(木) 21:37:27.71ID:QDtIOmQS
最新の観測では等方ではないはずだが
0658ご冗談でしょう?名無しさん
垢版 |
2019/11/28(木) 21:38:09.97ID:laEogmkK
>>654
宇宙論では宇宙モデルの状態を簡潔にする幾つかの条件を仮定してる。
コペルニクス原理、宇宙の一様性、現在地球上での時間換算など
0659ご冗談でしょう?名無しさん
垢版 |
2019/11/28(木) 21:45:05.42ID:30GxNMaS
>>640
遠隔作用は簡単だぞ。
作用が伝わる時間は距離に比例する。
それが遠隔作用だ。
だから、時間とは距離だとも言える。

人類は地球上で
作用反作用が一瞬で伝わるスケールでしか実験をしたことがない。

宇宙スケールで
反作用を受けるまで時間がかかるような系では
エネルギー保存則も成り立たん。
それゆえ宇宙にはエネルギーが無限にある。

エネルギー保存則に縛られているサルどもは
反作用に時間を要する本当の物理学にまったく思いが至らんバカなんだよ。

空間にエネルギーがあるとする近接作用論は
完全な妄想である。

くっくっく
0660ご冗談でしょう?名無しさん
垢版 |
2019/11/28(木) 21:53:08.89ID:30GxNMaS
>>648
ただし、特殊相対論では自転速度の速い展望台の時刻は
地上より遅れて見える。

もっとも、相対論はデタラメであるのは言うまでもなく、
重力が大きければ原子レベルで束縛されて振動が遅くなり
時計が遅れるというのが真相である。

くっくっく
0661ご冗談でしょう?名無しさん
垢版 |
2019/11/28(木) 22:05:53.80ID:???
>作用が伝わる時間は距離に比例する

って、それを近接作用および場の理論って言うんじゃない?w
0662ご冗談でしょう?名無しさん
垢版 |
2019/11/28(木) 22:08:39.88ID:???
>>657
地球から見た宇宙マイクロ波背景放射等方は等方じゃないけど、等方に見える系もあるってこと
我々が言う「宇宙の年齢」ってのはその系で測った宇宙誕生からの経過時間のことだ
0663ご冗談でしょう?名無しさん
垢版 |
2019/11/28(木) 22:09:20.44ID:30GxNMaS
>>648
電磁波は
電磁誘導と変位電流が織り成す作用が遠隔で伝わるものを
便宜的に空間波であるとしただけで、そんなものは実際には存在せん。
真空には何もない。

電磁誘導と変位電流は言わば「交流」なので
必ず時間を必要とする。それが光速となって現れる。
光など実在しないが、遠隔作用の伝達速度をそう言っているだけだ。

ところが、クーロン力は言わば「直流」なので
その伝達速度は不明である。
誰も測定したことがないのだ。

これを認識しているヤツは
世界で100人もおらんだろうな。

くっくっく
0664ご冗談でしょう?名無しさん
垢版 |
2019/11/28(木) 22:14:33.89ID:30GxNMaS
近接作用ってのは
空間にエネルギーが存在するって考えだぞ。

遠隔作用は
空間には何もない、作用は直接伝わるって考えだ。

アホかボケが
くっくっく
0667ご冗談でしょう?名無しさん
垢版 |
2019/11/28(木) 22:19:06.73ID:???
>エネルギー保存則も成り立たん。
だからとっととエネルギーを無尽蔵に生み出してくれよ
そうすりゃみんな苦苦苦にひれ伏すだろうさ
0668ご冗談でしょう?名無しさん
垢版 |
2019/11/28(木) 22:27:09.52ID:???
相間くっくっくの デタラメ三昧

>遠隔作用は 空間には何もない、作用は直接伝わる

なら電磁力が間の物質で伝播速度が遅くなったり、大質量物体があると伝播速度が変わるのかな?
0669ご冗談でしょう?名無しさん
垢版 |
2019/11/28(木) 22:27:17.56ID:30GxNMaS
1クーロンを2クーロンに瞬時に増やしたとする。
遠方で観測するのは交流である電磁波なんだよな。
直流電界が2倍になった速さを実は誰も測定したことがない。

まことにアホしかおらんのが
この地球なのである。

くっくっく
0670ご冗談でしょう?名無しさん
垢版 |
2019/11/28(木) 22:29:26.64ID:???
>1クーロンを2クーロンに瞬時に増やしたとする。

意味不明です。
何の電荷を増やすのですか?
あなたの書き込みは全般的に意味不明です。
0672ご冗談でしょう?名無しさん
垢版 |
2019/11/28(木) 22:29:48.53ID:30GxNMaS
>なら電磁力が間の物質で伝播速度が遅くなったり、大質量物体があると伝播速度が変わるのかな?

このサルは
光速が水中などで遅くなる理由を説明できないアホザルだよな。

くっくっく
0673ご冗談でしょう?名無しさん
垢版 |
2019/11/28(木) 22:33:43.76ID:30GxNMaS
光速は
真空中で最高速度である理由を説明せよ。

くっくっく
0675ご冗談でしょう?名無しさん
垢版 |
2019/11/28(木) 22:37:22.47ID:???
>>672
>遠隔作用は 空間には何もない、作用は直接伝わる
から、クックックの遠隔作用が
>光速が水中などで遅くなる理由
を誰にでも分るように説明してくれ
0677ご冗談でしょう?名無しさん
垢版 |
2019/11/28(木) 22:55:31.35ID:hUwcu6gJ
↓↓↓↓↓↓↓↓↓↓↓↓↓↓↓↓↓↓

>群速度は光速を超えられますが?

↑↑↑↑↑↑↑↑↑↑↑↑↑↑↑↑↑↑


これが物理板の実力です
専門板なのに異常にレベルが低い
せいぜい物理の少しできる高校生レベル
0678ご冗談でしょう?名無しさん
垢版 |
2019/11/28(木) 22:58:37.60ID:yrlinfaE
ここの回答者は、群速度がわからなかったんですね(笑)

他人に説教する前に自分がまずはちゃんとわかったほうがいいと思いますけど
0680ご冗談でしょう?名無しさん
垢版 |
2019/11/28(木) 23:00:54.14ID:yrlinfaE
とてもそうは思えませんけどw

てか、意味わかってたら間違えようないですよねぇ
0682ご冗談でしょう?名無しさん
垢版 |
2019/11/28(木) 23:06:16.12ID:???
マイケル・ファラディは多数の実験結果からコンデンサに蓄えた電荷が変わらないのに
間に入れた絶縁物質の種類で電極に働く力が変わる、さらに導電電流による磁力が
電流と垂直方向などから電荷同士の遠隔作用の力ではなく、空間中の電磁場と電荷の近接相互作用であることを発見した。
0684ご冗談でしょう?名無しさん
垢版 |
2019/11/28(木) 23:09:48.33ID:rYUwID4z
>>659
くっくっく先生、ご教授どうもありがとうございますした。もう少しだけ教えてはいただけないでしょうか。

1:「遠隔作用論」&「近接作用論」を英語ではなんというのでしょうか?
2:この際、もう一度電磁気を基礎から勉強し直そうと思います。ぜひ先生が、
今参照されている、もしくは、お薦めの教科書等あれば、教えては頂けないでしょうか。
0685ご冗談でしょう?名無しさん
垢版 |
2019/11/28(木) 23:27:50.30ID:yrlinfaE
>>681
EMANさんは逆に書いてますよ

恥ずかしい人ですね(笑)
0686ご冗談でしょう?名無しさん
垢版 |
2019/11/28(木) 23:33:05.54ID:???
遠隔って量子もつれくらいじゃなかったか?わからんから遠隔ってことにしとこうってくらいの意味で。
光子による電磁力はもちろん重力子仮定した時点で重力も近接だよなあ、普通。
だから近接には光速に依存した到達時間があるし遠隔にはないはずなんだが。
0687ご冗談でしょう?名無しさん
垢版 |
2019/11/28(木) 23:33:33.40ID:rYUwID4z
KKK 以外の皆様へのお詫び

パンドラの匣を開けてもぉてどもごめんなさいwww
0688ご冗談でしょう?名無しさん
垢版 |
2019/11/28(木) 23:41:14.22ID:???
古代ギリシャの時代から電気力と磁気力の存在が認識されており
古代から中世までの学者は、帯電物体同士、磁性体同士の遠隔作用だと考えていた。

近代になりデカルトやライプニッツは哲学的に、離れた物体同士に直接作用する力
などはあり得ないと考えてニュートンの遠隔作用の万有引力を批判した
ファラディによる空間中の電磁場の概念は、マックスウェルによって完成された。
一般相対性理論、場の量子論などの現代物理学は、全て近接相互作用の理論である。
0689ご冗談でしょう?名無しさん
垢版 |
2019/11/28(木) 23:43:11.02ID:???
とりあえずマクスウェル方程式は近接作用の理論なんで
999はそれを否定しているってことでええんかね?
0692ご冗談でしょう?名無しさん
垢版 |
2019/11/29(金) 00:08:12.71ID:???
>>685
emanも群速度は光速を超えてもいいと書いてますよ?
ページ内検索すれば出てきますよ
まあemanは先端速度と群速度の区別はついてるんでしょうね
0693ご冗談でしょう?名無しさん
垢版 |
2019/11/29(金) 00:08:38.24ID:???
原始人の時代から現代まで蔓延ってる、霊能力とか超能力などは遠隔作用を持ち込まないと
説明できないからな。
0695ご冗談でしょう?名無しさん
垢版 |
2019/11/29(金) 00:23:56.11ID:T8lNB/51
>>692
それは特定の場合だとも書いてますね
今は真空中を伝搬する話だと思うので、群速度は光速超えないで正解ですよ

あとEMANさんの書き方だと、位相速度も群速度も光速超えて良いということですから、あなたが群速度は光速超えても良い、とそこだけ取り出してくるのはおかしいですよね

やはりあなたが勘違いしているのだと考えたほうがしっくりきますね(笑)
0697ご冗談でしょう?名無しさん
垢版 |
2019/11/29(金) 00:35:10.51ID:???
>>695
位相速度が光速を超えてもいいのはその通りですよ?
上では群速度のやり取りがあったのでそこに乗っかっただけですよ
あなたの勘違いを私に押し付けないでくださいよ
情報伝達速度を正しく認識できていなかったのはお見通しですよ(笑)
これを認識していれば光速を超えるかの比較は群速度と位相速度ではなく群速度と情報伝達速度ですから
0698ご冗談でしょう?名無しさん
垢版 |
2019/11/29(金) 00:37:11.53ID:???
まあ群速度を情報伝達速度と勘違いしているケースは他でも見受けられますからね
このスレにいても不思議じゃありません
0700ご冗談でしょう?名無しさん
垢版 |
2019/11/29(金) 00:45:43.26ID:T8lNB/51
>>897
真空中を伝わる電磁波の話ですよね今は
わからないんですか?
0702ご冗談でしょう?名無しさん
垢版 |
2019/11/29(金) 00:52:15.70ID:???
分散がない場合は群速度と位相速度は一致するので片方しか言わないのはおかしいという指摘が的外れになりますよ
0704ご冗談でしょう?名無しさん
垢版 |
2019/11/29(金) 00:56:30.29ID:T8lNB/51
なら普通に光の速度は光速超えるでいいですよね

わざわざ群速度に限定する必要ないんですから

てか分散ない時は超えられないと思うんですけど
0705ご冗談でしょう?名無しさん
垢版 |
2019/11/29(金) 01:06:28.35ID:???
初めに群速度に限定したのは>>674ですね
そこからの流れと>>679で分散がある場合の話ですよね
分散のない状況なら678から680までのやり取りがシュール過ぎますね
どちらにせよ私より前のレスで群速度に言及されている以上群速度と言うのは当然です
0706ご冗談でしょう?名無しさん
垢版 |
2019/11/29(金) 01:25:44.06ID:???
まあ>>685の段階で位相速度が超えることしか知らなかったことは読み取れます
ちなみにemanでは先端速度というワードが登場しています
位相速度と群速度という二つの速度ではなくて情報伝達速度も含めた三つの速度を考えるべきなんです
この心構えを>>703の講座で学べるかもしれません
このスレのオフ会があるなら呼びたいものですね
0707ご冗談でしょう?名無しさん
垢版 |
2019/11/29(金) 01:31:55.27ID:VIRlE2uo
>>689
ところがね、しっかりした教科書、参考書読むと、そう単純なハナシでもないのよ。
いや間違ってるって言ってんじゃなくて。最初は遠隔作用の元格闘して、最終的に
近接作用論&場が物理的実体として認識されるまでの過程学ぶと面白いよ。太田の
三部作(できれば四部作)熟読してみてみ。


それでそこそこ
うまいこと言って、
0708ご冗談でしょう?名無しさん
垢版 |
2019/11/29(金) 01:44:28.07ID:VIRlE2uo
>>690
いったい何をreferenceとしているのか、
どういう教科書の元、どんな教育過程をたどれば
ああいった強固な思想に辿り着いたのか、理論が完成したのか、
気にならない?
0710ご冗談でしょう?名無しさん
垢版 |
2019/11/29(金) 03:26:27.21ID:???
>>709
近接作用が前提である電磁気学でも、両方が静止状態で働く電荷のクーロン力は
厳密に成り立つ。
微小電荷粒子が(光速に比べ)非常に低速で動く場合も近似的に成り立つ。

導線電流のように内部で莫大な数の荷電粒子が運動している状態、電荷の高速運動
ではクーロン力だけではもはや説明できない。
初等学校教育ではそれが上手く説明できないから、電気力と磁力の2元論に戻ってしまう。
0715ご冗談でしょう?名無しさん
垢版 |
2019/11/29(金) 08:35:45.99ID:T8lNB/51
>>706
オフ会いいですね
あなたを殺すのが楽しみです
0719ご冗談でしょう?名無しさん
垢版 |
2019/11/29(金) 12:29:45.91ID:???
>emanでは先端速度
それでも解決しない
電磁波が無い状態から電磁波が伝播していく状態ならば、”先端速度”の意味
があるが
例えば2波長の電磁波による進行波の定常状態では”先端速度”など定義できない
進行波はエネルギーを伝播している、その伝播速度は光速を超えるのか?
進行波のエネルギー伝播速度は、位相速度か群速度か?
0721ご冗談でしょう?名無しさん
垢版 |
2019/11/29(金) 13:27:48.90ID:ys1Gy+TP
先端のない波はエネルギー運びません
もう運び終わってます
0723ご冗談でしょう?名無しさん
垢版 |
2019/11/29(金) 14:22:32.63ID:ys1Gy+TP
先端のない波はエネルギー運びません
もう運び終わってます
0724ご冗談でしょう?名無しさん
垢版 |
2019/11/29(金) 14:24:02.53ID:XoH05IoA
アホをアホ呼ばわりはあかんな
0725ご冗談でしょう?名無しさん
垢版 |
2019/11/29(金) 14:31:33.21ID:???
マイクロ波送電で受信アンテナで電磁波が終端してるが、終端の位置は変化してない
電力エネルギーは進行波で伝播されてる。
繰り返しだが進行波のエネルギー伝播速度は、位相速度か群速度か?
0728ご冗談でしょう?名無しさん
垢版 |
2019/11/29(金) 14:41:08.56ID:XoH05IoA
およよ
0732ご冗談でしょう?名無しさん
垢版 |
2019/11/29(金) 15:26:44.60ID:???
「二酸化炭素を食べる大腸菌」が遺伝子操作で誕生
https://gigazine.net/news/20191128-e-coli-eat-co2/
実験室条件下で独立栄養生物に生まれ変わった大腸菌株は、
二酸化炭素濃度10%の大気中であれば、およそ18時間ごとに倍増するそうです。
--------

この細菌を二酸化炭素だけで98気圧もある金星にばらまくと
金星の二酸化炭素は固定化してしまい
地表気圧も窒素だけの3気圧になると思います
減圧すると水蒸気も水に戻って金星地表面に水がたまり
そこにシアノバクテリアを撒けば酸素が供給されるでしょう
0737ご冗談でしょう?名無しさん
垢版 |
2019/11/29(金) 17:20:24.67ID:XoH05IoA
最先端波動物理
0739ご冗談でしょう?名無しさん
垢版 |
2019/11/29(金) 17:32:08.67ID:ys1Gy+TP
昔は空を眺めるくらいしかやることなかったからです
0743ご冗談でしょう?名無しさん
垢版 |
2019/11/29(金) 18:18:18.38ID:T8lNB/51
昔の人は地球が中心なんですよ

太陽も火星も木星も全て地球を中心として回っているとしていました
0744ご冗談でしょう?名無しさん
垢版 |
2019/11/29(金) 18:42:50.98ID:XoH05IoA
現代の俺様は、俺様が中心なんだよ。一般相対性理論の教えるところだ。
0745ご冗談でしょう?名無しさん
垢版 |
2019/11/29(金) 18:48:09.96ID:???
>>743
公転という不正確なものを小中学校で教えるのをやめた方がいい
それは嘘なのだ

極端な例は木星で
太陽と木星の共通重心は、太陽表面から5万kmも離れたところにある
地球も共通重心が太陽の中あるだけで太陽の周りをまわっているわけではない
0746ご冗談でしょう?名無しさん
垢版 |
2019/11/29(金) 18:49:16.65ID:T8lNB/51
そんなこと言ったら何にも教えられなくなりますよね
0747ご冗談でしょう?名無しさん
垢版 |
2019/11/29(金) 19:29:57.83ID:???
>>745
>太陽と木星の共通重心は、太陽表面から5万kmも離れたところにある

バカの屁理屈だね、木星の公転の意味は
木星の楕円長に比べた太陽位置との差は非常に小さく無視できるという意味の”公転”だ。
0749ご冗談でしょう?名無しさん
垢版 |
2019/11/29(金) 20:09:13.56ID:0r9cXNtp
>>584
http://www.ton.scphys.kyoto-u.ac.jp/~sasa/public.pdf

似たようなことしてるの見つけた。
数理物理学的にもオモロイね。
アンサンブルをどう選ぶかがポイントで、
うまく選べば、シャノンエントロピーはそのままでは変化しないけど温度やエネルギーは平衡へ緩和するよううにできる。
面白いのはボルツマン分布から始めると熱接触しても熱は移動しない。ボルツマン分布から少しずれてるのが大事なようて。
だいぶ、スッキリ
0751ご冗談でしょう?名無しさん
垢版 |
2019/11/29(金) 20:16:10.45ID:T8lNB/51
>>748
冥王星は1930年とかすごく最近みたいですよ発見されたの

遠くの星だから肉眼では見えないんじゃないですかね
0752ご冗談でしょう?名無しさん
垢版 |
2019/11/29(金) 20:16:11.62ID:i97N4ob+
>>709
どこでどう躓いているのかがわかれば、
「治療法」も見つかるかもしれません。
そうしたら、これ以上荒らされなくなりますよ。
0753ご冗談でしょう?名無しさん
垢版 |
2019/11/29(金) 20:39:18.54ID:???
>>752
一般的に扱うと、電磁場の近接作用の理論は理解するのが難しいので
個別の近似モデルを使う。逆に近似モデルを一般化することはできない。

例えば、電気抵抗のモデルでは抵抗棒に電流を流すと内部でジュール熱が発生するが
電磁場(エネルギー)の理論によれば、抵抗棒内に発生する電場により外部の電磁場の
エネルギーが抵抗棒内部に吸収されて熱が発生し続ける。
0755ご冗談でしょう?名無しさん
垢版 |
2019/11/29(金) 21:23:00.05ID:i97N4ob+
>>753
よろしければ、お薦めの電磁気学の教科書など、
二三、ご紹介頂けないでしょうか?あなたが
最も信頼を置く先生とか。
0756ご冗談でしょう?名無しさん
垢版 |
2019/11/29(金) 21:27:48.71ID:i97N4ob+
>>749
金鳥サッサこんなことしてたのかw
なんだか、「柏木達彦の多忙な夏」とか「数学ガール」の真似か!?
と思ったら、その通りだと本人が白状しててわろた。
0757ご冗談でしょう?名無しさん
垢版 |
2019/11/29(金) 21:34:17.58ID:0r9cXNtp
>>756
おもろいよね。
扱ってる問題も。
0758ご冗談でしょう?名無しさん
垢版 |
2019/11/29(金) 21:43:55.94ID:i97N4ob+
>>757
Lieb-Yngvason の最近の論文、お読みになりました?
0759ご冗談でしょう?名無しさん
垢版 |
2019/11/29(金) 21:51:54.97ID:???
>>754
バカの上塗りしたいのか
太陽が太陽系の重心運動の周期が10年以上もあり、太陽の自転周期約10時間に
比べても無視できる意味の ”恒星”だ。
0760ご冗談でしょう?名無しさん
垢版 |
2019/11/29(金) 22:02:48.22ID:i97N4ob+
>>749
ちょっと待ってください。下から3行目辺り、
ボルツマン分布から始めると、熱接触しても熱は移動しないって、
20度の水が、80度の水と接触しても…って意味じゃありませんよね?
0761ご冗談でしょう?名無しさん
垢版 |
2019/11/29(金) 22:08:47.95ID:???
恒星系の共通重心が外にあるなんて、連星ではざらにあること。

そもそも「恒星」という名称は歴史的経緯で使われているだけであり、
もはや「恒なる星」が定義ではないだろ。空間的な意味でも時間的な意味でも。
何が『「恒」星が聞いてあきれる』なんだか。こっちが呆れるわ
0762ご冗談でしょう?名無しさん
垢版 |
2019/11/29(金) 22:11:20.16ID:0r9cXNtp
>>760
そういう意味です。そういうおかしな結果になってしまう。温度の異なるボルツマン分布かを仮定した系をくっつけると。
0763ご冗談でしょう?名無しさん
垢版 |
2019/11/29(金) 22:14:41.81ID:0r9cXNtp
木曜日 昼の式42です。
0764ご冗談でしょう?名無しさん
垢版 |
2019/11/29(金) 22:17:54.31ID:i97N4ob+
>>732
そんな細菌、うっかり大量に畑に撒かれたりしたら…
0765ご冗談でしょう?名無しさん
垢版 |
2019/11/29(金) 22:31:49.00ID:i97N4ob+
>>753
「電磁場の近接作用の理論は理解するのが難しい」って、どういう意味で仰ってます?
皆さん理解して使ってますよ?
さらに、「難しいから、近似モデルを使う」って、
前後の繋がりが、さっぱり意味不明なんですが?
0766ご冗談でしょう?名無しさん
垢版 |
2019/11/29(金) 22:34:55.62ID:???
っていうか
電磁気も一般相対論も
電磁力や重力の自己結合の取り扱いがどうなってるのか未だに全然わからん

電磁気ですらわからんのに時空が歪む一般相対論なんてわかるはずもなく
0767ご冗談でしょう?名無しさん
垢版 |
2019/11/29(金) 22:42:58.76ID:T8lNB/51
上のpdfチラッてみたんですけど、結局ぬるま湯がお湯と水に勝手に分かれる現象も可能性はほぼ0だけどあるってことでいいんですよね?
0768ご冗談でしょう?名無しさん
垢版 |
2019/11/29(金) 22:47:42.42ID:0SjT3pjt
>>767
そういうことだと思います。
0769ご冗談でしょう?名無しさん
垢版 |
2019/11/29(金) 22:53:52.10ID:0SjT3pjt
ただしボルツマン分布というかカノニカル分布からだとそもそもエネルギーは移動しないという結果になる。時間発展含めて熱力学と整合するアンサンブルを考えてる。
0770ご冗談でしょう?名無しさん
垢版 |
2019/11/29(金) 23:33:49.26ID:i97N4ob+
>>766
もしかしてそれ「今まで一度もマトモに勉強したことない」
って状態であると、普通は言いませんか?
0771ご冗談でしょう?名無しさん
垢版 |
2019/11/29(金) 23:35:34.70ID:T8lNB/51
私もよくわかってないので教えていただきたいですね
0772ご冗談でしょう?名無しさん
垢版 |
2019/11/29(金) 23:38:40.32ID:i97N4ob+
>>767
それ、大昔からある、ごくごくごくフツーの統計物理の主張と、
どこがどう違うの?一言で言うと、ナニを今さら…。
0773ご冗談でしょう?名無しさん
垢版 |
2019/11/29(金) 23:41:03.48ID:T8lNB/51
>>772
ある無矛盾な公理系τの任意のモデルに対してある論理式φが常に真となるならば、τからφがLKにおいて証明可能となることを示せ

ということですね
0774ご冗談でしょう?名無しさん
垢版 |
2019/11/29(金) 23:43:40.90ID:0SjT3pjt
>>772
767はこのpdfが当たり前のことを否定しようとしてないことを確認するために聞いただけだと思いますよ。
なんせエネルギーが移動しないという変な結果が途中で出てくるので。
0775ご冗談でしょう?名無しさん
垢版 |
2019/11/29(金) 23:46:02.11ID:0SjT3pjt
まあ読んでみてください。自分はかなり面白かったし疑問が、完全にではないにしても解けた気がする。
0776ご冗談でしょう?名無しさん
垢版 |
2019/11/30(土) 01:01:02.59ID:8bm3uBbo
佐々先生!大変な誤りを発見致しました!

「片方の解しか見えないない」わろたw
0777ご冗談でしょう?名無しさん
垢版 |
2019/11/30(土) 01:04:26.74ID:8bm3uBbo
ソラの

 「熱い湯に水を足すとぬるま湯になるのも
  量子力学の結果なんですよね?」

には違和感あり。
0778ご冗談でしょう?名無しさん
垢版 |
2019/11/30(土) 01:26:51.10ID:8bm3uBbo
>>773
うわあああ、難しいよお!
今の物理って、こんなんなっちゃってるのか。ヴィトゲンシュタインかよ!
語り得ない問題には沈黙するわ。

>>774
ただの確認作業でしたか、それは失礼。
「否定しようとしてない」けど、「変な結果」は出て来ちゃうんですね。
0779ご冗談でしょう?名無しさん
垢版 |
2019/11/30(土) 01:35:17.51ID:/+Bidv8u
>>778
変な結果なんだけどよく考えたら当然なんだとわかる。カノニカル分布は熱力学(つまり平衡状態)と整合するアンサンブル。平衡状態なんだからカノニカル分布同士接触しても例え温度な差があっても各々は平衡状態なんだからエネルギーは移動しない。
ということだろうと。
0781ご冗談でしょう?名無しさん
垢版 |
2019/11/30(土) 06:16:24.06ID:???
準静的過程って実際の物理学的操作なの?
実現に誇張表現抜きに無限の時間が必要だから宇宙が滅んでも準静的過程は実現できないと思うんだけど

なんか田崎がエルゴード性を否定した時のロジックで話せば準静的過程も否定できる気がするんだけど
0782ご冗談でしょう?名無しさん
垢版 |
2019/11/30(土) 06:49:00.12ID:/+Bidv8u
ポイントは
(理想気体に限らずですが)平衡状態の熱力学は時間空間で変化のないマクロな状態を語っている。

一方でマクロな状態もミクロな状態に還元される。ミクロでは力学の法則に従っていて時間の可逆性がある。

このミクロとマクロの間がどうつながっているのか?という話しです。

平衡への緩和という実際の現象を熱力学とも力学とも整合するように統計力学でちゃんと記述するのは難しいという話しです。
0783ご冗談でしょう?名無しさん
垢版 |
2019/11/30(土) 06:57:14.10ID:/+Bidv8u
典型性に基づいて統計力学の基礎づけにする話はとても興味深いのですがまだ十分に検討されてないような気がしてますがどうなんでしょうか?
以前研究会か特別講義かなんかで、平衡への緩和時間の試算をしてるのを見たけど、現実より極端に短い時間(msecとかμsecとか)になり
うまくないね、とかやってたのを覚えてるが、
そのあとどうなったのかな?
0784ご冗談でしょう?名無しさん
垢版 |
2019/11/30(土) 07:07:47.50ID:/+Bidv8u
>>781
例えばお風呂のお湯が何時間もかかって室温に戻るという過程は温度がどう下がっていくかを温度計で測れるしその時その時は熱力学の法則が成り立っているから準静的過程と見ていいんじゃないかな。もちろん近似的ではあるだろうが。
0786ご冗談でしょう?名無しさん
垢版 |
2019/11/30(土) 07:19:41.21ID:/+Bidv8u
>>785
うそお。ならないと思うけど。コンプレッサーを室外に置いてるの?クーラーみたいに。
0788ご冗談でしょう?名無しさん
垢版 |
2019/11/30(土) 09:13:02.60ID:p7xzniDk
お財布が凍るからやめとけ
0789ご冗談でしょう?名無しさん
垢版 |
2019/11/30(土) 09:47:47.54ID:/+Bidv8u
>>787
あったまるんじゃないですか。
0791ご冗談でしょう?名無しさん
垢版 |
2019/11/30(土) 12:17:36.80ID:???
>>785
ほぼ断熱された部屋が用意できなければ科学的な再現性がある物理実験ができない。
できなければ、熱力学による断熱された部屋の思考実験をすればよい。(電力は外部供給)
馬鹿でなければ、熱力学の法則から結果は誰でも判る。
0792ご冗談でしょう?名無しさん
垢版 |
2019/11/30(土) 12:40:39.08ID:kM2j9JNm
>>791
その結果を聞いてるんだと思いますが。
0793ご冗談でしょう?名無しさん
垢版 |
2019/11/30(土) 12:42:48.06ID:p7xzniDk
電気が熱を吸収して輸送することはないのかな?
0794ご冗談でしょう?名無しさん
垢版 |
2019/11/30(土) 12:44:47.60ID:kM2j9JNm
>>793
あるでしょう。
0795ご冗談でしょう?名無しさん
垢版 |
2019/11/30(土) 12:58:35.36ID:???
>>742
分からんから天動説だったのさ

>>754
微小項を無視するセンスがないと科学はできない

>>777
量子力学の結果でないものなんかないから
今更だな

>>784
準静的は可逆的の意味しかないから
速くても遅くても意味はない

>>793
ペルチェ効果だろ
0796ご冗談でしょう?名無しさん
垢版 |
2019/11/30(土) 13:01:04.25ID:???
冷蔵庫の熱ポンプモーターは外部から電力供給で動いている。
積分された供給電力は必ず正になる。
0797ご冗談でしょう?名無しさん
垢版 |
2019/11/30(土) 13:16:24.52ID:???
>>796
断熱の部屋に外部からwtの仕事により熱エネルギーが増加する
熱ポンプを止め静的に熱平衡状態になれば部屋の平均温度は上がる。
0798ご冗談でしょう?名無しさん
垢版 |
2019/11/30(土) 13:33:06.35ID:Ukrk/fLv
いやでもやっぱ少しは下がるんじゃないですか?

水とお湯の話と一緒だと思うんですけど

冷蔵庫と部屋は熱平衡ではないですよね
0799ご冗談でしょう?名無しさん
垢版 |
2019/11/30(土) 13:37:52.47ID:Ukrk/fLv
少し下がるけど時間が経ったらまた上がるんですかね
0800ご冗談でしょう?名無しさん
垢版 |
2019/11/30(土) 13:39:26.23ID:/+Bidv8u
>>798
いろいろ細かい話が出てるけど、そもそも冷蔵庫は庫内の熱を奪って後ろからか庫外に捨てているんです。クーラーのように室外に捨ててるわけじゃないので開けっ放しにしたら室内の熱を室内で移動しているだけのことにしかなりません。熱ポンプの仕事や効率を考えなくても。
0801ご冗談でしょう?名無しさん
垢版 |
2019/11/30(土) 13:40:55.16ID:Ukrk/fLv
それはそうですけど、冷蔵庫内部と部屋の中にはたしかに温度差ありますよ

少しは下がると思います
実際、冷蔵庫開けると冷たいですよ
0802ご冗談でしょう?名無しさん
垢版 |
2019/11/30(土) 13:42:38.48ID:Ukrk/fLv
少なくとも、冷蔵庫のスイッチ落として放置したら多分少しは下がりますよね
0803ご冗談でしょう?名無しさん
垢版 |
2019/11/30(土) 13:42:41.13ID:/+Bidv8u
ああ。室内の温度の高いところから庫内の温度の低いとこに熱が移動するから少し下がるのでは?と言ってるんですね。
それはあるね。
0804ご冗談でしょう?名無しさん
垢版 |
2019/11/30(土) 13:47:12.43ID:/+Bidv8u
整理すると
冷蔵庫を開けっ放しにすると庫内の温度分布が均一になる過程で温度が下がる部分もあるが最終的には熱ポンプの発生する熱で温度は上がる。かな。
0805ご冗談でしょう?名無しさん
垢版 |
2019/11/30(土) 13:48:06.88ID:Ukrk/fLv
開けっ放しにしなかった時と比べたら下がると思います
0806ご冗談でしょう?名無しさん
垢版 |
2019/11/30(土) 13:50:47.54ID:Ukrk/fLv
まなんにせよ、冷蔵庫の中みたいにヒェッヒエになるってことはないでしょうってことですね
0807ご冗談でしょう?名無しさん
垢版 |
2019/11/30(土) 13:50:53.13ID:p7xzniDk
やはり、熱は、電気で運んで冷やしてるのかな?
0808ご冗談でしょう?名無しさん
垢版 |
2019/11/30(土) 13:52:00.27ID:???
>>801
キミは”熱ポンプ”の意味を理解していない。
簡単に言えば熱(エネルギーを)移動させるメカだ、冷蔵庫の中の熱を外壁に移動させてるだけだが
熱力学では、熱ポンプを動かすには、外部からの仕事が必要で、その仕事も熱に変わってしまう。

熱力学を認めない、自分の感覚だけ信じるというなら物理はやめた方がいい。
0809ご冗談でしょう?名無しさん
垢版 |
2019/11/30(土) 13:54:34.73ID:Ukrk/fLv
>>808
冷蔵庫開けると部屋の中と幾分か温度が平均化されて温度が多少下がります
仮に同じ温度になったとすると、ポンプの発生する熱によりまただんだん上がります

温度の平均化の分は下がりますよね、と言ってるんです
0810ご冗談でしょう?名無しさん
垢版 |
2019/11/30(土) 14:04:41.13ID:???
>>807
>>797 で部屋の温度が上がるという意味は、断熱部屋の中で
冷蔵庫(熱ポンプ)を動かす前の部屋の平均温度と
冷蔵庫(熱ポンプ)を或る時間動かして止めた、後の部屋の平均温度
の差のことだ。
0811ご冗談でしょう?名無しさん
垢版 |
2019/11/30(土) 14:08:09.35ID:Ukrk/fLv
それ冷蔵庫買ったばっかりの状況ってことですか?

今考えたいのと違うと思いますけど

そういうことなら、すでにポンプを動かして冷蔵庫と部屋の中で温度差すでにある状態でポンプ切ったらどうなるかを考えないとダメだと思いますけど
0812ご冗談でしょう?名無しさん
垢版 |
2019/11/30(土) 14:16:11.71ID:???
>>811
物理(学)で扱えるのは再現可能な実験ということだ
自分勝手にコロコロ変えれるなら、他人には理解不能だから物理はやめたほうが良い。
0813ご冗談でしょう?名無しさん
垢版 |
2019/11/30(土) 14:17:07.80ID:Ukrk/fLv
今考えたいのは冷蔵庫開けっ放しにしたらどうなりますかーってお話ですね

再現可能だと思います

冷蔵庫開けっ放しにしてポンプ切ったらどうなるかも再現可能です
0815ご冗談でしょう?名無しさん
垢版 |
2019/11/30(土) 14:35:19.05ID:Ukrk/fLv
私は質問者じゃないですけど、違うと思うんですけど
質問者が聞きたいのは冷蔵庫購入当時の気温ではなく、今の温度ですよ?


関係ないですけど冷蔵庫って、開けっ放しだと冷やそうと頑張ってポンプフル稼働するみたいですね
思ったより電気代とか温度上がるのかもしれません
0816ご冗談でしょう?名無しさん
垢版 |
2019/11/30(土) 14:42:55.43ID:???
>>811
>それ冷蔵庫買ったばっかりの状況ってことですか?
笑うしかないが、冷蔵庫買ったあとはコンセント入れっぱなしだからそう考えるのかな

開いた冷蔵庫の熱ポンプのON/OFF動作を完全に止めたければ、コンセントを抜くしかない。
0817ご冗談でしょう?名無しさん
垢版 |
2019/11/30(土) 14:58:01.72ID:/+Bidv8u
だいたい同じこと言ってると思うけど。
前提の置き方が違うだけで。
0818ご冗談でしょう?名無しさん
垢版 |
2019/11/30(土) 14:58:32.07ID:???
>>816
>>810 の環境にしたければ、扉の開いた冷蔵庫でコンセントが抜かれた状態で
ほぼ断熱部屋の室温(できれば電力量計も)を測り、コンセント入れて熱ポンプ運転(on/off)
させ一定時間経過した後、コンセントを抜き、十分空気が混ざった状態で温度(電力量計)で比較すればよい。
ついでに電力料金も調べられるだろ。
0819ご冗談でしょう?名無しさん
垢版 |
2019/11/30(土) 15:13:28.33ID:kM2j9JNm
冷蔵庫を開ける前より開けっ放しにした後の方がポンプの発熱量は増えるんだっけ?
開ける前はただポンプの発生する熱が室内に捨てられるだけだったのが、開けっ放しにしたら
室内での熱移動になるから下がるような気がしてきた。つまり室内に放出される熱は、

開ける前:ポンプの発熱+冷蔵庫内から奪った熱
開けた後:ポンプの発熱
なんで開けた後の方が室内に放出される熱は減らない?
0820ご冗談でしょう?名無しさん
垢版 |
2019/11/30(土) 15:17:13.69ID:???
断熱部屋の中が条件でないなら、冷蔵庫の放熱面を部屋の外部に接触させ、扉を内側に
開けば、クーラーと同じ原理で部屋内部が冷えるのは誰でも解かる。
0821ご冗談でしょう?名無しさん
垢版 |
2019/11/30(土) 15:25:22.23ID:/+Bidv8u
>>820
断熱部屋のなかが条件です。
冷蔵庫の扉を内側放熱面を外部に接触させません。
0822ご冗談でしょう?名無しさん
垢版 |
2019/11/30(土) 15:34:15.45ID:kM2j9JNm
820は819に言ってる?断熱部屋の前提です。
冷蔵庫開ける前も後も奪った熱を室内に放熱してるのは同じ、ポンプが発熱してるのも同じ。熱を奪う対象が冷蔵庫内だけだったのが部屋全体になるから下がる。
0823ご冗談でしょう?名無しさん
垢版 |
2019/11/30(土) 15:37:04.22ID:QL3EzH23
結局、上がるか下がるかはわからないんじゃないでしょうか

冷蔵庫の設計上の問題も絡みそうですしね
開けっ放しにするとフル稼働するんですから
0824ご冗談でしょう?名無しさん
垢版 |
2019/11/30(土) 15:37:45.87ID:kM2j9JNm
冷蔵庫開ける前と後の比較でね。
部屋が断熱なら時間とともに温度は上がっていくのは冷蔵庫開ける前も開けっ放しにした後も同じ。冷蔵庫全体では発熱してるから。
0825ご冗談でしょう?名無しさん
垢版 |
2019/11/30(土) 15:42:55.00ID:???
>>821
理想的な熱ポンプは熱を移動させるだけだから、冷蔵庫の放熱面で空気温度が上がり
冷蔵庫の内部では空気温度が下がる。
扉を開けっ放しで、2つの空気を完全に混ぜれば、冷蔵庫を起動す前の温度と同じになる。
(第一法則)ここまで解かれば
現実の熱ポンプは外部からの仕事(電気エネルギー)が必要になる(第二法則)
その電気エネルギーは熱に変わり、部屋の中の熱エネルギーが増える、つまり温度が上がる。
0826ご冗談でしょう?名無しさん
垢版 |
2019/11/30(土) 15:43:56.83ID:???
冷蔵庫は,庫内の温度を設定温度に下げようと動作し,電力を消費し続ける。
部屋の温度がどうなるかは自明だろ。
馬鹿なの?
0827ご冗談でしょう?名無しさん
垢版 |
2019/11/30(土) 15:48:58.47ID:QL3EzH23
一旦下がるけどまた上がる
どこまで上がるかはわからない

でも多分上がるってことですかね
0828ご冗談でしょう?名無しさん
垢版 |
2019/11/30(土) 15:49:26.94ID:/+Bidv8u
>>825
現実の熱ポンプは外部からの仕事(電気エネルギー)が必要になる(第二法則)
その電気エネルギーは熱に変わり、部屋の中の熱エネルギーが増える、つまり温度が上がる。

↑は冷蔵庫を開ける前も開けっ放しにしたあとも同じですよね。冷蔵庫を開ける前はこの熱は発生しないのですか?
0829ご冗談でしょう?名無しさん
垢版 |
2019/11/30(土) 15:50:30.17ID:QL3EzH23
閉めてても部屋の温度は上がり続けてるってことですかね
0830ご冗談でしょう?名無しさん
垢版 |
2019/11/30(土) 15:50:41.53ID:/+Bidv8u
>>826
馬鹿なのでいろいろ悩んでます。
0831ご冗談でしょう?名無しさん
垢版 |
2019/11/30(土) 15:53:25.83ID:/+Bidv8u
>>829
断熱部屋ならそうだと思います。
でその上がり方は、開閉によらず、ポンプはフル稼動する前提なら、開けた方が小さいのではないかなと思えてきた。
0833ご冗談でしょう?名無しさん
垢版 |
2019/11/30(土) 16:01:37.57ID:???
>>831
馬鹿にする訳ではないのですが
物理的な思考がほんとにできないんですね
ちゃんとした本をよく考えながら読むことで訓練するしかないです
0834ご冗談でしょう?名無しさん
垢版 |
2019/11/30(土) 16:02:27.66ID:/+Bidv8u
断熱部屋を二つ用意する。部屋同士は断熱壁で仕切られている。片方にクーラーもう片方に室外機を置いてフル稼動させる。クーラーのある部屋が冷蔵庫内だと考えて
室外機のある部屋の温度を考えれば問題設定としては同じだよね。
で二つの部屋をつなぐ扉を開けるのと閉めるのとどちらが、室外機のある部屋の温度が下がるか?扉を開けた方だと思わない?
0835ご冗談でしょう?名無しさん
垢版 |
2019/11/30(土) 16:03:59.85ID:Ukrk/fLv
開けてから最初の方は冷気がもれて少し下がるかもしれません
フル稼働して熱が出てもその分と中和されてどっちの影響が大きいかはわかりません

冷蔵庫内部と部屋の温度の中和が終わったら、その後もフル稼働しているので、温度上昇の影響は、開け放った直後に比べて大きくなるんですかね
0836ご冗談でしょう?名無しさん
垢版 |
2019/11/30(土) 16:05:45.25ID:/+Bidv8u
>>833
物理的な思考で誤りを指摘して考えを正して下さるとありがたいです。
0837ご冗談でしょう?名無しさん
垢版 |
2019/11/30(土) 16:05:59.82ID:Ukrk/fLv
>>834
開けた場合と、開けなかった場合を比較するわけですね

開けた場合の方が下がるでしょうね
0838ご冗談でしょう?名無しさん
垢版 |
2019/11/30(土) 16:07:33.47ID:/+Bidv8u
>>837
開けた場合と開けない場合の比較ですね。
0839ご冗談でしょう?名無しさん
垢版 |
2019/11/30(土) 16:10:02.44ID:/+Bidv8u
ただ現実の冷蔵庫では稼動するしないの制御が入るのでその仕様次第。かなと。
0841ご冗談でしょう?名無しさん
垢版 |
2019/11/30(土) 17:01:22.79ID:/+Bidv8u
そうなの?
0842ご冗談でしょう?名無しさん
垢版 |
2019/11/30(土) 17:02:23.95ID:/+Bidv8u
じゃあコンプレッサーの発熱がなくなり開けるより温度下がるね。
0844ご冗談でしょう?名無しさん
垢版 |
2019/11/30(土) 17:18:41.49ID:qUng/Nx7
何と何を比較したいんだ?

温度差のある2室間の戸を開放して、
コンプレーサーを停止又はコンプレッサーによる熱移動以外の発熱が無視できる短時間で、2室の空気を混合すれば、
温度が、開放前の高温側の温度より下がることは考えるまでもないだろ


> じゃあコンプレッサーの発熱がなくなり開けるより温度下がるね。
どう考えればこうなるんだ
0845ご冗談でしょう?名無しさん
垢版 |
2019/11/30(土) 17:24:04.88ID:/+Bidv8u
失礼。開ける前より、の誤記です。
0846ご冗談でしょう?名無しさん
垢版 |
2019/11/30(土) 17:39:04.92ID:???
旧ヤマトの波動エンジンは
超音速のタキオン粒子を断熱圧縮して空間を歪ませ
その歪みの増減の波にのって船体が推進するという方式だったようだ
無茶苦茶空間を歪ませるのがワープ
断熱圧縮して膨張したタキオン粒子をタービンにあててタービンを回転させ、
そのタービン軸出力で圧縮機を駆動する
余剰のタキオン粒子は後部ノズルから噴射もしてこれも推進力にしている

2199、正確には2202以降のヤマトの波動エンジンに使われているのはM理論。
ひも理論でこの世は11次元でできているが、4次元以外の残りの5次元の「余剰次元」が隠れている。
その余剰次元を展開して膨大なポテンシャルエネルギーを取り出すのが波動エンジン
余剰次元を波動砲口から射線上に展開するのが波動砲
リメイク版ヤマト2202の1話のセリフにアンドロメダの波動砲発射に対して
「余剰次元の爆縮を確認」というのがあり、この設定を反映している
0847ご冗談でしょう?名無しさん
垢版 |
2019/11/30(土) 18:14:48.86ID:BxfhWb2d
吉田春夫著『キーポイント力学』を読んでいます。

r(t)
θ(t)

を t の関数とします。

吉田さんは、 r を θ で微分しています。

これは許されるのでしょうか?

θ(t0) = θ(t1) ではあるが、 r(t0) ≠ r(t1) となるときにはどうなるのでしょうか?
0848ご冗談でしょう?名無しさん
垢版 |
2019/11/30(土) 18:17:36.28ID:Ukrk/fLv
よくわかりませんけど、普通に媒介変数の微分なんじゃないですか?
0849ご冗談でしょう?名無しさん
垢版 |
2019/11/30(土) 18:18:53.34ID:BxfhWb2d
r が θ の関数にならないときにはどうするのでしょうか?
0850ご冗談でしょう?名無しさん
垢版 |
2019/11/30(土) 18:21:00.35ID:BxfhWb2d
p.120に

「動径 r を角度 θ の関数として表すために、 θ を独立変数とする微分方程式を導こう。」

などと書いています。

ポイント6の「惑星はなぜ楕円軌道を描くか」というチャプターです。
0851ご冗談でしょう?名無しさん
垢版 |
2019/11/30(土) 18:23:22.73ID:Ukrk/fLv
物理的に考えたらそんな場合ないですよね

数学的にはヤコビアン計算して云々すりゃなんかでるんじゃないですか?
陰関数定理とかで
0852ご冗談でしょう?名無しさん
垢版 |
2019/11/30(土) 18:38:15.36ID:BxfhWb2d
>>851

>物理的に考えたらそんな場合ないですよね

↑なぜでしょうか?
0854ご冗談でしょう?名無しさん
垢版 |
2019/11/30(土) 18:42:17.37ID:???
惑星の楕円軌道を論じるというコンテクストを理解できないバカだから
一般の場合を包含してないとかわめき始める
0855ご冗談でしょう?名無しさん
垢版 |
2019/11/30(土) 18:50:13.89ID:Ukrk/fLv
てか難しいこと考えなくても、普通に角運動量保存則とかからでるんじゃなかったでしたっけ
rとθの関係式
0856ご冗談でしょう?名無しさん
垢版 |
2019/11/30(土) 18:57:56.23ID:T0v7Klek
>>778
幸子、それヴィトゲンシュタインちゃう、ゲーデルや!
0857ご冗談でしょう?名無しさん
垢版 |
2019/11/30(土) 19:11:02.54ID:BxfhWb2d
例えば、

恒星の周りを公転している惑星がだんだん恒星に近づいていくような場合にはどうするのでしょうか?
0858ご冗談でしょう?名無しさん
垢版 |
2019/11/30(土) 19:13:19.39ID:/+Bidv8u
バカだなんだと憎まれ口叩かずにいられないのがいるようだけど、みんな大人の対応でスルーしてるけど、おさとが知れるというか、人としてのレベルというか、底が見えてしまうから、やめた方がいいと思うよ。
0859ご冗談でしょう?名無しさん
垢版 |
2019/11/30(土) 19:29:15.69ID:BxfhWb2d
あ、 0 ≦ θ ≦ 2*π じゃなくてもいいですね。
0860ご冗談でしょう?名無しさん
垢版 |
2019/11/30(土) 19:29:58.11ID:Ukrk/fLv
>>857
こういう運動ってそもそもあるんでしたっけ?
0861ご冗談でしょう?名無しさん
垢版 |
2019/11/30(土) 19:33:53.37ID:BxfhWb2d
惑星の初速度が小さい場合には、恒星に近づいて行ってぶつかってしまうのではないでしょうか?
0863ご冗談でしょう?名無しさん
垢版 |
2019/11/30(土) 19:54:35.71ID:T0v7Klek
>>795
「量子力学の結果でないものなんかない」にも、やはり同様に違和感を覚えます。

 物理学のハナシしてるんですから、こういう場合、古典論との対比
(量子力学を持ち出さないと説明がつかないか)で語るのが普通
なのではありませんか?
0866ご冗談でしょう?名無しさん
垢版 |
2019/11/30(土) 20:27:32.63ID:T0v7Klek
>>840
たった今、わが家の冷蔵庫の扉開けましたか、コンプレッサー動いたまんま。





ドア開けてストップするのは、中の「扇風機」や。
空気掻き回すための。
0868ご冗談でしょう?名無しさん
垢版 |
2019/11/30(土) 20:48:20.18ID:T0v7Klek
>>864
ところが物理学で「勝利」し続けてきたのが還元主義だ
というのも間違いのない事実でありーの。
0870ご冗談でしょう?名無しさん
垢版 |
2019/11/30(土) 20:56:31.36ID:T0v7Klek
>>867
そんな実験などする必要はない。コンプレッサーは、庫内温度が設定温度に達すると停止、
設定温度より上昇したら動き出す。
0872ご冗談でしょう?名無しさん
垢版 |
2019/11/30(土) 21:22:36.99ID:T0v7Klek
>>863
   「熱い湯に水を足すと、ぬるま湯になる」のも量子力学の結果 

って聞くと、

 だったら、2原子分子気体の原子間振動も、ネグらずに、ちゃんと考慮に入れてやれよ、
自由度変わっちゃうけどな。実験とまったく合わない計算結果出てくるし…

…あ、量子効果考慮に入れてネグるんだから、これは不適切な例や!www
やっぱ「量子力学の結果でないものはない」のか?そーなのか!?そーゆー意味なのか?
0873ご冗談でしょう?名無しさん
垢版 |
2019/11/30(土) 21:29:17.99ID:p7xzniDk
こいつがアホなのも量子力学かな。
0874ご冗談でしょう?名無しさん
垢版 |
2019/11/30(土) 21:33:49.64ID:T0v7Klek
>>871
必要ない。サーモスタットの動作なんだから。そうなるように作ってあるんだから、
完全に予測できる。ドア開け続けたら、庫内温度上昇するから、コンプレッサ動き続ける。
ラジエータに霜がついたら、霜取りのため一時停止するけど。
0876ご冗談でしょう?名無しさん
垢版 |
2019/11/30(土) 21:49:54.53ID:T0v7Klek
>>806
ヒエっヒエどころか、熱っちんちんになるよお。
0877ご冗談でしょう?名無しさん
垢版 |
2019/11/30(土) 22:18:31.83ID:T0v7Klek
>>872
あれ?「量子効果」っちゅーよか、縮退してて高温にならんと効いてこないから、
気にせんでええ、だったっけか?よお思い出せんわ。量子力学適用せんとうまい
説明でけんちゅうのんは、確かやけど。
0878ご冗談でしょう?名無しさん
垢版 |
2019/11/30(土) 22:31:57.62ID:T0v7Klek
>>874
あのさあ、「準静的」ってさ、なんかモヤモヤーっとした曖昧模糊とした
イメージだけの表現でなく、もっと物理的なちゃんとした定義あるよ。
プリゴジーヌとか。もお今ドキは、時代遅れで流行ってないの?
0880ご冗談でしょう?名無しさん
垢版 |
2019/12/01(日) 05:10:39.90ID:???
質問です。
2017年京都府立医大物理大問1の問6についてなのですが、解説を読むと、板Aとおもりの重心の加速度がx軸正の向きにgsinθとなるそうです。

重心の運動方程式は、Rgを重心のベクトル、NをMにはたらく垂直抗力として
(m+M)×(d^2Rg)/(dt^2)=Mg+mg+N
だと考えたのですが、ここからでは加速度gsinθを導けません。どのように考えたらよいでしょうか。
https://i.imgur.com/iJfiGpl.gif
https://i.imgur.com/H6NDBBV.gif
0884ご冗談でしょう?名無しさん
垢版 |
2019/12/01(日) 11:18:04.26ID:1CvAEKe7
高校物理では、微分積分を使わない。それは不自然だという人がいます。

でも、教科書を読むと例えば、円運動とか単振動とかのところで、まさに微分の考え方を説明していますよね。

積分も面積として説明されていますよね。

微分方程式が登場しないのが不満なんですかね?
0887ご冗談でしょう?名無しさん
垢版 |
2019/12/01(日) 11:40:17.29ID:CxI1wuMN
>>884
本質的なことがなにもわかってない人たちなんですよ

微分方程式はma=Fの表示の仕方の一部でしかないのにですね
0889ご冗談でしょう?名無しさん
垢版 |
2019/12/01(日) 13:20:44.38ID:5HrQKlc6
高校でもそう習ってるはずですけど

記号を使うかどうかの問題です
その物理概念をどう数式に起こすかの問題です
0891ご冗談でしょう?名無しさん
垢版 |
2019/12/01(日) 13:42:09.46ID:5HrQKlc6
それも高校でやってるはずです
微分の記号はなくても、グラフの問題とかありましたよね
0892ご冗談でしょう?名無しさん
垢版 |
2019/12/01(日) 13:51:35.71ID:???
力学の力(定義とみる)方程式には f=ma とは別の方程式があるのは高校レベルでも習うが
気づかない人が多い。
0893ご冗談でしょう?名無しさん
垢版 |
2019/12/01(日) 14:00:18.48ID:CxI1wuMN
F=dp/dtってことですか?
0894ご冗談でしょう?名無しさん
垢版 |
2019/12/01(日) 14:38:49.89ID:Om8nOYZU
それは力積の定義でしょ
0895ご冗談でしょう?名無しさん
垢版 |
2019/12/01(日) 14:39:27.67ID:CxI1wuMN
相対論でも使える式ですよ
0897ご冗談でしょう?名無しさん
垢版 |
2019/12/01(日) 15:09:34.54ID:???
高校物理で微積微積言ってる人、飛び抜けてできる人とセンター8割位の人の二種類が多い気がする
0898ご冗談でしょう?名無しさん
垢版 |
2019/12/01(日) 16:54:09.68ID:???
>>893
>F=dp/dtってことですか?
それは基本的に f=ma と同じ式だが、それと対になる別の力(の定義とみる)方程式がある。
0899ご冗談でしょう?名無しさん
垢版 |
2019/12/01(日) 16:56:34.29ID:Om8nOYZU
p=mvが成り立たたないと同じとは言えないね
0900ご冗談でしょう?名無しさん
垢版 |
2019/12/01(日) 16:57:26.60ID:CxI1wuMN
なんのことですかそれって
0903ご冗談でしょう?名無しさん
垢版 |
2019/12/01(日) 20:01:38.02ID:???
>>901
正解ですね
何を言いたいかというと、相対性理論や量子論をかじってから再考してみれば
運動量の定義による力と、ポテンシャルによる力の定義があることが解かる。

ニュートン力学を調べれば判るが、運動方程式 F=d/dt^2(x) 力の基本定義から
演繹によって運動量、仕事によるポテンシャルを導出し、力学的エネルギー保存
も同様。
こんな上手く行き過ぎるのはニュートン力学(とユークリッド空間)だけで起こる
力のイリュージョンではないか?
実際に、力とは筋力感覚を抽象化した見掛けの物理量であり、理論的に必要でもない
物理的な相互作用は、ポテンシャル(エネルギー)の変化と運動量の変化で直接記述できるだろう。

f=ma 止めて、中高教育からハミルトン形式の様で理解し易い物理教科書は作れないのだろうか。
0905ご冗談でしょう?名無しさん
垢版 |
2019/12/01(日) 20:07:42.57ID:???
>>903
コンピューターゲームのミドルウェアに入ってる物理エンジンを設計するためのゲーム物理の教科書がそういうノリに近い。
0907ご冗談でしょう?名無しさん
垢版 |
2019/12/01(日) 20:17:22.16ID:Qzx93Wj8
ニュートン力学の力がポテンシャルの空間微分として書けない場合がありますね

拘束力とか非保存力とかいうやつですね

ハミルトン形式だとめんどくさい拘束条件つける必要ありますから、そういうわけわからないことするよりは何でもかんでも力としてしまうというニュートン力学に利はあるわけですね

一般相対性理論まで話を広げると、運動方程式というのはただの測地線の方程式となってしまって、力というのは結局クリストッフェル記号の時間成分だという結論になるわけですね
0908ご冗談でしょう?名無しさん
垢版 |
2019/12/01(日) 20:30:33.84ID:???
>>903
素粒子物理学者が、宇宙には4つの基本的な力があるとマスメディア向けに発表してるが
実際の実験で素粒子の「力」など観測している科学者などいない。
にもかかわらず
学校教育の試験問題も「力]ありきの問題ばかりで、現代物理に行くにはブレークスルー
が大変だ。
0910ご冗談でしょう?名無しさん
垢版 |
2019/12/01(日) 20:32:19.65ID:Qzx93Wj8
違いますよね(笑)

ニュートン力学、ラグランジュ力学、ハミルトン力学、それぞれ別物ですよ
0912ご冗談でしょう?名無しさん
垢版 |
2019/12/01(日) 20:42:55.43ID:Qzx93Wj8
てか

d/dt^2

こういうトンデモ数式書き込んで何も思わないような人が高校物理批判してるんでしょうね
習いたての知識を披露したくて仕方ないって感じなんでしょうか

一度直してもおかしいと思わないというのは、なんなんでしょうね
0914ご冗談でしょう?名無しさん
垢版 |
2019/12/01(日) 20:48:12.60ID:Qzx93Wj8
古典的な描像を用いるなら力ベクトルはありますよ

その描像で現実とつじつまがあわないのは、あくまで量子効果が無視できないからであって、力かポテンシャルかという話ではありませんよ
0916ご冗談でしょう?名無しさん
垢版 |
2019/12/01(日) 20:53:09.67ID:Qzx93Wj8
d/dt^2

こんなの見たことないですよー
0918ご冗談でしょう?名無しさん
垢版 |
2019/12/01(日) 21:04:52.13ID:???
>>915
そういう意味ではない
「力」は物理的な相互作用に必要な量か、無くても(複雑になるが)記述・解析できるなら
「力」は物理的に無いのと同じだろ。
0920ご冗談でしょう?名無しさん
垢版 |
2019/12/01(日) 21:11:59.48ID:Qzx93Wj8
量子論は力はなくてもというよりポテンシャルじゃないと量子化できませんよね

力はあってもなくても良いという話が通用するのは古典論です

古典論の話ならニュートン力学という力を全目に押し出す体系がありますし、力があると思った方が直感的に明らかだと思うんですけど
0921ご冗談でしょう?名無しさん
垢版 |
2019/12/01(日) 21:12:48.88ID:Qzx93Wj8
>>919
良いんじゃないですか

少なくとも高校物理は古典論のお話なんですから
0922ご冗談でしょう?名無しさん
垢版 |
2019/12/01(日) 21:25:27.92ID:???
>>919
つまり
現代的な物理では1階時間微分の別々の量を、ニュートン力学では2階微分の「力」一式で記述する。理論的に矛盾が無ければ、物理現象にどこまで有効かは問題ではない。
0923ご冗談でしょう?名無しさん
垢版 |
2019/12/01(日) 21:31:37.02ID:Qzx93Wj8
>>922
それもなんか気になる言い回しですよねー

解析力学は現代的なんですかって問題がまずありますよね

あれpとqに分けたりしてますけど、結局、xとdx/dt=vで書きましょう、てだけですからね

ニュートンの運動方程式
m*d^2x/dt^2=F



mdv/dt=F
v=dx/dt

はい、二変数になりました

これだけですよ結局

解析力学もニュートン力学も同じ古典論なんですから得られる結果は全く同じです

量子論の話ならまた別ですけどなんかそういう雰囲気でもなさそうですから一応書いておきますね
0925ご冗談でしょう?名無しさん
垢版 |
2019/12/01(日) 21:38:15.27ID:Qzx93Wj8
わからないんですね
0929ご冗談でしょう?名無しさん
垢版 |
2019/12/01(日) 21:47:41.80ID:Qzx93Wj8
>>927
はぁ?
意味わかりませんね(笑)
普通生準変数か何かのことなのかなとか思いませんか?

で、現代物理学だとそれが基本的な方程式になってるんですかね
よくわかりませんね

>>926
なんかBB2Cから書き込めなくなっちゃったんです
0931ご冗談でしょう?名無しさん
垢版 |
2019/12/01(日) 22:25:13.71ID:???
相間くっくっくは、運動量、エネルギー、保存の概念が 2階時間微分のf=ma だけから
導出できると死ぬまで信じ込んでる f=ma (と遠隔作用の作用反作用)の原理主義者といえる。
0932ご冗談でしょう?名無しさん
垢版 |
2019/12/01(日) 23:32:25.24ID:yedAEMRy
>>929
うちもだ。なんか規制かかってるな。
誰かスレッド立てるの頼めませんか?

 熱力学・統計力学・エントロピー・エルゴード

って感じのタイトルでーの。
0934ご冗談でしょう?名無しさん
垢版 |
2019/12/02(月) 01:36:45.58ID:3tsr6e3R
アマゾンの 白井光雲「現代の熱力学」共立 レビュー内で、白井氏が、

 「非平衡状態でもエントロピーは定義できる」「それはLieb-Yngvasonの論文に書いてある」
と、サポートページで述べられていた、なる記述があるのですが、当該ページを探しても見当たりません。

 ただし「Lieb-Yngvason」ではなく、「Gyftopoulos and Beretta」に言及したページなら
見つかりました。そのページの参考文献には「Lieb-Yngvason」も挙げられていましたが。

 www.cmp.sanken.osaka-u.ac.jp/~koun/therm/dsf/discuss-ch4.html
0935ご冗談でしょう?名無しさん
垢版 |
2019/12/02(月) 01:48:40.57ID:3tsr6e3R
>>934
何しろ2014年末のレビューなので、消えてしまっているのかも知れませんが、
どなたか白井氏が
「非平衡状態でもエントロピーは定義できる」「それはLieb-Yngvasonの論文にも書いてある」
と発言されたページをご覧になった方はおられませんか?

ちなみにLieb-Yngvasonには、'The entropy concept for non-equilibrium states'2013,2014

なる論文もあります。
0937ご冗談でしょう?名無しさん
垢版 |
2019/12/02(月) 15:37:56.40ID:???
>>933
>非保存力がある場合には明らかに Newton 形式が有利
それは間違いだ、例えば簡単な思考実験

垂直な2枚の平板に挟まった煉瓦が摩擦で落下する運動をF=maの力を使わないで求める。
接触面には運動と逆方向に発生する位置の摩擦エネルギー(ミクロでは電磁気的)が定義できる。
(重力エネルギー)−(摩擦エネルギー)= 物体の運動エネルギー
ニュートン力学の運動エネルギーでは (1/2)mv^2 から物体の運動が求まる。

ポイントは、エネルギーの式が f=ma が成り立たない相対性理論においても有効だということ。
現代の物理学を少しでも理解したいならば、「力を信じないことだ」。
0938ご冗談でしょう?名無しさん
垢版 |
2019/12/02(月) 15:40:49.05ID:RyUyTQna
摩擦エネルギーはどのようにかけますか?
0939ご冗談でしょう?名無しさん
垢版 |
2019/12/02(月) 15:42:55.95ID:RyUyTQna
あと相対論でもf=ma成り立ちますからね

fとaが相対論的になるだけで

相対論は古典論ですから力の描像が正確に成り立ちます
0942ご冗談でしょう?名無しさん
垢版 |
2019/12/02(月) 15:56:17.22ID:RyUyTQna
書いてみてください
0943ご冗談でしょう?名無しさん
垢版 |
2019/12/02(月) 16:02:27.36ID:RyUyTQna
ハミルトン形式で摩擦が書けるということですよね

非保存力の扱いって私もよく知らないので知りたいです
0944ご冗談でしょう?名無しさん
垢版 |
2019/12/02(月) 16:11:08.15ID:pUPnMWYv
ハイゼンベルグ方程式を立てればいい。解くのは、お前たちの仕事だ
0945ご冗談でしょう?名無しさん
垢版 |
2019/12/02(月) 16:12:01.18ID:RyUyTQna
ハミルトニアンが建てられるんですかって感じなんですけどまず
0954ご冗談でしょう?名無しさん
垢版 |
2019/12/02(月) 21:06:14.91ID:RyUyTQna
場合分けだのなんだのって始めるなら最初からニュートンの運動方程式でいいと思うんですよねー

めんどくさいものは全部力としてまとめてしまうわけですから、こっちの方が明らかにスッキリしてますよね
0956ご冗談でしょう?名無しさん
垢版 |
2019/12/02(月) 21:16:37.77ID:RyUyTQna
場合分け入れるラグランジアン作るくらいなら、普通に摩擦力そのものを扱った方が絶対いいですよね
0959ご冗談でしょう?名無しさん
垢版 |
2019/12/02(月) 21:51:14.84ID:RyUyTQna
>>951
どうでしょうも何も意味不明なんですけどー
0961ご冗談でしょう?名無しさん
垢版 |
2019/12/02(月) 22:19:34.02ID:RyUyTQna
運動方程式は満たすんですかねこれで
0962ご冗談でしょう?名無しさん
垢版 |
2019/12/02(月) 23:15:34.49ID:???
>>935
こんなの公開してる

http://www.cmp.sanken.osaka-u.ac.jp/~koun/therm/theory/theory.html

Lieb-Yngvasonについては,99の方も,
「それらを取り込むだけの能力は著者にはなかった」
だそうだから,非平衡系について書いてあったとしても
意味のあるような内容ではないだろう
0963ご冗談でしょう?名無しさん
垢版 |
2019/12/02(月) 23:16:31.09ID:???
そもそもラグランジアンどう立てるかが議論になる時点でニュートン形式の方が有利なのは明らかなんだよね
0966ご冗談でしょう?名無しさん
垢版 |
2019/12/03(火) 00:28:50.60ID:AUkVfS64
>>962
わざわざ調べて下さり、ありがとうございます。

実はアマゾンの書評内で、「beta」氏が、「白井氏が誤解している」旨のことを書いておられるのですが、誤解しているのはbeta氏ではないのか?と思っているのです。
0967ご冗談でしょう?名無しさん
垢版 |
2019/12/03(火) 00:32:42.32ID:oybJnBxg
>>951
やっぱり運動量の定義に時間が陽に依存するようになるのでなんかすごい気持ち悪いですね

そんな不自然なことしてまで摩擦を解析力学的に扱う必要性が見えないんですけど
0968ご冗談でしょう?名無しさん
垢版 |
2019/12/03(火) 01:10:27.42ID:???
ニュートンの運動方程式で解くほうが早いとか、新理論を作るとかの話ではない。

100年前に物理学が刷新されたというのに、日本の中高の物理教育現場では
明治維新の頃と変わらない様なニュートンの力学を刷り込み教育しているのが
問題なのだ。
 現代物理・科学技術の職種ではニュートン力学程度の人材では使い物にならないだろう。
始めから力ありきの物理を刷り込まれると普通の学生は、量子力学、相対性理論で
ブレークスルーができないと挫折する。(経験してる人は多いだろ)
今の日本は小子化で日本人大学生が減り続けてるから簡単に落ちこぼれては困るのだ。

相対論・量子力学に近似の運動学(実質的にニュートン力学)の学習にすれば良い
その為には、力ありきの f=ma がない様な物理テキストが必要になるということ。
0969ご冗談でしょう?名無しさん
垢版 |
2019/12/03(火) 01:29:57.80ID:oybJnBxg
摩擦力をポテンシャル的に扱うためだけに運動量を時間に陽に依存する定義にするのは馬鹿げていると思います
0971ご冗談でしょう?名無しさん
垢版 |
2019/12/03(火) 01:49:36.63ID:oybJnBxg
そうなんですか?
0972ご冗談でしょう?名無しさん
垢版 |
2019/12/03(火) 01:54:01.64ID:oybJnBxg
てか>>968も相対論が力ありきで話さないことってまずないと思うんですけど

相対論てむしろ力のローレンツ変換はどうなるかーとかって考えませんか?
ラグランジアン云々はオマケじゃないですか?
0973ご冗談でしょう?名無しさん
垢版 |
2019/12/03(火) 01:57:50.23ID:???
>>972
基本的にはポテンシャルありきですよね?
あとローレンツ変換云々を言うなら余計に解析力学でやりたいですよね?
相対論ならラグランジアンがローレンツ変換で不変(厳密には作用が不変)であるということが大前提として理論を組み立てられるからですね
0975ご冗談でしょう?名無しさん
垢版 |
2019/12/03(火) 02:35:22.92ID:Es3N4YJc
>>974
私もダウンロードしましたが、何しろ百ページ超を越えるばかりか、
中身が本当に難解で困ってます。あなたは理解されたようですから、
教えてください。Lieb-Yngvason'The Physics and Mathmatics of the Second Law of Thermodynamics'1997,1999 には、

     「非平衡状態でもエントロピーは定義できる」と書いてあるのですか?ないのですか?
0976ご冗談でしょう?名無しさん
垢版 |
2019/12/03(火) 03:09:01.16ID:i3g7gBwp
>>972
量子力学が生まれなかったらラグランジアンやハミルトニアンは理論の見通しを良くして
体系的に整理する単なるツールやオマケに過ぎなかったかも知れない。
我々が直接観測するのはラグランジアンではなく位置や運動量といった物理量だから。
なのでニュートン力学であれ相対論であれ運動方程式を立てられれば必要な情報は全て引き出せる。

でも量子力学ではハミルトニアンの形式(シュレディンガー方程式)が理論の基礎方程式になっていて、
逆に運動方程式というものはないから、こっちがオマケでなく主役になる。
0977ご冗談でしょう?名無しさん
垢版 |
2019/12/03(火) 03:16:38.68ID:LAkRXp0Y
>>975
白井氏の立場を確認。「▪エントロピーは非平衡状態でも定義できる
…一方、内部エネルギーUやエントロピーSは非平衡状態でも値が確定する量である。
非平衡状態でも定義できるUやSやらにより、非平衡状態を含めた一般的な状態間遷移
について記述したものが上記のGyftopoulos-Beretta'Thermodynamics-Foundations and Applications'
Dover 2005 である。見事な論理展開である。」

https://www.cmp.sanken.osaka-u.ac.jp/~koun/therm/dsf/discuss-ch4.html

より。


0979ご冗談でしょう?名無しさん
垢版 |
2019/12/03(火) 10:06:02.98ID:oybJnBxg
>>976
で、運動量の定義変えてまで摩擦を解析力学で扱うメリットはなんなんですか?
0981ご冗談でしょう?名無しさん
垢版 |
2019/12/03(火) 10:35:41.63ID:oybJnBxg
あなたの大好きな正準形式にしたいなら運動量必要ですよね
0983ご冗談でしょう?名無しさん
垢版 |
2019/12/03(火) 11:00:43.10ID:oybJnBxg
p=mvじゃなくなりますよ
0985ご冗談でしょう?名無しさん
垢版 |
2019/12/03(火) 11:47:07.28ID:oybJnBxg
そんなこと知ってますけど
0987ご冗談でしょう?名無しさん
垢版 |
2019/12/03(火) 15:55:04.23ID:yWOZ+8ra
不可視光の色の絵具を人間が見たら、どんな色に見えますか?
0990ご冗談でしょう?名無しさん
垢版 |
2019/12/03(火) 23:04:50.20ID:???
ラグランジアンもハミルトニアンもすぐに書き表せないのに、
ハミルトン形式のpをどうやって求めるんですかねえ
0993ご冗談でしょう?名無しさん
垢版 |
2019/12/04(水) 00:24:00.84ID:1EPRLLw4
>>991
「質量保存即」って聞いたことない?
0994ご冗談でしょう?名無しさん
垢版 |
2019/12/04(水) 01:03:43.20ID:QVbcuAoS
>>987わかる人いませんか?
不可視光の色を吸収するから人間には黒く見るんでしょうか
0995ご冗談でしょう?名無しさん
垢版 |
2019/12/04(水) 01:04:24.41ID:QVbcuAoS
不可視光の色を吸収→不可視光以外の色を吸収
0996ご冗談でしょう?名無しさん
垢版 |
2019/12/04(水) 02:19:13.51ID:v59LwFh8
>>993
ない。君の特許を許す
0999ご冗談でしょう?名無しさん
垢版 |
2019/12/04(水) 11:28:34.15ID:Un9eJr06
1000ご冗談でしょう?名無しさん
垢版 |
2019/12/04(水) 11:28:46.66ID:Un9eJr06
うめ
10011001
垢版 |
Over 1000Thread
このスレッドは1000を超えました。
新しいスレッドを立ててください。
life time: 23日 13時間 33分 14秒
10021002
垢版 |
Over 1000Thread
5ちゃんねるの運営はプレミアム会員の皆さまに支えられています。
運営にご協力お願いいたします。


───────────────────
《プレミアム会員の主な特典》
★ 5ちゃんねる専用ブラウザからの広告除去
★ 5ちゃんねるの過去ログを取得
★ 書き込み規制の緩和
───────────────────

会員登録には個人情報は一切必要ありません。
月300円から匿名でご購入いただけます。

▼ プレミアム会員登録はこちら ▼
https://premium.5ch.net/

▼ 浪人ログインはこちら ▼
https://login.5ch.net/login.php
レス数が1000を超えています。これ以上書き込みはできません。

ニューススポーツなんでも実況